LSN- OB Quiz #1

Pataasin ang iyong marka sa homework at exams ngayon gamit ang Quizwiz!

A pregnant patient would like to know a good food source of calcium other than dairy products. Your best answer is A. Legumes B. Yellow vegetables C. Lean meat D. Whole grains

A. Legumes Rationale: Although dairy products contain the greatest amount of calcium, it is also found in legumes, nuts, dried fruits, and some dark green leafy vegetables. Yellow vegetables are rich in vitamin A. Lean meats are rich in protein and phosphorous. Whole grains are rich in zinc and magnesium

Despite warnings, prenatal exposure to alcohol continues to exceed by far exposure to illicit drugs. A diagnosis of fetal alcohol syndrome (FAS) is made when there are visible markers in each of three categories. Which is category is not associated with a diagnosis of FAS? A. Respiratory conditions B. Impaired growth C. CNS abnormality D. Craniofacial dysmorpologies

A. Respiratory conditions Rationale: Respiratory difficulties are not a category of conditions that are related to FAS. Other abnormalities related to FAS include: organ deformities, genital malformations and kidney and urine defects. Impaired growth is a visible marker for FAS. A CNS abnormality with neurologic and intellectual impairments is a category used to assist in the diagnosis of FAS. An infant with FAS manifests at least two craniofacial abnormalities such as microcephaly, short palpebral fissures, poorly developed philtrum, thin upper lip or flattening of the maxillary

Which part of the mature sperm contains the male chromosomes? A. The head of the sperm B. The middle portion of the sperm C. X-bearing sperm D. The tail of the sperm

A. The head of the sperm Rationale: The head of the sperm contains the male chromosomes that will join the chromosomes of the ovum. The middle portion of the sperm supplies energy for the tail's whip-like action.If an X-bearing sperm fertilizes the ovum, the baby will be female. The tail of the sperm helps propel the sperm toward the ovum.

The level of practice a reasonably prudent nurse provides is called: A. The standard of care B. Risk management C. A sentinel event D. Failure to rescue

A. The standard of care Rationale: Guidelines for standards of care are published by various professional nursing organizations. The standard of care for neonatal nurses is set by the Association of Women's Health, Obstetric, and Neonatal Nurses (AWHONN). The Society of Pediatric Nurses is the primary specialty organization that sets standards for the pediatric nurse. Risk management identifies risks and establishes preventative practices, but it does not define the standard of care. Sentinel events are unexpected negative occurrences. They do not establish the standard of care. Failure to rescue is an evaluative process for nursing, but it does not define the standard of care

Two basic concepts in the philosophy of family centered pediatric nursing care are: A. enabling and empowerment B. empowerment and bias C. enabling and curing D. empowerment and self control

A. enabling and empowerment

A pregnant woman, at 38 weeks of gestation diagnosed with marginal placenta previa, has just given birth to a healthy newborn male. The nurse recognizes that the immediate focus for the care of this woman would be: A. preventing hemorrhage B. relieving pain C. preventing infection D. fostering attachment of the woman with her new son

A. preventing hemorrhage Rationale: hemorrhage is a major potential postpartum complication because the implementation site of the placenta is in the lower uterine segment, which has a limited capacity to contract after birth; infection is another major complication, but it is not the immediate focus of care; B and D are also important but not to the same degree as hemorrhage, which is life threatening

Gestational diabetes mellitus (GDM) is a carbohydrate intolerance that develops during pregnancy. The following are factors associated with a higher risk for GDM. (Select all that apply.) A. Overweight (body mass index [BMI] 25 to 25.9), obese (BMI 30 to 39.9), or morbidly obese (≥40) B. Hypertension C. Member of a high-risk ethnic group D. Maternal age younger than 18 years

A.B.C. Overweight (body mass index [BMI] 25 to 25.9), obese (BMI 30 to 39.9), or morbidly obese (≥40); Hypertension; Member of a high-risk ethnic group Rationale: Factors associated with a higher risk for GDM are similar to those for type 2 diabetes, including a high maternal BMI, hypertension, and membership among a high-risk ethnic group (Hispanic, African, Native American, South or East Asian, or Pacific Islands ancestry). Maternal age younger than 18 years is not a risk factor for developing GDM.

Administering an opioid antagonist to a woman who is opioid dependent will result in the opioid abstinence syndrome. The nurse would recognize which of the following clinical manifestations as evidence that this syndrome is occurring? (Select all that apply) A. Yawning B. Piloerection C. Anorexia D. Coughing E. Dry skin, eyes, and nose F. Miosis

A.B.C. Yawning; Piloerection ; Anorexia Rationale: D.E.F. are not associated with opioid abstinence syndrome

A nurse is evaluating a woman's breastfeeding technique. Which of the following actions would indicate that the woman needs further instruction regarding breast feeding to ensure success (Select all that apply) A. Washes her breasts and nipples thoroughly with soap and water twice a day B. Massages a small amount of breast milk into her nipple and areola before and after each feeding C. Lines her bra with a thick plastic lined pad to absorb leakage D. Positions baby supporting back and shoulder securely and then brings her breast toward the baby, putting the nipple in the baby's mouth E. Feeds her baby every 2 to 3 hours F. Inserts her finger into the corner of her baby's mouth between the gums before removing him from the breast

A.C.D. Washes her breasts and nipples thoroughly with soap and water twice a day; Lines her bra with a thick plastic lined pad to absorb leakage; Positions baby supporting back and shoulder securely and then brings her breast toward the baby, putting the nipple in the baby's mouth Rationale: nipples should not be washed using soap; plastic liners can keep nipples and areola moist and increase the risk for tissue breakdown; bring baby to breast, not breast to baby

Systemic lupus erythematosus is a chronic multisystem inflammatory disease that affects skin, joints, kidney, lungs, CNS, liver, and other organs. Maternal risks include (select all that apply) A. Premature rupture of membranes (PROM) B. Fetal death resulting in stillbirth C. Hypertension D. Preeclampsia E. Renal complications

A.C.D.E. Premature rupture of membranes (PROM); Hypertension; Preeclampsia; Renal complications Rationale: PROM, hypertension, preeclampsia, and renal complications are all maternal risks associated with SLE. Stillbirth and prematurity are fetal risks of SLE

During pregnancy there are a number of changes that occur as a direct result of the presence of the fetus. Which of these adaptations meet this criteria? (Select all that apply) A. Leukorrhea B. Development of the operculum C. Quickening D. Ballottement E. Lightening

A.C.E. Leukorrhea; Quickening; Lightening Rationale: Leukorrhea is a white or slightly gray vaginal discharge that develops in response to cervical stimulation by estrogen and progesterone. Quickening is the first recognition of fetal movements or feeling life. Quickening is often described as a flutter and is felt earlier in multiparous women that in primiparas. Lightening occurs when the fetus begins to descent into the pelvis. This occurs two weeks before labor in the nullipara and at the start of labor in multipara. Mucous fills the cervical canal creating a plug otherwise known as the operculum. Thee opereculum acts as a barrier against bacterial invasion during the pregnancy. Passive movement of thee unengaged fetus is referred to as ballottement

A woman's last menstrual period was June 10. Her estimated date of delivery (EDD) is A. April 7 B. March 17 C. March 27 D. April 17

B. March 17 Rationale: April 7 would be subtracting 2 months instead of 3 months and then subtracting 3 days instead of adding 7 days. To determine the EDD, the nurse uses the first day of the last menstrual period (June 10), subtract 3 months (March 10) and adds 7 days (March 17). March is thee correct month, but instead of adding 7 days, 17 days were added (C). April 17 is subtracting 2 months instead of 3 months

For which of the infectious diseases can a woman be immunized? A. Toxoplamosis B. Rubella C. Cytomegalovirus D. Herpes virus type 2

B. Rubella Rationale: There is no vaccine available for toxoplasmosis. Rubella is the only infectious disease for which a vaccine is available. There is no vaccine available for cytomegalovirus. There is no vaccine available for herpes virus type 2

Which nursing intervention is an independent function of the nurse? A. Administering oral analgesics B. Teaching the woman perineal care C. Requesting diagnostic studies D. Providing wound care to a surgical incision

B. Teaching the woman perineal care Rationale: Administering oral analgesics is a dependent function; it is initiated by a physician and carried out by the nurse. Nurses are now responsible for various independent functions, including teaching, counseling, and intervening in nonmedical problems. Interventions initiated by the physician and carried out by the nurse are called dependent functions. Requesting diagnostic studies is a dependent function. Providing wound care is a dependent function; it is usually initiated by the physician through direct orders or protocol

A primigravida is being monitored in her prenatal clinic for preeclampsia. What finding should concern her nurse? A. Blood pressure increase to 138/86mmHg B. Weight gain of 0.5kg during the past 2 weeks C. A dipstick value of +3 for protein in her urine D. Pitting pedal edema at the end of the day

C. A dipstick value of +3 for protein in her urine Rationale: Generally, hypertension is defined as a BP of 140/90 mmHg or an increase in systolic pressure of 30 mmHg or 15mmHg diastolic pressure. Preeclampsia may be manifested as a rapid weight gain of more than 2kg in 1 week. Proteinuria is defined as a concentration of 1+ or greater via dipstick measurement. A dipstick value of 3+ should alert the nurse that additional testing or assessment should be made. Edema occurs in many normal pregnancies as well as in women with preeclampsia. Therefore, the presence of edema is no longer considered diagnostic of preeclampsia

When planning a parenting class, the nurse should explain that the leading cause of death in children 1 to 4 years of age in the United States is A. Premature birth B. Congenital anomalies C. Accidental death D. Respiratory tract illness

C. Accidental death Rationale: Disorders of short gestation and unspecified low birth weight make up one of the leading causes of death in neonates. One of the leading causes of infant death after the first month of life is congenital anomalies. Accidents are the leading cause of death in children ages 1 to 19 years. Respiratory tract illnesses are a major cause of morbidity in children

What finding on a prenatal visit at 10 weeks could suggest a hydatidiform mole? A. Complaint of frequent mild nausea B. Blood pressure of 120/80 mm Hg C. Fundal height measurement of 18 cm D. History of bright red spotting for 1 day, weeks ago

C. Fundal height measurement of 18 cm Rationale: Nausea increases in a molar pregnancy because of increase production of hCG. A woman with a molar pregnancy may have early onset pregnancy induced hypertension. The uterus in a hydatidiform molar pregnancy is often larger than would be expected on the basis of the duration of the pregnancy. The history of bleeding is normally described as being brownish

In the United States, the leading cause of maternal mortality is which one of the following? A. Unsafe abortion B. Infection C. Gestational hypertension D. diabetes

C. Gestational hypertension

What condition indicates concealed hemorrhage in an abruptio placentae? A. Decrease in abdominal pain B. Bradycardia C. Hard, boardlike abdomen D. Decrease in fundal height

C. Hard, boardlike abdomen Rationale: Abdominal pain may increase. The patient will have shock symptoms that include tachycardia. Concealed hemorrhage occurs when the edges of the placenta do not separate. The formation of a hematoma behind the placenta and subsequent infiltration of the blood into the uterine muscle results in a very firm, boardlike abdomen. The fundal height will increase as bleeding occurs.

A woman from Southeast Asia has been in the United States for 6 months. She just delivered her first child. The nurse notices that the woman does not drink any of the fluids that are offered (ice water, iced tea, and juices) and mentions this concern at the team meeting. The nurse manager states the problem may be cultural and that the woman may consider the postpartum period to be "cold." If this is correct, which fluids may be acceptable to the woman? A. Sour juices (such as grapefruit) B. Sweet tea or sweet juices (such as apple) C. Hot tea, lukewarm or hot water D. She will not drink any fluids during this time and will need to meet her fluid needs through foods with high water content.

C. Hot tea, lukewarm or hot water Rationale: Because the postpartum period is "cold," the woman will not drink cold fluids. She may feel this would cause the loss of more heat and would have ill effects on her health. Drinking sour juice or sweet tea does not prevent the loss of heat, which the woman is trying to prevent. Fluids are important during the early postpartum period to prevent or correct dehydration from the labor process. It is important for the nurse to assist the woman to meet her fluid needs without interfering with her cultural beliefs.

To maintain the corpus luteum and the continuing supply of estrogen and progesterone, the zygote secretes which hormone? A. Prolactin B. Luteinizing hormone C. Human chorionic gonadotropin D. Oxytocin

C. Human chorionic gonadotropin Rationale: The cells of the zygote secrete human chorionic gonadotropin. The hCG feeds back into the ovum to prolong the corpus luteum. Prolactin is produced in the pituitary glands and is necessary for breast milk production. Luteinizing hormone is produced in the pituitary glands and is necessary for the release of the mature ovum at ovulation. Oxytocin is produced in the pituitary glands and is necessary for contractions during labor and the let-down reflex with breastfeeding.

A 27 year old pregnant woman is seeing a nurse-midwife for prenatal care. Her first baby was born by cesarean because the baby was too large to fit through the woman's pelvis. She has also developed gestational diabetes during this pregnancy. When discussing with the woman her options for places of birth, what is the best choice for her? A. Home birth B. Birth center C. LDRP in a hospital setting D. It does not matter

C. LDRP in a hospital setting Rationale: Home births and free standing birthing centers should be used for patients with very low risk for complications. The woman's past history and present complication with this pregnancy place her in a high risk category. Therefore, she needs to be in a setting where emergency care is quickly available. Because of her complications, the woman is not a candidate for home birth or a birth center. The woman needs to be in a setting where emergency care is quickly available

During a centering pregnancy group meeting, the nurse teaches patients that the fetal period is best described as one of A. Development of basic organ systems B. Resistance of organs to damage from external agents C. Maturation of organ systems D. Development of placental oxygencarbon dioxide exchange

C. Maturation of organ systems Rationale: Basic organ systems are developed during the embryonic period. The organs are always at risk for damage from external sources; however, the older the fetus, the more resistant the organs will be. The greatest risk is when the organs are developing. During the fetal period, the body systems grow in size and mature in function to allow independent existence after birth. The placental system is complete by week 12, but that is not the best description of the fetal period.

What comment by a new mother exhibits understanding of her toddlers response to a new sibling? A. I can't believe he is sucking his thumb again. B. He is being difficult, and I don't have time to deal with him C. My husband is going to stay with the baby so I can take our son to the park tomorrow. D. When we brought the baby home, we made our son stop sleeping in the crib

C. My husband is going to stay with the baby so I can take our son to the park tomorrow. Rationale: It is normal for a child to regress when a new sibling is introduced into the home. The toddler may have feelings of jealousy and resentment toward the new baby taking the attention from him. Frequent reassurance of parental love and affection are important. It is important for a mother to seek time alone with her toddler to reassure him that he is loved. Changes in sleeping arrangements should be made several weeks before the birth so that the child does not feel displaced by the new baby.

A traditional family structure in which male and female partners and their children live as an independent unit is known as a(n)_____ family. A. Extended B. Binuclear C. Nuclear D. Blended

C. Nuclear Rationale: Extended families include other blood relatives in addition to the parents. A binuclear family involves two households. Approximately two thirds of US households meet the definition of a nuclear family. This is also known as the traditional family. A blended family is reconstructed after divorce and involves the merger of two families

The nurse has just administered metoclopramide (Reglan) to a woman in labor. Which of the following would be an expected effect of this medication? A. Analgesia B. Nausea and vomiting C. Potentiation of opioid analgesics D. Respiratory depression

C. Potentiation of opioid analgesics Rationale: metoclopramide is used to prevent or treat nausea and vomiting, it does not provide analgesia nor cause respiratory depression; it can increase the effectiveness of opioid analgesics

The primary symptom present in abruptio placentae that distinguishes it from placenta previa is A. Vaginal bleeding B. Rupture of membranes C. Presence of abdominal pain D. Changes in maternal vital signs

C. Presence of abdominal pain Rationale: Both may have vaginal bleeding. Rupture of membranes may occur with both conditions. Pain in abruptio placentae occurs in response to increased pressure behind the placenta and within the uterus. Placenta previa manifests with painless vaginal bleeding. Maternal vital signs may change with both if bleeding is pronounced

One of the most effective methods for preventing venous stasis is to A. Wear elastic stockings in the afternoon B. Sleep with the foot of the bed elevated C. Rest often with the feet elevated D. Sit with the legs crossed

C. Rest often with the feet elevated Rationale: Elastic stockings should be applied before lowering the legs in the morning. Elevating the legs at night may cause pressure on the diaphragm and increase breathing problems. Elevating the feet and legs improves venous return and prevents venous stasis. Sitting with legs crossed will decrease circulation in the leg and increase venous stasis

When is the best time to determine gestational age based on biparietal diameter through ultrasound? A. 4 to 6 weeks B. 5 to 7 weeks C. 7 to 10 weeks D. 12 to 20 weeks

D. 12 to 20 weeks Rationale: During this time frame, the gestation age is determined by measuring the crown to rump length (A.B.C.). The biparietal diameter, a method of assessing gestational age during the last half of pregnancy, is most accurate from 12 to 20 weeks

Infant mortality for infants in the United States less than 1 year of age: A. has dramatically increased B. is the same as it is in developing countries C. is the same as the rate in Japan D. has decreased over the past few years

D. has decreased over the past few years

Dizygotic twins develop from A. a single fertilized ovum and are always of the same sex. B. a single fertilized ovum and may be the same sex or different sexes. C. two fertilized ova and are the same sex. D. two fertilized ova and may be the same sex or different sexes.

D. two fertilized ova and may be the same sex or different sexes. Rationale: Dizygotic twins are two different zygotes, each conceived from a single ovum and a single sperm. They may be both male, both female, or one male and one female. Monozygotic twins are from a single fertilized ovum and are always the same sex. Dizygotic twins come from two fertilized ovum and may be the same sex, but they may also be different sexes.

A woman at 30 weeks of gestation with a Class II cardiac disorder calls her primary health care provider's office and speaks to the nurse practitioner. She tells the nurse that she has been experiencing a frequent, moist cough for the past few days. In addition, she has been feeling more tired and is having difficulty completing her routine activities as a result of some difficulty with breathing. The nurse's best response would be: A. "Have someone bring you to the office so we can assess your cardiac status." B. "Try to get more rest during the day because this is a difficult time for your heart." C. "Take an extra diuretic tonight before you go to bed, since you mat be developing some fluid in your lungs." D. "Ask your family to come over an do your housework for the next few days so you can rest."

A. "Have someone bring you to the office so we can assess your cardiac status." Rationale: this woman is exhibiting signs of cardiac decompensation; further information regarding her cardiac status is required to determine what further action would be needed

A woman who is 12 weeks' pregnant comes to the clinic for counseling concerning an abortion. The nurse is aware that the woman needs further teaching when she says A. "I know that this pregnancy is just a group of round cells at this point." B. "I think you can hear the baby's heartbeat now." C. "You can tell if the baby is a girl or boy." D. "The baby's intestines are inside the abdomen."

A. "I know that this pregnancy is just a group of round cells at this point." Rationale: Early in the zygote stage of development, the fertilized ovum divides into cells. They resemble a ball at that point. By 12 weeks, the fetus has arms, legs, a head, and major organs. The heartbeat can be detected as early as 8 weeks with ultrasound. Male and female external genitalia can be distinguished by appearance during the 12th week. The intestines are originally contained within the umbilical cord because the liver and kidneys occupy most of the abdominal cavity. By the 10th week, the intestines are contained within the abdominal cavity.

During a presentation on prenatal care, the student nurse stated, "In 2000 the maternal mortality rate for African American women was 22." The number "22" in this statement means there have been A. 22 maternal deaths per 100,000 live births B. 22 serious maternal illnesses per 1000 live births C. 88 maternal deaths per 100 live births D. 88 serious maternal illness per 100 live births

A. 22 maternal deaths per 100,000 live births Rationale: Maternal mortality rate is based on the number of maternal deaths from childbirths or complications of pregnancy, childbirth, or puerperium per 100,000 live births. It does not deal with serious illnesses.

The nurse is teaching a woman in her second trimester about an upcoming ultrasonography exam. The nurse knows her teaching has been successful when the woman states A. "I will drink several glasses of water about an hour before I come in for the test." B. "I will empty my bladder just before the test." C. "I will not eat or drink anything for 8 hours prior to coming in for the test." D. "I will plan on staying at the doctor's office for about 2 hours after the test so you can check to make sure the baby was not harmed during the procedure."

A. "I will drink several glasses of water about an hour before I come in for the test." Rationale: Drinking several glasses of clear fluid 1 hour before the time of the examination will produce a full bladder. The bladder will displace the intestines and elevate the uterus for better visibility. It is not necessary to be NPO for 8 hours prior to this exam. No fetal post-procedure assessments are necessary.

After you complete your nutritional counseling for a pregnant woman, you ask her to repeat your instructions so that you can assess her understanding of the instructions given. Which statement indicates that she understands the role of protein in her pregnancy? A. "Protein will help my baby grow." B. "Eating protein will prevent me from becoming anemic." C. "Eating protein will make my baby have strong teeth after he is born." D. "Eating protein will prevent me from being diabetic."

A. "Protein will help my baby grow." Rationale: Protein is the nutritional element. basic to growth. An adequate protein intake is essential to meeting the increasing demands of pregnancy. These demands arise from the rapid growth of the fetus; the enlargement of the uterus, mammary glands, and placenta; the increase in the maternal blood volume; and the formation of amniotic fluid. Iron intake prevents anemia. Calcium intake is needed for fetal bone and tooth development. Glycemic control is needed in diabetics; protein is one nutritional factor to consider, but this is not the primary role of protein intake

The nurse is explaining the results of a maternal serum alpha-fetoprotein screening test to the woman. The nurse knows the woman does not understand the teaching if she says A. "Since the levels were within normal limits, I know the baby does not have any anomalies." B. "I know that the levels are high, but that does not always mean something is wrong with the baby." C. "Since I am not sure about the date of my last menstrual period, the test result cannot be accurately interpreted." D. "Since the levels are low, my baby may have Down syndrome."

A. "Since the levels were within normal limits, I know the baby does not have any anomalies." Rationale: Alpha-fetoprotein levels are a screening test. Some fetal defects are covered by skin and do not produce elevated levels of AFP. MSAFP levels are a screening test and must be viewed as the first step in a series of diagnostic procedures that are indicated if abnormal concentrations are found. Inaccurate estimation of gestational age can result in false-positive or false-negative results. Low levels are an indication of Down syndrome; more testing is indicated.

A 28-year-old postpartum patient tells the nurse, "I won't be going home for about 2 weeks." To clarify the statement, the nurse may state A. "Tell me what you mean when you say you won't be going home for 2 weeks." B. "I know you are excited." C. "So you will be here with us for 2 more weeks?" D. "Go on ...."

A. "Tell me what you mean when you say you won't be going home for 2 weeks." Rationale: To clarify a statement is to ensure understanding and accuracy of the message. The nurse can clarify a message by asking for further specific information.

The nurse is teaching a young adolescent pregnant woman about proper diet. The nurse starts by collecting a 24-hour dietary recall. Which statement by the nurse would increase the likelihood of compliance to dietary changes by the woman? A. "This diet looks good but is low in calcium. Which of these high-calcium foods do you think you could include in your diet?" B. "This diet is low in calcium and you need to drink more milk. How about one glass for breakfast and one for lunch?" C. "You must not like milk. Let's try eating cheese instead." D. "You must drink more milk. The baby's health may be jeopardized if you don't start taking in more calcium now.

A. "This diet looks good but is low in calcium. Which of these high-calcium foods do you think you could include in your diet?" Rationale: This statement allows the woman to have control over the changes necessary. When she is in control, compliance increases. Statements that belittle the woman or place blame will decrease the likelihood of compliance.

An infant was born with multiple physical defects. A few hours after the birth the mother tells the nurse: "I told the doctor over and over I thought something was wrong, but he never did anything about it and now it is too late!" The best response by the nurse would be A. "You are angry because no one listened to you?" B. "There was nothing the doctor could do to prevent this from happening." C. "At least the baby is alive—many mothers lose their babies." D. "You need to get over being angry and concentrate on what is best for the baby."

A. "You are angry because no one listened to you?" Rationale: Anger is an expected emotion from the mother. The nurse should encourage her to express her feelings. The other comments belittle the feelings of the woman andwill block further communication.

The formula used to guide a time out as a disciplinary method is A. 1 minute per each year of the child's age B. To relate the length of the time out to the severity of the behavior C. never use time out for a child younger than 4 years of age D. To follow the time out with a treat

A. 1 minute per each year of the child's age Rationale: It is important to structure time out in a time frame that allows the child to understand why he or she has been removed from the environment. Relating time to a behavior is subjective and is inappropriate when the child is very young. Time out can be used with the toddler. Negative behavior should not be reinforced with a positive action

The mother of a 5 year old female inpatient on the pediatric unit asks the nurse if she could provide information regarding the recommended amount of television viewing time for her daughter. The nurse responds that the appropriate amount of time a child should be watching television is A. 1-2 hours per day B. 2-3 hours per day C. 3-4 hours per day D. 4 hours or more

A. 1-2 hours per day Rationale: The American Academy of Pediatrics (AAP) encourages all parents to monitor their childrens media exposure and limit screen time to no more than 1 to 2 hours per day. The AAP also recommends that parents remove televisions and computers from their childrens bedrooms and monitor programs that have sexual or violent content. Two hours per day is the outer limit of media exposure according to the AAP. Three to four hours per day is too much television per the AAP guidelines. In this situation, parents need to more carefully monitor the amount of television viewing time. Watching television for 4 hours or more is an excessive amount of screen time per the AAP guidelines. In this situation, parents need to more carefully monitor the amount of television viewing times

Which man is most likely to have abnormal sperm formation resulting in infertility? A. A 20 year old man with undescended testicles B. An uncircumcised 40 year old man C. A 35 year old man with previously treated sexually transmitted disease D. A 16 year old adolescent who is experiencing nocturnal emissions

A. A 20 year old man with undescended testicles Rationale: For normal sperm formation, a man's testes must be cooler than his core body temperature. Circumcision does not prevent fertility. Scar tissue in the fallopian tubes as a result of a sexually transmitted disease can be a cause of infertility in women. Nocturnal emissions of seminal fluid are normal and expected in teenagers

The average man is taller than the average woman at maturity because of A. A longer period of skeletal growth. B. Earlier development of secondary sexual characteristics. C. Earlier onset of the growth spurt. D. Starting puberty at an earlier age.

A. A longer period of skeletal growth. Rationale: The man's greater height at maturity is the combined result of beginning the growth spurt at a later age and continuing it for a longer period. Girls develop earlier than boys. Boys growth spurts start at a later age. Girls start puberty approximately six months to 1 year earlier than boys

In helping bereaved parents cope and move on, nurses should keep in mind that A. A perinatal or parental grief support group is more likely to be helpful if the needs of the parents are matched with the focus group B. When pictures of the infant are take for keepsakes, no close ups should be taken of any congenital anomalies C. No significant differences exist in grieving individuals from various cultures, ethnic groups, and religions D. In emergency situations, nurses who are so disposed must resist the temptation to baptize the infant in the absence of a priest or minister

A. A perinatal or parental grief support group is more likely to be helpful if the needs of the parents are matched with the focus group Rationale: For example, a religious based group may not work for nonreligious parents. Close up pictures of the baby must be taken as the infant was, congenital anomalies and all. Although death and grieving are events shared by all people, mourning rituals, traditions, and taboos, vary by culture, ethnicity, and religion. Differences must be respected. Baptism for some religious groups can be performed by a layperson, such as a nurse, in an emergency situation when a priest is not available

Which nutritional recommendation about fluids is accurate? A. A woman's daily intake should be 8-10 cups B. Coffee should be limited to no more than 2 cups, but tea and cocoa can be consumed without worry C. Of the artificial sweeteners, only aspartame has not been associated with any maternity health concerns D. Water with fluoride is especially encouraged because it reduces the childs risk of tooth decay

A. A woman's daily intake should be 8-10 cups Rationale: 8-10 cups is the standard for fluids; however, they should b the right fluids. All beverages containing caffeine, including tea, cocoa, and some soft drinks, should be avoided or should be drunk only in limited amounts. Artificial sweeteners, including aspartame, have no ill effects on the normal mother or fetus. However, mothers with phenylketonuria (PKU) should avoid aspartame. No evidence indicates that prenatal fluoride consumption reduces childhood tooth decay. However, it still helps the mother

Which assessment finding should convince the nurse to hold the next dose of magnesium sulfate? A. Absence of deep tendon reflexes B. Urinary output of 100mL total for the previous 2 hours C. Respiratory rate of 14 breaths/min D. Decrease in blood pressure from 160/100 to 140/85

A. Absence of deep tendon reflexes Rationale: Because absence of deep tendon reflexes is a sign of magnesium toxicity, the next scheduled dose should not be administered. Calcium gluconate is the antidote that should be administered. An hourly output of less than 30mL could indicate toxicity. A respiratory rate of less than 12 breaths/min could indicate toxicity. Decrease in blood pressure is an expected side effect of magnesium sulfate

In counseling a patient who has decided to relinquish her baby for adoption, the nurse should A. Affirm her decision while acknowledging her maturity in making it B. Question her about her feelings regarding adoption C. Tell her she can always change her mind about adoption D. Ask her if anyone is coercing her into the decision to relinquish her baby

A. Affirm her decision while acknowledging her maturity in making it Rationale: A supportive, affirming approach by the nurse will strengthen the patients resolve and help her to appreciate the significance of the event. The teen needs help in coping with her feelings about this decision. It is important that the teenager is treated as an adult, with the assumption that she is capable of making an important decision on her own

Which of the following group descriptions is most accurate regarding those persons who should participate in preconception counseling? A. All women and their partners as they make decisions about their reproductive future, including becoming parents B. All women during their childbearing years C. Sexually active women who do not use birth control D. Women with chronic illnesses such as diabetes who are planning to get pregnant

A. All women and their partners as they make decisions about their reproductive future, including becoming parents Rationale: all women, not just specific groups, should participate in preconception care 1 year before planning to get pregnant

Prenatal testing for the human immunodeficiency virus HIV is recommended for which women? A. All women, regardless of risk factors B. A woman who has had more than one sexual partner C. A woman who has had a sexually transmitted infection D. A woman who is monogamous with her partner

A. All women, regardless of risk factors Rationale: An HIV test is recommended for all women, regardless of risk factors. The incidence of perinatal transmission from and HIV positive mother to her fetus ranges from 25-35%. Women who test positive for HIV can then be treated. All women should be tested for HIV, although this patient is at an increased risk of contacting the disease (B). Regardless of past sexual history, all women should have an HIV test completed prenatally. Although this patient is apparently monogamous, an HIV test is still recommended

The priority nursing intervention when admitting a pregnant woman who has experienced a bleeding episode in late pregnancy is to A. Assess fetal heart rate (FHR) and maternal vital signs B. Perform a venipuncture for hemoglobin and hematocrit levels C. Place clean disposable pads to collect any drainage D. Monitor uterine contractions

A. Assess fetal heart rate (FHR) and maternal vital signs Rationale: Assessment of the FHR and maternal vital signs will assist the nurse in determining the degree of the blood loss and its effect on the mother and fetus. The most important assessment is to check mother/fetal well being. The blood levels can be obtained later. It is important to assess future bleeding, but the top priority is mother/fetal well being. Monitoring uterine contractions is important, but not the top priority.

What part of the nursing process includes the collection of data on vital signs, allergies, sleep patterns, and feeding behaviors? A. Assessment B. Planning C. Intervention D. Evaluation

A. Assessment Rationale: Assessment is the gathering of baseline data. Planning is based on baseline data and physical assessment. Implementation if the initiation and completion of nursing interventions. Evaluation is the last step in the process and involves determining whether the goals were met

The nurse is admitting a woman to the labor unit. During the admission procedure, the nurse obtains the woman's blood pressure, pulse, respirations, temperature, and fetal heart rate. The nurse is using which part of the nursing process? A. Assessment B. Planning C. Implementation D. Evaluation

A. Assessment Rationale: The assessment phase of the nursing process is a systematic collection of relevant data. Vital signs are considered patient data. The planning stage includes setting priorities, establishing goals, and planning interventions. The implementation stage is carrying out the plan of care. The evaluation stage is determining if the goals were met.

What order should the nurse expect for a patient admitted with a threatened abortion? A. Bed rest B. Ritodrine IV C. NPO D. Narcotic analgesia every 3hours prn

A. Bed rest Rationale: Decreasing the woman's activity level may alleviate the bleeding and allow the pregnancy to continue. Ritodrine is not the first drug of choice for tocolytic medications. There is no reason for having the woman NPO. At times, dehydration may produce contractions, so hydration is important. Narcotic analgesia will not decrease the contractions. It may mask the severity of the contractions

A baby is born with blood type AB. The father is type A, and the mother is type B. The father asks why the baby has a different blood type from those of his parents. The nurses answer should be based on the knowledge that A. Both A and B blood types are dominant B. The baby has a mutation of the parents blood types C. Type A is recessive and links more easily with type B. D. Types A and Bare recessive when linked together

A. Both A and B blood types are dominant Rationale: Types A and B are equally dominant, and the baby can thus inherit one from each parent. The infant has inherited both types from the parents; it is not a mutation. Both types A and B are equally dominant.

Approximately 12% to 26% of all clinically recognized pregnancies end in miscarriage. Which is the most common cause of spontaneous abortion? A. Chromosomal abnormalities B. Infections C. Endocrine imbalance D. Immunologic factors

A. Chromosomal abnormalities Rationale: At least 60% of pregnancy losses result from chromosomal abnormalities that are incompatible with life. Maternal infection may be a cause of early miscarriage. Endocrine imbalances such as hypothyroidism or diabetes are possible causes for early pregnancy loss. Women who have repeated early pregnancy losses appear to have immunologic factors that play a role in spontaneous abortion incidents

The placenta allows exchange of oxygen, nutrients, and waste products between the mother and fetus by A. Contact between maternal blood and fetal capillaries within the chorionic villi B. Interaction of maternal and fetal pH levels within the endometrial vessels C. A mixture of maternal and fetal blood within the intervillous spaces D. Passive diffusion of maternal carbon dioxide and oxygen into the fetal capillaries

A. Contact between maternal blood and fetal capillaries within the chorionic villi Rationale: Fetal capillaries within the chorionic villi are bathed with oxygen- and nutrient-rich maternal blood within the intervillous spaces. The endometrial vessels are part of the uterus. There is no interaction with the fetal blood at this point. Maternal and fetal blood do not normally mix. Maternal carbon dioxide does not enter into the fetal circulation

With regard to the association of maternal diabetes and other risk situations affecting mother and fetus, nurses should be aware that: A. Diabetic ketoacidosis (DKA) can lead to fetal death at any time during pregnancy. B. Hydramnios occurs approximately twice as often in diabetic pregnancies. C. Infections occur about as often and are considered about as serious in diabetic and nondiabetic pregnancies. D. Even mild to moderate hypoglycemic episodes can have significant effects on fetal well-being.

A. Diabetic ketoacidosis (DKA) can lead to fetal death at any time during pregnancy. Rationale: Prompt treatment of DKA is necessary to save the fetus and the mother. Hydramnios occurs 10 times more often in diabetic pregnancies. Infections are more common and more serious in pregnant women with diabetes. Mild to moderate hypoglycemic episodes do not appear to have significant effects on fetal well being

The multiple marker screen is used to assess the fetus for which condition? A. Downs syndrome B. Diaphragmatic hernia C. Congenital cardiac abnormality D. Anencephaly

A. Downs syndrome Rationale: The maternal serum level of alpha-fetoprotein is used to screen for Trisomy 18 or 21, neural tube defects, and other chromosomal anomalies. The quadruple marker test dose not detect this fetal anomaly. Additional testing, such as ultrasonography would be required to diagnose diaphragmatic hernia. Congenital cardiac abnormality would most likely be identified during and ultrasound examination. The quadruple marker test would not detect anencephaly

A nurse determines that a child consistently displays predictable behavior and is regular in performing daily habits. Which temperament is the child displaying? A. Easy B. Slow to warm up C. Difficult D. Shy

A. Easy Rationale: Children with an easy temperament are even tempered, predictable, and regular in their habits. They react positively to new stimuli. The slow to warm up temperament type prefers to be inactive and moody. A high activity level and adapting slowly to new stimuli are characteristics of a difficult temperament. Shyness is a personality type and not a characteristic of temperament

When the pregnant diabetic woman experiences hypoglycemia while hospitalized, the nurse should intervene by having the patient: A. Eat six saltine crackers. B. Drink 8 oz of orange juice with 2 tsp of sugar added. C. Drink 4 oz of orange juice followed by 8 oz of milk. D. Eat hard candy or commercial glucose wafers.

A. Eat six saltine crackers. Rationale: Crackers provide carbohydrates in the form of polysaccharides. Orange juice and sugar will increase the blood sugar, but not provide a slow burning carbohydrate to sustain the blood sugar. Milk is a disaccharide and orange juice is a monosaccharide. This will provide an. increase in blood sugar but will not sustain to level. This provides only monosaccharides (D)

Which of these is a secondary sexual characteristic? A. Female breast development B. Production of sperm C. Maturation of ova D. Secretion of gonadotropin-releasing hormone

A. Female breast development Rationale: A secondary sexual characteristic is one not directly related to reproduction, such as development of the characteristic female body form. Production of sperm is directly related to reproduction and is a primary sexual characteristic. Maturation of ova is directly related to reproduction and is a primary sexual characteristic. Secretion of hormones is directly related to production and is a primary sexual characteristic

Which is a secondary sexual characteristic? A. Female breast development B. Production of sperm C. Maturation of ova D. Secretion of gonadotropin-releasing hormone

A. Female breast development Rationale: A secondary sexual characteristic is one not directly related to reproduction, such as development of the characteristic female body form. Sperm production, maturation of ova, and secretion of gonadotropin-releasing hormone are directly related to reproduction and therefore are not secondary sexual characteristics.

Rh incompatibility can occur if the woman is Rh negative and her A. Fetus is Rh positive B. Husband is Rh positive C. Fetus is Rh negative D. Husband and fetus are both Rh negative

A. Fetus is Rh positive Rationale: For Rh incompatibility to occur, the mother must be Rh negative and her fetus Rh positive. The husbands Rh factor is a concern only as it relates to the possible Rh factor of the fetus. If the fetus is Rh negative, the blood types are compatible and no problems should occur. If the fetus is Rh negative, the blood type with the mother is compatible. The husband's blood type does not enter into the problem

When is the most accurate time to determine gestational age through ultrasound? A. First trimester B. Second trimester C. Third trimester D. There is no difference in accuracy among the trimesters

A. First trimester Rationale: During the first trimester, measurement of the crown-rump length of the embryo is the most reliable indicator of gestational age. Gestational age determination by ultrasonography is increasingly less accurate after the first trimester.

What is a goal of a patient with the following nursing diagnosis: Imbalanced Nutrition: Less than Body Requirements related to diet choices inadequate to meet nutrient requirements of pregnancy? A. Gain a total of 30lb B. Take daily supplements consistently C. Decrease intake of snack foods D. Increase intake of complex carbohydrates

A. Gain a total of 30lb Rationale: A weight gain of 30lb is one indication that the patient has gained a sufficient amount for the nutritional needs of pregnancy. A daily supplement is not the best goal for this patient. It does not meet the basic need of proper nutrition during pregnancy. Decreasing snack foods may be the problem and should be assessed. However, assessing the weight gain is the best method of monitoring nutritional intake for this pregnancy. Increasing the intake of complex carbohydrates is important for this patient, but monitoring the weight gain should be the end goal.

Which major neonatal complication is carefully monitored after the birth of the infant of a diabetic mother? A. Hypoglycemia B. Hypercalcemia C. Hypobilirubinemia D. Hypoinsulinemia

A. Hypoglycemia Rationale: The neonate is at higher risk for hypoglycemia because fetal insulin production is accelerated during pregnancy to metabolize excessive glucose from the mother. At birth, the maternal glucose supply stops and the neonatal insulin exceeds the available glucose, leading to hypoglycemia. Hypocalcemia and hyperbilirubinemia are complications of infants of a diabetic mother. Because fetal insulin production is accelerated during pregnancy, the neonate shows hyperinsulinemia.

Which major neonatal complication is carefully monitored after the birth of the infant of a diabetic mother? A. Hypoglycemia B. Hypercalcemia C. Hypobilirubinemia D. Hypoinsulinemia

A. Hypoglycemia Rationale: The neonate is at the highest risk for hypoglycemia because fetal insulin production is accelerated during pregnancy to metabolize excessive glucose from the mother. At birth, the maternal glucose supply stops and the neonatal insulin exceeds the available glucose, leading to hypoglycemia. Hypocalcemia is associated with preterm birth, birth trauma, and asphyxia, all common problems of the infant of a diabetic mother. Excess erythrocytes are broken down after birth, releasing large amounts of bilirubin into the neonates circulation, which results in hyperbilirubinemia. Because fetal insulin production is accelerated during pregnancy, the neonate shows hyperinsulinemia

Which comment by a woman in her first trimester indicates ambivalent feelings? A. I wanted to become pregnant, but I'm scared about being a mother B. I haven't felt well since this pregnancy began C. I'm concerned about the amount of weight gained D. My body is changing so quickly

A. I wanted to become pregnant, but I'm scared about being a mother Rationale: Ambivalence refers to conflicting feelings. This does not reflect conflicting feelings. By expressing concerns over a normal occurrence, the woman is trying to confirm the pregnancy. The woman is trying to confirm the pregnancy when she expresses concerns over normal pregnancy changes. She is not expressing conflicting feelings

Toxoplasmosis is a protozoal infection transmitted through organisms in raw and undercooked meat or through contact with contaminated cat feces. While providing education to the pregnant woman, the nurse evaluates the learning and understands that the patient requires further instruction when she states A. I will be certain to empty the litter boxes regularly B. I won't eat raw eggs C. I had better wash all of my fruits and vegetables D. I need to be cautious when cooking meat

A. I will be certain to empty the litter boxes regularly Rationale: The patient should avoid contact with materials that are possibly contaminated with cat feces while pregnant. This includes cat litter boxes, sand boxes, and garden soil or handling animals. The patient should avoid undercooked eggs and unpasteurized milk. All fruits and vegetables should be washed thoroughly before eating. Meat should be cooked thoroughly to an internal temperature of at least 160F or as high as 180F for poultry. All surfaces should be washed after they come into contact with uncooked meat. The patient should be instructed not to use the same utensils or cutting board for meat and produce

In assessing knowledge of a pregestational woman with type I diabetes concerning changing insulin needs during pregnancy the nurse recognizes that further teaching is warranted when the woman states A. I will need to increase my insulin dosage during the first 3 months of pregnancy. B. Insulin dosage will likely need to be increased during the second and third trimesters C. Episodes of hypoglycemia are more likely to occur during the first 3 months D. Insulin needs should return to normal within 7 to 10 days after birth if I am bottle feeding

A. I will need to increase my insulin dosage during the first 3 months of pregnancy. Rationale: Insulin needs are reduced in the first trimester due to increased insulin production by the pancreas and increased peripheral sensitivity to insulin. This statement is accurate and signifies understanding. Insulin resistance begins as early as 14 to 16 weeks of gestation and continues to rise until it stabilizes during the last few weeks of pregnancy. This statement is correct. During the first trimester maternal blood glucose levels are reduced and the insulin response to glucose is enhanced therefore this is when an episode of hypoglycemia is most likely to occur. For the non breastfeeding mother insulin levels return to normal within 7 to 10 days. Lactation utilized maternal glucose, therefore the mothers insulin requirements will remain low during lactation. On completion of weaning mothers prepregnancy insulin requirement is reestablished

A patient informs the nursery nurse that she has taken cocaine throughout her pregnancy. The nurse needs to be alert to signs of which problem in the woman's newborn? A. IUGR B. Respiratory distress C. Infections D. Low blood pressure

A. IUGR Rationale: Cocaine causes vasoconstriction of placental vessels, which can inhibit the growth of the fetus. Newborns exposed to cocaine in utero are not at risk for respiratory distress, infections, or low blood pressure.

The nurse is teaching a woman how to administer insulin sq. Which precautions should the nurse emphasize during the teaching session? A. Insulin is injected slowly (over 2 to 4 seconds). B. To prevent hypoglycemia, a meal should be taken 60 minutes after regular insulin is injected. C. Insulin should be injected with the needle inserted at a 45-degree angle. D. Aspirating when injecting into subcutaneous tissue is necessary.

A. Insulin is injected slowly (over 2 to 4 seconds) Rationale: Insulin is injected slowly (over 2 to 4 seconds) to allow tissue expansion and minimize pressure, which can cause insulin leakage. To prevent hypoglycemia, a meal should be taken 30 minutes after regular insulin is injected not 60 minutes. Unless the woman is very thin, insulin should be injected with the short needle inserted at a 90-degree angle so that the tip of the needle reaches the fatty tissue layer. Aspirating when injecting into subcutaneous tissue is not necessary.

With regard to anemia, nurses should be aware that: A. It is the most common medical disorder of pregnancy B. It can trigger reflex bradycardia C. The most common form of anemia is caused by folate deficiency D. Thalassemia is a European version of sickle cell anemia

A. It is the most common medical disorder of pregnancy Rationale: Iron deficiency anemia causes 75% of anemias in pregnancy. It is difficult to meet the pregnancy needs for iron through diet alone. Reflex bradycardia is a slowing of the heart in response to the blood flow increases immediately after birth. The most common form of anemia is iron deficiency anemia. Both thalassemia and sickle cell hemoglobinopathy are hereditary but not directly related or confined to geographic areas

A patient notices that the doctor writes positive Chadwicks sign on her chart. She asks the nurse what this means. The nurse's best response is A. It refers to thee bluish color of the cervix in pregnancy B. It means the cervix is softening. C. The doctor was able to flex the uterus against the cervix D. That refers to a positive sign of pregnancy

A. It refers to thee bluish color of the cervix in pregnancy Rationale: Increased vascularity of the pelvic organs during pregnancy results in the bluish color of thee cervix, vagina, and labia, called Chadwicks sign. Softening of the cervix is Goodells sign. The softening of thee lower segment of the uterus. (Hegers sign) can allow the uterus to be flexed against the cervix. Chadwicks sign is a probable indication of pregnancy

The recommended diet for pregnancy differs from the recommended diet for lactation, because A. Lactating women require more calorie and protein B. Pregnant women need more calcium C. Lactating women require fewer vitamins D. Pregnant women require more iron and protein

A. Lactating women require more calorie and protein Rationale: The lactating woman needs 200 calories and 5g of protein more than the pregnant woman. Calcium needs are the same for pregnancy and lactation. Vitamin needs are higher during pregnancy and lactation. Protein requirements are higher during lactation

To increase her folic acid intake, what snack would be most beneficial for a pregnant woman? A. Orange juice and fortified cereal B. Ham sandwich with mustard C. Strawberries with bananas D. Tuna fish salad and tomato

A. Orange juice and fortified cereal Rationale: Both orange juice and fortified cereal are excellent sources of folic acid. Most meats and condiments are not high in folic acid. The bread may have folic acid added if it is enriched. Strawberries, bananas, most meats and tomatoes are not high in folic acid.

Screening at 24 weeks of gestation reveals that a pregnant woman has gestational diabetes mellitus (GDM). In planning her care, the nurse and the woman mutually agree that an expected outcome is to prevent injury to the fetus as a result of GDM. The nurse identifies that the fetus is at greatest risk for: A. Macrosomia. B. Congenital anomalies of the central nervous system. C. Preterm birth. D. Low birth weight.

A. Macrosomia. Rationale: Poor glycemic control later in pregnancy increases the rate of fetal macrosomia. Poor glycemic control during the preconception time frame and into the early weeks of pregnancy is associated with congenital anomalies. Preterm labor or birth is more likely to occur with severe diabetes and is the greatest risk in woman with pregestational diabetes. Increased weight, or macrosomia, is the greatest risk factor for this woman.

People who have two copies of the same abnormal autosomal dominant gene will usually be A. More severely affected by the disorder than will people with one copy of the gene B. Infertile and unable to transmit the gene C. Carriers of the trait but not affected with the disorder D. Mildly affected with the disorder

A. More severely affected by the disorder than will people with one copy of the gene Rationale: People who have two copies of an abnormal gene are usually more severely affected by the disorder because they have no normal gene to maintain normal function. Infertility may or may not be caused by chromosomal defects. A carrier of the trait has one recessive gene. Those mildly affected with the disorder will have only one copy o the abnormal gene.

Which principle of teaching should the nurse use to ensure learning in a family situation? A. Motivate the family with praise and positive reinforcement B. Present complex subject material first, while the family is alert and ready to learn C. Families should be taught by using medical jargon so they will be able to understand the technical language used by physicians D. Learning is best accomplished using the lecture format

A. Motivate the family with praise and positive reinforcement Rationale: Praise and positive reinforcement are particularly important when a family is trying to master a frustrating task, such as breastfeeding. Learning is enhanced when the teaching is structured to present the simple tasks before the complex material. Even though a family may understand English fairly well, they may not understand the medical terminology or slang terms. A lively discussion stimulates more learning than a straight lecture, which tends to inhibit questions.

Which analysis of maternal serum may predict chromosomal abnormalities in the fetus? A. Multiple-marker screening B. Lecithin/sphingomyelin (L/S) ratio C. Biophysical profile D. Type and crossmatch of maternal and fetal serum

A. Multiple-marker screening Rationale: Maternal serum can be analyzed for abnormal levels of alpha-fetoprotein, human chorionic gonadotropin, and estriol. The multiple marker screening may predict chromosomal defects in the fetus. The L/S ratio is used to determine fetal lung maturity. A biophysical profile is used for evaluating fetal status during the antepartum period. Five variables are used, but none is concerned with chromosomal problems. The blood type and crossmatch will not predict chromosomal defects in the fetus

A woman who delivered her baby 6 hours ago complains of headache and dizziness. The nurse administers an analgesic but does not perform any assessments. The woman then has a grand mal seizure, falls out of bed, and fractures her femur. How would the actions of the nurse be interpreted in relation to standards of care? A. Negligent because the nurse failed to assess the woman for possible complications B. Negligent because the nurse medicated the woman C. Not negligent because the woman had signed a waiver concerning the use of side rails D. Not negligent because the woman did not inform the nurse of her symptoms as soon as they occurred

A. Negligent because the nurse failed to assess the woman for possible complications Rationale: By not assessing the woman, the nurse failed to meet the established standards of care. The first element of negligence relates to whether the nurse has a duty to provide care to the woman. The care that the nurse provides must meet the established standards of care. By not first assessing the woman, the nurse does not meet the established standards of care. The nurse could be found negligent. The nurse is responsible for assessing the woman

When the nurse is alone with a battered patient, the patient seems extremely anxious and says, It was all my fault. The house was so messy when he got home and I know he hates that. The best response by the nurse is A. No one deserves to be hurt. Its not your fault. How can I help you? B. What else to you do that makes him angry enough to hurt you C. He will never find out what we talk about. Don't worry. We're here to help you D. You have to remember that he is frustrated and angry so he takes it out on you

A. No one deserves to be hurt. Its not your fault. How can I help you? Rationale: The nurse should stress that the patient is not at fault.(A) This is placing the blame on the woman(B) This is false reassurance. In order to assist the woman, many times the batterer will find out about the conversation.(C). This is placing the blame on the woman and finding excuses for the batterer

A gravida patient at 32 weeks of gestation reports that she has severe lower back pain. The nurses assessment should include A. Observation of posture and body mechanics B. Palpation of the lumbar spine C. Exercise pattern and duration D. Ability to sleep for at least 6 hours uninterrupted

A. Observation of posture and body mechanics Rationale: Correct posture and body mechanics can reduce lower back pain caused by increasing lordosis. Pregnancy should not cause alterations in the spine. Any assessment for malformation should be done early in the pregnancy. Certain exercises can help relieve back pain. Rest is important for well being, but the main concern with back pain is to assess posture and body mechanics.

A new mother asks the nurse about the white substance covering her infant. The nurse explains that the purpose of vernix caseosa is to A. Protect the fetal skin from amniotic fluid B. Promote normal peripheral nervous system development C. Allow transport of oxygen and nutrients across the amnion D. Regulate fetal temperature

A. Protect the fetal skin from amniotic fluid Rationale: Prolonged exposure to amniotic fluid during the fetal period could result in breakdown of the skin without the protection of the vernix caseosa. Normal peripheral nervous system development is dependent on nutritional intake of the mother. The amnion is the inner membrane that surrounds the fetus. It is not involved in the oxygen and nutrient. exchange. The amniotic fluid aids in maintaining fetal temperature

When caring for a woman with mild preeclampsia, it is critical that during assessment the nurse is alert for signs of progress to severe preelcampsia. Progress to severe preeclampsia would be indicated by which of the following assessment findings? A. Proteinuria of 3+ or greater B. Platelet of 200,000/mm^3 C. Deep tendon reflexes 2+, ankle clonus is absent D. BP of 154/94 and 156/100, 6 hours apart

A. Proteinuria of 3+ or greater Rationale: with severe preeclampsia, the DTRs would be more than 3+ with possible ankle clonus; the BP would be more than 160/110; thrombocytopenia with a platelet level less than 150,000mm^3

On which aspect of fetal diagnostic testing do parents usually place the most importance? A. Safety of the fetus B. Duration of the test C. Cost of the procedure D. Physical discomfort caused by the procedure

A. Safety of the fetus Rationale: Although all of these are considerations, parents are usually most concerned about the safety of the fetus.

A 4 year old is hospitalized for treatment of Pneumonia. The nurse informs the child's mother that the pediatric unit is a Family Centered Child Care Unit. What does this mean for her? A. She will be allowed input into her child's care B. She will not be able to stay at night with her child but must stay during the day C. She will not be allowed to visit her child, because it is considered to cause emotional distress D. She will be responsible for her child's total care

A. She will be allowed input into her child's care Rationale: Family Centered Child Care recognizes and respects the pivotal role of the family in the child's life. It supports families and views parents and professionals as equal partners. Family Centered Child Care units encourage parents to stay with the child around the clock. The child's mother will share care of the child as the parents and professionals are viewed as equals

A pregnant woman's mother is worried that her daughter is not big enough at 20 weeks. The nurse palpates and measures the fundal height at 20cm, which is even with the woman's umbilicus. What should the nurse report to the woman and her mother? A. The body of the uterus is at thee belly button level, just where it should be at this time B. You're right. We'll inform the practitioner immediately C. When you come for next months appointment, we'll check you again to make sure that the baby is growing D. Lightening has occurred, so the fundal height is lower than expected

A. The body of the uterus is at thee belly button level, just where it should be at this time Rationale: At 20 weeks, the fundus is usually located at the umbilical level. Because the uterus grows in a predictable pattern, obstetric nurses should know that the uterus of 20 weeks of gestation is located at the level of the umbilicus. This is incorrect information. At 20 weeks, the uterus should be at the umbilical level (B). By avoiding the direction question, this might increase the anxiety of both the mother and grandmother. The descent of the fetal head (lightening) occurs in late pregnancy

The various systems and organs develop at different stages. Which statement is accurate? A. The cardiovascular system is the first organ system to function in the developing human. B. Hematopoiesis originating in the yolk sac begins in the liver at 10 weeks. C. The body changes from straight to C-shaped at 8 weeks. D. The gastrointestinal system is mature at 32 weeks.

A. The cardiovascular system is the first organ system to function in the developing human. Rationale: The heart is developmentally complete by the end of the embryonic stage. Hematopoiesis begins in the liver during thee 6th week. The body becomes C-shaped at 21 weeks. The gastrointestinal system is complete at 36 weeks

Both members of an expectant couple are carriers for phenylketonuria (PKU), an autosomal recessive disorder. In counseling them about the risk to their unborn child, the nurse should tell them that A. The child has a 25% chance of being affected B. The child will be a carrier, like the parents C. The child has a 50% chance of being affected D. One of four of their children will be affected

A. The child has a 25% chance of being affected Rationale: Each child born to the couple who carries an autosomal recessive trait has a 25% chance of having the disorder, because the child receives either a normal or abnormal gene from each parent. If one member of the couple has the autosomal recessive disorder, all of their children will be carriers. If both parents are carriers, each child has a 50% chance of being a carrier. The child has a 25% chance of being affected and a 50% chance of being a carrier. Each child has the identical odds of being affected. Because both parents are carriers, each child has a 25% chance of being affected, a 50% chance of being a carrier, and a 25% chance of not having the gene transmitted

The nurse is assessing a 3-year-old African-American child in the clinic for the first time. The child's height and weight are at the 25th percentile on the commonly used growth chart from the National Center for Health Statistics. How should the nurse interpret these data? A. The child's growth is within normal limits. B. The child's growth is not within normal limits. C. The chart is not accurate for African-American children. D. The chart is not useful until several measurements are plotted over time.

A. The child's growth is within normal limits. Rationale: The 25th percentile is within the normal range. Children from different ethnic and racial groups are included in the statistics, making the growth chart representative. The chart is useful both for screening and for assessment over time.

Which part of the mature sperm contains the male chromosomes? A. The head of the sperm B. The middle portion of the sperm C. X-bearing sperm D. the tail of the sperm

A. The head of the sperm Rationale: The head of the sperm contains the male chromosomes that will join the chromosomes of the ovum. The middle portion of the sperm supplies energy for the tails whip like action. If an X-bearing sperm fertilizes the ovum, the baby will be female. The tail of the sperm helps propel the sperm toward the ovum

What is the most likely to be a concern for the older mother? A. The importance of having enough rest and sleep B. Information about effective contraceptive methods C. Nutrition and diet planning D. Information about exercise and fitness

A. The importance of having enough rest and sleep Rationale: The woman who delays childbearing may have unique concerns, one of which is having less energy than younger mothers. The older mother usually has more financial means to search out effective contraceptive methods. The older mother is usually better off financially and can afford better nutrition. Information about exercise and fitness is readily available

Spontaneous termination of a pregnancy is considered to be an abortion if: A. The pregnancy is less than 20 weeks. B. The fetus weighs less than 1000 g. C. The products of conception are passed intact. D. No evidence exists of intrauterine infection.

A. The pregnancy is less than 20 weeks. Rationale: An abortion is the termination of pregnancy before the age of viability (20 weeks). The weight of the fetus is not considered because some fetuses of an older age may have a low birth weight. A spontaneous abortion may be complete or incomplete. A spontaneous abortion may be caused by many problems, one being intrauterine infection

Home nursing care has experienced dramatic growth since 1990. The nurse who works in this setting must function independently within established protocols. Which statement related to nursing care of the child at home is most correct? A. The technology dependent infant cab safely be cared for at home B. Home care increases readmissions to the hospital for a child with chronic conditions C. There is increased stress for the family when a sick child is being cared for at home D. The family of the child with a chronic condition is likely to be separated from their support system if the child is cared for at home

A. The technology dependent infant cab safely be cared for at home Rationale: Greater numbers of technology dependent infants and children are now cared for at home. The numbers include those needing ventilator assistance, total parenteral nutrition, IV medications, apnea monitoring, and other device assisted nursing care. Optimal home care can reduce the rate of readmisson to the hospital for children with chronic conditions. Consumers often prefer home care because of the decreased stress on the family when the patient is able to remain at home. When the child is cared for at home the family is less likely to be separated from their support system because of the need for hospitalization

A pregnant woman's biophysical profile score is 8. She asks the nurse to explain the results. The nurse's best response is A. The test results are within normal limits B. Immediate delivery by cesarean birth is being considered C. Further testing will be performed to determine the meaning of this score D. An obstetric specialist will evaluate the results of this profile and, within the next week, will inform you of your options regarding delivery

A. The test results are within normal limits Rationale: The normal biophysical score ranges from 8 to 10 points if the amniotic fluid volume is adequate. A normal score allows conservative treatment of high risk patients. Delivery can be delayed if fetal well being is indicated. Scores less than 4 should be investigated, and delivery could be initiated sooner than planned. This score is within normal range, and no further testing is required at this time. The results of the biophysical profile are usually available immediately after the procedure is performed

One of the assessments performed in the delivery room is checking the umbilical cord for blood vessels. Which finding is considered to be within normal limits? A. Two arteries and one vein B. Two arteries and two veins C. Two veins and one artery D. One artery and one vein

A. Two arteries and one vein Rationale: The umbilical cord contains two arteries and one vein to transport blood between the fetus and the placenta. The option is abnormal and may indicate other anomalies. Any option other than two arteries and one vein is considered abnormal and requires further assessment. The presence of one umbilical artery is considered an abnormal finding. This infant would require further assessment for other anomalies

One of the assessments performed in the delivery room is checking the umbilical cord for blood vessels. Which finding is considered to be within normal limits? A. Two arteries and one vein B. Two arteries and two veins C. Two veins and one artery D. One artery and one vein

A. Two arteries and one vein Rationale: The umbilical cord contains two arteries and one vein to transport blood between the fetus and the placenta. Two arteries and two veins are abnormal and may indicate other anomalies. Two veins and one artery are abnormal and may indicate other anomalies. One artery and one vein are abnormal and may indicate other anomalies.

Prophylaxis of subacute bacterial endocarditis is given before and after birth when a pregnant woman has: A. Valvular disease. B. Congestive heart disease. C. Arrhythmias. D. Postmyocardial infarction.

A. Valvular disease. Rationale: Prophylaxis for intrapartum endocarditis and pulmonary infection may be provided for women who have mitral valve prolapse. Prophylaxis for intrapartum endocarditis is not indicated for a patient with congestive heart failure. Prophylaxis for intrapartum endocarditis is not necessary for a woman with underlying arrhythmias. A woman who is postmyocardial infarction does not require prophylaxis for intrapartum endocarditis.

A common effect of both smoking and cocaine use in the pregnant woman is: A. Vasoconstriction B. Increased appetite C. Changes in insulin metabolism D. Increased metabolism

A. Vasoconstriction Rationale: Both smoking and cocaine use cause vasoconstriction, which results in impaired placental blood flow to the fetus. Both smoking and cocaine use decrease the appetite. Smoking and cocaine do not change insulin metabolism. Smoking can increase metabolism

Which response by the nurse to the womans statement, I'm afraid to have a cesarean birth, would be most therapeutic? A. What concerns you the most about cesarean birth? B. Everything will be okay C. Don't worry about it. It will be over soon D. The doctor will be in later, and you can talk to him

A. What concerns you the most about cesarean birth? Rationale: Focusing on what the woman is saying and asking for clarification is the most therapeutic response. This response belittles the womans feelings (B). This response will indicate that the womans feelings are not important (C). This response does not allow the woman to verbalize here feelings when she desires

A woman tells the nurse at a prenatal interview that she has quit smoking, only has a glass of wine with dinner, and has cut down on coffee to four cups a day. What response by the nurse will be most helpful in promoting a lifestyle change? A. You have made some good progress toward having a healthy baby. Lets talk about the changes you have made. B. You need to do a lot better than that. You are still hurting your baby C. Here are some pamphlets for you to study. They will help you find more ways to improve. D. Those few things won't cause any trouble. Good for you

A. You have made some good progress toward having a healthy baby. Lets talk about the changes you have made. Rationale: Praising her for making positive changes in an effective technique for motivating a patient. She still has to identify the risk factors to optimize the results (A). This statement is belittling to the patient. She will be less likely to confide in the nurse (B). The nurse is not acknowledging the efforts that the woman has already accomplished. Those accomplishments need to be praised to motivate the woman to continue (C). Alcohol and coffee consumption are still major risk factors and need to be addressed in a positive, nonjudgemental manner (D)

The average man is taller than the average woman at maturity because of A. a longer period of skeletal growth. B. earlier development of secondary sexual characteristics. C. earlier onset of the growth spurt. D. starting puberty at an earlier age.

A. a longer period of skeletal growth. Rationale: The man's greater height at maturity is the combined result of beginning the growth spurt at a later age and continuing it for a longer period of time. Males start puberty 6 months to 1 year later than females.

A primigravada at 10 weeks of gestation reports slight vaginal spotting without passage of tissue and mild uterine cramping. When examined, no cervical dilation is noted. The nurse caring for this woman would: A. anticipate that the woman will be sent home and placed on bed rest with instructions to avoid stress and orgasm B. prepare the woman for dilation and curettage C. inform the woman that frequent blood tests will be required to check the level of estrogen D. tell the woman that the doctor will most likely perform a cerclage to help her maintain her pregnancy

A. anticipate that the woman will be sent home and placed on bed rest with instructions to avoid stress and orgasm Rationale: the woman is experiencing a threatened abortion; therefore, a conservative approach is attempted first; b reflects management of an inevitable and complete or incomplete abortion; blood tests for HCG and progesterone levels would be done; cerclage or suturing of the cervix is done for recurrent, spontaneous abortion associated with premature dilation of the cervix

When teaching a group of mothers of preteen girls, the nurse explains that the earliest outward sign of puberty starting is A. breast changes. B. body contours. C. body hair. D. skeletal growth.

A. breast changes Rationale: Breast changes, such as nipple enlargement and protrusion, are the earliest outward changes of puberty. Body contours and hair develop as puberty progresses. The growth spurt begins about 1 year after initial breast development.

To administer an IM injection safely to a 6-year-old, the nurse must be aware of the child's developmental stage. This knowledge will assist the nurse in gaining the child's cooperation prior to the treatment. During this process, the nurse is functioning in the role of A. care provider. B. teacher. C. advocate. D. manager.

A. care provider Rationale: The care provider provides direct nursing care. The role includes understanding developmental stages and appropriately altering care to meet the patient's needs. In the role of teacher, patient education is the priority. In the role of advocate, the nurse will speak on behalf of the patient to express the patient's wishes and needs. In the role of manager, the nurse coordinates care of the patient.

A woman who is 8 months' pregnant had a biophysical profile test done. The results give a score of 4/10. The nurse can anticipate that the next plan of action may be to A. consider delivery. B. send the woman home and retest in 1 week. C. retest the woman in 3 hours. D. admit the woman for bed rest and to monitor maternal vital signs.

A. consider delivery. Rationale: A score of 4 out of 10 (4/10) is nonreassuring. Delivery may be considered as an option, because the fetus is at risk. Three hours will not alter the test results significantly. Interventions may be necessary with a score of 4/10. A biophysical profile indicates the condition of the fetus, not the woman.

When a nurse faces a difficult problem, the thinking process should be controlled and directed toward finding solutions or opinions. This form of thinking is termed A. critical thinking. B. nursing process. C. undirected thinking. D. emotional thinking.

A. critical thinking. Rationale: Critical thinking is controlled and directed. It includes recognizing assumptions, examining biases, analyzing the need for closure, collecting and analyzing data, and evaluating emotions and environmental factors. Nursing process is a way the nurse can assess the patient, plan care, and determine the success of the care. Undirected thinking is not focused on finding solutions. Emotional thinking deals with the emotional side of an issue and does not focus on the facts.

A woman who is 8 months pregnant has been advised to have an amniocentesis. She asked the nurse the reason for the procedure. The usual reason for an amniocentesis during this period of pregnancy would be to A. determine fetal lung maturity. B. identify chromosomal abnormalities. C. measure amniotic fluid amounts. D. detect levels of alpha-fetoprotein.

A. determine fetal lung maturity. Rationale: The usual reason for amniocentesis during the third trimester is to determine fetal lung maturity. Amniocentesis to identify chromosomal abnormalities or to detect levels of AFP are done during the second trimester. Amniocentesis is not done to measure amniotic fluid amounts.

The use of benzodiazepines can potentiate the action of analgesics and reduce nausea. When preparing to administer a benzodiazepine to a laboring woman, the nurse could expect to give which one of the following medications A. diazepam (Valium) B. promethazine (Phenergan) C. butorphanol tartrate (Stadol) D. fentanyl (Sublimaze)

A. diazepam (Valium) Rationale: promethazine is a phenothiazine, butorphanol is an opioid agonist-antagonist analgesic; fentanyl is an opioid agonist analgesic

An example of atraumatic care would be to: A. eliminate or minimize distress experienced by a child in a health care setting. B. restrict visiting hours to adults only. C. perform invasive procedures only in treatment room. D. permit only traditional clinical practices.

A. eliminate or minimize distress experienced by a child in a health care setting.

The nurse notes that the hemoglobin level of a woman at 35 weeks of gestation is 11.5 g/dL. The nurse's next action should be to A. note that this is within the normal range for pregnancy. B. note that this is within the normal range for an average adult. C. call the physician; this shows mild anemia. D. recall that the RBC count increases slightly during pregnancy.

A. note that this is within the normal range for pregnancy. Rationale: The normal range of hemoglobin for pregnancy is greater than 11 g/dL in the first and third trimesters and greater than 10.5 g/dL in the second trimester. The normal range of hemoglobin for an average female is 12 to 16 g/dL. With pregnancy the levels are lower due to the increased iron requirements of the fetus. The RBC count decreases slightly because of hemodilution.

A woman has tested human immunodeficiency virus (HIV) positive and has now discovered that she is pregnant. Which statement indicates that she understands the risks of this diagnosis? A. even though my test is positive, my baby might not be affected. B. I know I will need to have an abortion as soon. as possible C. This pregnancy will probably decrease the chance that I will develop AIDS D. My baby is certain to have AIDS and die within the first year of life

A. even though my test is positive, my baby might not be affected. Rationale: The fetus is likely to test positive for HIV in the first 6 months until the inherited immunity from the mother wears off. Many of these babies will convert to HIV negative status. With the newer drugs, the risk for infection of the fetus has decreased. The pregnancy will increase thee chance of converting. With the newer drugs, the risk for infection of the fetus has decreased. Also, the life span of an infected newborn has increased

A woman is giving birth to her third child in a setting that allows her husband and children to be actively involved in the process. The nurse caring for her must also consider the husband and the two children as patients and work to meet their needs. This type of setting is termed A. family centered care B. emergency care C. hospice care D. individual care

A. family centered care Rationale: Family centered care is any setting where the pregnant woman and her family are treated as one unit. The nurse assumes a major role in teaching, counseling, and supporting the family. In emergency care settings, the nurse deals primarily with the patient who is having difficulty. In hospice care settings, the nurse deals with the patients who have terminal illnesses. Individual care deals only with the patient and does not include the family

When comparing therapeutic communication with social communication, therapeutic communication is A. goal-directed and focused. B. mutually satisfying and rewarding. C. a mutual give and take between the nurse and patient. D. predictable.

A. goal-directed and focused Rationale: Therapeutic communication is patient-oriented and goal-directed. The focus is the patient. Social communication provides mutual satisfaction, whereas therapeutic communication focuses on the patient's needs only. Social communication is a mutual give and take, whereas therapeutic communication focuses on the patient's needs only and is not predictable.

During her first prenatal visit to the clinic, a woman gives the following obstetric history: a boy born 9 years ago at full term, twin girls born 5 years ago at 36 weeks, a miscarriage at 9 weeks 2 years ago. The nurse correctly records her obstetric history as A. gravida 4, para 2, aborta 1. B. gravida 3, para 3, aborta 1. C. gravida 4, para 3, aborta 1. D.gravida 3, para 2, aborta 1.

A. gravida 4, para 2, aborta 1. Rationale: The woman is currently pregnant and has been pregnant 3 more times; that makes her a gravida 4. She has delivered two pregnancies after 20 weeks of gestation; that makes her a para 2. The twin girls count as one pregnancy. She delivered one pregnancy prior to 20 weeks; that makes her an aborta 1.

A 42-year-old is at the clinic for her first prenatal visit. The nurse is doing the initial assessment and is aware that the woman is at risk for A. having a spontaneous abortion prior to 12 weeks. B. having a sexually transmitted disease. C. not obtaining adequate prenatal care. D. developing abnormalities of the reproductive organs.

A. having a spontaneous abortion prior to 12 weeks. Rationale: Women older than 40 years have a 26% risk of spontaneous abortion. Older women have a lower rate of STDs and are more likely to obtain adequate prenatal care compared to adolescents. Older women are not at higher risk for developing abnormalities of the reproductive organs.

During teaching to an antepartum patient, it is important for the nurse to give information about ways to avoid some birth defects. One area of concern that can be taught at this time is A. lifestyle changes. B.the need for genetic counseling. C. the need for adequate exercise. D. the importance of keeping all of her prenatal appointments.

A. lifestyle changes. Rationale: Lifestyle changes such as stopping alcohol consumption, stopping smoking, and avoiding chemicals and medications that are teratogenic can prevent some birth defects. Genetic counseling should be done prior to pregnancy. Adequate exercise and prenatal check-ups are important for a healthy outcome of a pregnancy but are not associated with birth defects.

The nurse is admitting a woman to the labor unit. When reviewing the prenatal record, the nurse notices that the woman did not gain the adequate amount of weight that was recommended for her pregnancy. The nurse is aware that the neonate will be at risk for A. low birth weight. B. high birth weight. C. respiratory problems. D. cardiovascular problems.

A. low birth weight. Rationale: Low birth weights for the neonate are associated with insufficient weight gain during pregnancy. Respiratory and cardiovascular problems are not associated with insufficient weight gain during pregnancy.

The nurse providing care for the pregnant woman understands that a factor indicating the need for fetal diagnostic procedures is A. maternal diabetes B. maternal age older than 30 years C. previous infant more than 3000g at birth D. weight gain of 25 pounds

A. maternal diabetes Rationale: Diabetes is a risk factor in pregnancy because of possible impairment of placental perfusion. A maternal age greater than 35 years is an indication for testing. Having had another infant weighing greater than 4000g at birth is an indication for testing. Excessive weight gain is an indication for testing. Normal weight gain is 15 to 35 pounds

People who have two copies of the same abnormal autosomal dominant gene will usually be A. more severely affected by the disorder than people with one copy of the gene. B. infertile and unable to transmit the gene. C. carriers of the trait but not affected with the disorder. D. mildly affected with the disorder.

A. more severely affected by the disorder than people with one copy of the gene. Rationale: People who have two copies of an abnormal gene are usually more severely affected by the disorder because they have no normal gene to maintain normal function. Having two copies of the same abnormal autosomal dominant gene does not always lead to infertility. Therefore, the individual may be able to transmit the gene. Because the gene is dominant, the individual will manifest the disorder and not just be a carrier.

Congenital anomalies can occur with the use of antiepileptic drugs, including (select all that apply) A. Cleft lip B. Congenital heart disease C. Neural tube defects D. Gastroschisis E. Diaphragmatic hernia

A.B.C. Cleft lip; Congenital heart disease; Neural tube defects Rationale: Congenital anomalies that can occur with AEDs include cleft lip or palate, congenital heart disease, urogenital defects, limb reduction, mental retardation, and neural tube defects. This is referred to assess fetal hydantoin syndrome.

A pregnant woman is scheduled for a transvaginal ultrasound test to establish gestational age. In preparing this woman for the test, the nurse would: A. place the woman in a supine position with her hips elevated on a folded pillow B. instruct her to come for the test with a full bladder C. administer an analgesic 30 minutes before the test D. lubricate the vaginal probe with transmission gel

A. place the woman in a supine position with her hips elevated on a folded pillow Rationale: a supine position with hips elevated enhances the view of the uterus; a lithotomy position may also be used; a full bladder is not required for the vaginal ultrasound but would be needed for most abdominal ultrasounds; during the test the woman may experience some pressure but medication for pain relief before the test is not required; contact gel is used with the abdominal ultrasound; water-soluble lubricant may be used to ease insertion of the vaginal probe

A 16-year-old is being seen for the first time by the nurse practitioner. The young woman states that she has not had the onset of menstruation yet. Her breasts are developing and her pelvis has widened. The term used to describe this list of signs and symptoms is A. primary amenorrhea. B. secondary amenorrhea.

A. primary amenorrhea Rationale: Primary amenorrhea is when a girl has not started menstruation by the time she is 16 years old. Secondary amenorrhea is the absence of menstruation for at least three cycles after regular cycles have been established.

The nursery nurse is called in to the delivery room of a 22-year-old primigravida. The delivery nurse informs the nursery nurse that the patient had oligohydramnios throughout the pregnancy. In planning care for the newborn, the nursery nurse is aware that the baby may develop A. Respiratory problems B. there are no potential problems associated with oligohydramnios. C. low blood sugar. D. jaundice.

A. respiratory problems. Rationale: Prolonged oligohydramnios interferes with fetal lung development. Therefore, the nurse needs to assess for respiratory problems. Other potential problems include fibrous amniotic bands that can result in fetal deformations or intrauterine limb amputation. Low blood sugar and jaundice are not associated with oligohydramnios.

The nurse understands that prenatal growth and development proceed in a cephalocaudal pattern, meaning that A. the brain will develop first. B. the brain will develop last. C. the liver and spleen will develop first. D. the heart develops last.

A. the brain will develop first. Rationale: Cephalocaudal development means it occurs in a head-to-toe manner. Therefore, the brain will develop first, not last. Cephalocaudal development means it occurs in a head-to-toe manner. Therefore, the liver and spleen would not be the first organs to develop. Cephalocaudal development means it occurs in a head-to-toe manner. Therefore, the heart will be one of the first organs to develop.

When comparing the endometrial cycle with the ovarian cycle on day 22, A. the progesterone level is at its peak, but the LH level is low. B. the progesterone level is low, but the FSH level is at its peak. C. the estrogen level is low, but the LH level is at its peak. D. both the estrogen level and the LH level are at the peak.

A. the progesterone level is at its peak, but the LH level is low. Rationale: In the endometrial cycle on day 22, the progesterone level has reached its peak and will start decreasing in 1 to 2 days. In the ovarian cycle at the same time, the LH levels have already dropped and will remain low until about day 10 on the next cycle. On day 22, the progesterone level should about be at its peak, and the FSH level should be low. On day 22, the estrogen level is high, and the LH level is low.

When assessing women, it is important for the nurse to keep in mind the possibility that they are victims of violence. The nurse should A. use an abuse assessment screen during the assessment of every woman B. recognize that abuse rarely occurs during pregnancy C. Assess a woman's legs and back as the most commonly injured areas D. notify the police immediately if abuse is suspected

A. use an abuse assessment screen during the assessment of every woman Rationale: all women should be screened because abuse can happen to any woman; abuse often escalates during pregnancy; thee most commonly injured sites are head, neck, chest, abdomen, breasts, and upper extremities. If abuse is suspected the nurse needs to assess further to encourage the disclosure and then assist the woman to take action and formulate a plan

During the first 2 weeks after conception, the fertilized ovum is called a A. zygote. B. embryo. C. fetus. D. neonate.

A. zygote. Rationale: During the preembryonic stage, the cell formed by the union of an ovum and sperm is called the zygote. This stage lasts 2 weeks. The embryonic stage is from week 3 to week 8 after conception. The fetal stage lasts from week 9 until birth. After the fetus is born, it is called a neonate.

When taking an initial prenatal history on a woman, she admitted to cocaine use during the early days of the pregnancy. The nurse is aware that this would put her at risk for( Select all that apply) A. placenta previa. B. abruptio placentae. C. large for gestational age baby. D. ectopic pregnancy.

A.B placenta previa; abruptio placentae. Rationale: Cocaine use is associated with placenta previa and abruptio placentae. Cocaine use is not associated with large for gestational age babies or ectopic pregnancy.

A woman is expecting her second child. She expressed concern to the nurse about how her 4-year-old will adapt to the new baby. The following are some suggestions the nurse should include in her teaching. (Select all that apply) A. Come in and listen to the baby's heartbeat. B. Spend more time with grandmother to prepare him for being away from mother during the birth. C. Take a sibling class offered by the hospital. D. Decide which of your toys you would like to give to the new baby.

A.B.C. Come in and listen to the baby's heartbeat; Spend more time with grandmother to prepare him for being away from mother during the birth; Take a sibling class offered by the hospital. Rationale: A 4-year-old is curious about the changes in mother's body and the baby. By being included in the process, the child will not feel left out. It will also give the child an opportunity to ask questions. Children need to prepare for being away from mother during the birth and hospitalization. Starting early in the pregnancy to spend more time with the individual who will care for them will assist in the transition. Sibling classes provide an opportunity for children to discuss what newborns are like and what changes the new baby will bring to the family. Children need to be reassured that they are still maintaining an important role in the family. When they are asked to give up their possessions for the new baby, they may feel resentment.

A young female patient comes to the health unit at school to discuss her irregular periods. In providing education regarding the female reproductive cycle, the nurse describes the regular and recurrent changes related to the ovaries and uterine endometrium. Although this is generally referred to as the menstrual cycle, the ovarian cycle includes which phases? (Select all that apply) A. Follicular B. Ovulatory C. Luteal D. Proliferative E. Secretory

A.B.C. Follicular; Ovulatory; Luteal Rationale: The follicular phase is the period during which the ovum matures. It begins on day 1 and ends around day 14. The ovulatory phase occurs near thee middle of the cycle, approximately 2 days before ovulation. After ovulation and under the influence of the luteinizing hormone, the luteal phase corresponds with the last 12 days of the menstrual cycle. The proliferative and secretory phases are part of the endometrial cycle. The proliferative phase takes place during the first half of the ovarian cycle when the ovum matures. The secretory phase occurs during the second half of the cycle when the uterus is prepared to accept the fertilized ovum. These are followed by the menstrual phase if fertilization does not occur

Infants born to mothers dependent on opioids can exhibit the following signs (Select all that apply.) A. Neonatal abstinence syndrome (NAS) B. Low birth weight C. Increased incidence of SIDS D. Congenital defects

A.B.C. Neonatal abstinence syndrome (NAS); Low birth weight; Increased incidence of SIDS Rationale: Neonatal abstinence syndrome includes physical signs that affect all body systems as a result of fetal exposure to opioids. Most signs involve the neurologic and gastrointestinal systems. Infants may also have low birth weight and be at increased incidence of SIDS. Congenital defects are not associated with exposure to opioids in utero.

A nurse is working with a pregestational diabetic woman to plan the diet she will follow during pregnancy. Which of the following nutritional guidelines should be used to ensure a euglycemic state and appropriate weight gain (select all that apply)? A. Substantial bedtime snack composed of complex carbohydrates with some protein and fat B. Average calories per day of 2200 during the first trimester and 2500 during the second and third trimester C. Caloric distribution among three meals and one or two snacks D. Minimum of 45% carbohydrate daily E. Protein intake of at least 30% of the total kilocalories in a day F. Fat intake of 30% to 40% of all the daily caloric intake

A.B.C. Substantial bedtime snack composed of complex carbohydrates with some protein and fat; Average calories per day of 2200 during the first trimester and 2500 during the second and third trimester; Caloric distribution among three meals and one or two snacks Rationale: a minimum intake of 55% carbohydrates, 20% protein, and 25% fat is recommended daily

Because of the risk for toxoplasmosis infection, pregnant women are advised to (Select all that apply.) A. cook meat thoroughly to an internal temperature at least 160° F or as high as 180° F. B. avoid uncooked eggs and unpasteurized milk. C. avoid contact with materials that are possibly contaminated with cat feces when pregnant (cat litter boxes, sandboxes, garden soil). D. avoid contact with other people who have toxoplasmosis.

A.B.C. cook meat thoroughly to an internal temperature at least 160° F or as high as 180° F.; avoid uncooked eggs and unpasteurized milk.; avoid contact with materials that are possibly contaminated with cat feces when pregnant (cat litter boxes, sandboxes, garden soil Rationale: Toxoplasmosis is a protozoal infection caused by Toxoplasma gondii. Infection is transmitted through organisms in raw or undercooked meat, and through contact with infected cat feces, not from other people.

Maternal anemia is associated with the following except (Select all that apply.) A. preterm birth. B. low birth weight. C. pica. D. hemoglobin is lower than 13 g/dL in the first and third trimesters or lower than 12g/dL in the second trimester.

A.B.C. preterm birth; low birth weight.; pica. Rationale: Anemia is a condition in which a decline in circulating red blood cell mass reduces the capacity to carry oxygen to the vital organs of the mother or fetus. Significant maternal anemia is associated with preterm birth and low birth weight. Pica (consuming nonfood substances such as clay, dirt, ice, or starch) is also is a sign of iron deficiency anemia. A woman is usually considered anemic if her hemoglobin is lower than 11 g/dL in the first and third trimesters or lower than 10.5 g/dL in the second trimester.

Congenital disorders refer to those conditions that are present at birth. These disorders may be inherited and caused by environmental factors or maternal malnutrition. Toxic exposures have the greatest effect on development between 15 and 60 days of gestation. For the nurse to be able to conduct a complete assessment of the newly pregnant client, she should be knowledgeable regarding known human teratogens, which include (Select all that apply) A. Infections B. Radiation C. Maternal conditions D. Drugs E. Chemicals

A.B.C.D. Infections; Radiation; Maternal conditions; drugs Rationale: Exposure to radiation and a number of infections may result in profound congenital deformities. These include varicella, rubella, syphilis, parvovirus, CMV, toxoplasmosis. Certain maternal conditions such as diabetes and PKU may also affect organs and other parts of the embryo during this developmental period. Drugs such as antiseizure medication and some antibiotics, as well as chemicals including lead, mercury, tobacco, and alcohol, also may result in structural and functional abnormalities Coffee is not considered a teratogen

Today's nurse often assumes the role of teacher or educator. Patient teaching begins early in the childbirth process and continues throughout the postpartum period. Which strategies would be best to use for a nurse working with a teen mother? (Select all that apply) A. Computer based learning B. Videos C. Printed material D. Group discussion E. Models

A.B.C.D.E. Computer based learning; Videos; Printed material; Group discussion; Models Rationale: A number of factors influence learning at any age. One of the most significant considerations is developmental level. Teenage parents often have very different concerns and learn in a different way than older parents. Often grandparents are involved in the rearing of these children and must be able to review and understand the material. There is a wealth of new information that may not have been available when they became parents

Many teens wait until the second or even third trimester to seek prenatal care. The nurse should understand that the reasons behind this delay include (Select all that apply) A. Lack of realization that they are pregnant B. Uncertainty as to where to go for care C. Continuing to deny the pregnancy D. A desire to gain control over their situation E. Wanting to hide the pregnancy as long as possible

A.B.C.E. Lack of realization that they are pregnant; Uncertainty as to where to go for care; Continuing to deny the pregnancy; Wanting to hide the pregnancy as long as possible Rationale: These are all valid reasons for the teen to delay seeking prenatal care. An adolescent often has little to no understanding of the increased physiologic needs that a pregnancy places on her body. Once care is sought, it is often sporadic and many appointments are missed. The nurse should formulate a diagnosis that assists the pregnant teen to receive adequate prenatal care. The opposite is true. Planning for here pregnancy and impending birth actually provides some sense of control for the teen and increases feelings of competency. Receiving praise from the nurse when she attends her prenatal appointments will reinforce the young woman's positive self image

Transvaginal ultrasonography is often performed during the first trimester. While preparing your 6 week gestation patient for this procedure, she expresses concern over the necessity for this test. The nurse should explain that this diagnostic test may be necessary to determine which of the following? (Select all that apply) A. Multifetal gestation B. Bicornate uterus C. Presence and location of pregnancy (intrauterine or elsewhere) D. Amniotic fluid volume E. Presence of ovarian cysts

A.B.C.E.Multifetal gestation; Bicornate uterus; Presence and location of pregnancy (intrauterine or elsewhere); Presence of ovarian cysts Rationale: All of these conditions can be determined by transvaginal ultrasound in the first trimester of pregnancy. This procedure is also used for estimating gestational age, confirming fetal viability, identifying fetal abnormalities or chromosomal defects, and identifying the maternal abnormalities mentioned, as well as fibroids. Amniotic fluid volume is assessed during the second and third trimester. Conventional ultrasound would be used

A patient informs the nurse that she has taken cocaine throughout her pregnancy. The nurse needs to be alert to what signs of pregnancy complications? A. Abruptio placentae B. Preeclampsia C. Diabetes D. Preterm labor

A.B.D. Abruptio placentae; Preeclampsia; Preterm labor Rationale: Because cocaine causes vasoconstriction of placental vessels, the incidence of abruptio placentae increases. It stimulates uterine contractions resulting in increased incidence of premature rupture of membranes and preterm labor. Preeclampsia is also a complications of cocaine usage, but not diabetes.

Many communities now offer the availability of free standing birth centers to provide care for low risk women during pregnancy, birth, and postpartum. When counseling the newly pregnant woman regarding this option, the nurse should be aware that this type of care setting includes which advantages? (Select all that apply) A. Less expensive than acute care hospitals B. Access to follow up care for 6 weeks postpartum C. Equipped for obstetric emergencies D. Safe, home like births in a familiar setting E. Staffing by lay midwives

A.B.D. Less expensive than acute care hospitals; Access to follow up care for 6 weeks postpartum; Safe, home like births in a familiar setting Rationale: Women who are at low risk and desire a safe, home like birth are very satisfied with this type of care setting. The new mother may return to the birth center for postpartum follow up care, breastfeeding assistance, and family planning information for 6 weeks postpartum. Because birth centers do not incorporate advanced technologies into their services, costs are significantly less than those for a hospital setting. The major disadvantage of this care setting is that these facilities are not equipped to handle obstetric emergencies. Should unforeseen difficulties occur, the woman must be transported by ambulance to the nearest hospital. Birth centers are usually staffed by certified nurse-midwives (CNMs; however, in some states lay midwives may provide this service

Some problems associated with hospital births in the early 1960s include the following issues: A. Patient teaching was not valued B. Bonding was hindered due to strong medications given to the mother C. There was an increased use of midwives D. The father was not included in the process

A.B.D. Patient teaching was not valued; Bonding was hindered due to strong medications given to the mother; The father was not included in the process Rationale: The nurse's primary function was to follow medical orders, so teaching was not valued. Strong medications were given to the patient that left her heavily sedated. Fathers were usually sent to the waiting room. The use of lay midwives was declining at this time and nurse midwives were not well established

A pregnant woman reports that she works in a long term care setting and is concerned about the impending flu season. She asks about receiving the flu vaccine. As the nurse, you are aware that some immunizations are safe to administer during pregnancy. Which vaccines could this patient receive? (Select all that apply) A. Tetanus B. Hepatitis A and B C. Measles, mumps, and rubella (MMR) D. Influenza E. Varicella

A.B.D. Tetanus; Hepatitis A and B; Influenza Rationale: Inactivated vaccines such as those for tetanus, Hepatitis A, Hepatitis B and influenza are safe to administer for women who have a risk for contracting or developing the disease. Immunizations with live virus vaccines such as MMR, varicella (chickenpox), or smallpox are contraindicated during pregnancy because of the possible teratogenic effects on the fetus

Families are sometimes categorized into three types: (Select all that apply.) A. Traditional B. Nontraditional C. Low risk D. High risk

A.B.D. Traditional; Nontraditional; High Risk Rationale: Traditional families (also called nuclear families) are headed by two parents. The growing number of nontraditional families includes single-parent families, blended families, adoptive families, unmarried couples with children, multigenerational families, and homosexual parent families. Examples of high-risk families are those experiencing marital conflict and divorce, those with adolescent parents, those affected by violence against one or more of the family members, those involved with substance abuse, and those with a chronically ill child.

Diabetes refers to a group of metabolic diseases characterized by hyperglycemia resulting from defects in insulin action, insulin secretion, or both. Over time, diabetes causes significant changes in the microvascular and macrovascular circulations. These complications include: (select all that apply) A. Atherosclerosis. B. Retinopathy. C. IUFD. D. Nephropathy. E. Neuropathy.

A.B.D.E. Atherosclerosis; Retinopathy; Nephropathy; Neuropathy. Rationale: These structural changes are most likely to affect. a variety of systems, including the heart, eyes, kidneys, and nerves. Intrauterine fetal death (stillbirth) remains a major complication of diabetes in pregnancy, however, this is a fetal complication

In an effort to reduce prohibitive health care costs, many facilities have incorporated the use of unlicensed assistive personnel (UAP) into their care delivery model. Nurses supervising these employees must be aware of what each such employee is competent to do within his or her scope of practice. Which tasks can be delegated with supervision? (Select all that apply) A. Blood draws B. Medication administration C. Nursing assessment D. Housekeeping tasks E. Other diagnostic tests, such as electrocardiograms (ECG or EKGs)

A.B.D.E. Blood draws; Medication administration; Housekeeping tasks; Other diagnostic tests, such as electrocardiograms (ECG or EKGs) Rationale: With proper supervision and adequate instruction, unlicensed assistive personnel may perform all of these functions. In school settings, these personnel may be responsible for medication administration under the direction of the registered nurse. The nurse is always responsible for patient assessments and must make critical judgements to ensure patient safety. Use of the expert nurse to complete housekeeping or other mundane tasks is not a good use of human resources.

Throughout the world the rate of ectopic pregnancy has increased dramatically over the past 20 years. This is believed to be due primarily to scarring of the fallopian tube as a result of pelvic infection, inflammation, or surgery. The nurse who suspects that a patient has early signs of ectopic pregnancy should be observing her for symptoms such as (Select all that apply) A. Pelvic pain B. Abdominal pain C. Unanticipated heavy bleeding D. Vaginal spotting or light bleeding E. Missed period

A.B.D.E. Pelvic pain; Abdominal pain; Vaginal spotting or light bleeding; Missed period Rationale: A missed period or spotting can easily be mistaken by the patient as early signs of pregnancy. More subtle signs depend on exactly where the implantation occurs. The nurse must be thorough in her assessment because pain is not a normal symptom of early pregnancy. As the fallopian tube tears open and the embryo is expelled, the patient often exhibits severe pain accompanied by intrabdominal hemorrhage. This may progress to hypovolemic shock with minimal or even no external bleeding. In about half of women, shoulder and neck pain occurs due to irritation of the diaphragm from the hemorrhage

European American women are likely to believe which of the following regarding pregnancy and childbirth? (Select all that apply) A. The father of the baby should be actively involved in the labor and birth B. Pregnancy requires medical attention so prenatal care should begin early in pregnancy C. Birthing at home should be valued D. Pregnant women should participate in childbirth education E. The doctor is the head of the obstetrical care team F. Mothers should not begin to breastfeed until milk comes in

A.B.D.E. The father of the baby should be actively involved in the labor and birth; Pregnancy requires medical attention so prenatal care should begin early in pregnancy; Pregnant women should participate in childbirth education; The doctor is the head of the obstetrical care team Rationale: birthing in an institutional setting is valued and breastfeeding should start as soon as possible after birth

When communicating with an abused woman, which of the following statements should be avoided? (Select all that apply) A. Why do you think your husband hits you even though you are pregnant B. I cannot believe how terrible your husband is being to you C. The violence you are experiencing now is likely to continue and to get even worse as your pregnancy progresses D. Tell me why you did not go to to the shelter I recommended to you; they are very helpful and you would have avoided this beating if you had gone. E. Next time you come in for care, I want you to be sure to bring your husband so I can talk to him myself F. I am afraid for your safety and the safety of your other children

A.B.D.E. Why do you think your husband hits you even though you are pregnant; I cannot believe how terrible your husband is being to you; Tell me why you did not go to to the shelter I recommended to you; they are very helpful and you would have avoided this beating if you had gone; Next time you come in for care, I want you to be sure to bring your husband so I can talk to him myself Rationale; asking why questions re-victimizes and blames the victim; the nurse should not talk negatively about the abuser nor talk directly to him about suspicions of abuse

An anesthesiologist is preparing to being a continuous epidural block using a combination local anesthetic and opioid analgesic as a pain relief measure for a laboring woman. Nursing measures related to this type of nerve block would include which of the following? (Select all that apply) A. Assist the woman into a modified Sim's position or upright position with back curved for administration of the block B. Alternate her position from side to side every hour C. Assess the woman for headaches because they commonly occur in the postpartum period if an epidural is used for labor D. Assist the woman to urinate every 2 hours during labor to prevent bladder distention E. Prepare the woman for use of forceps- or vacuum- assisted birth because she will be unable to bear down F. Assess blood pressure frequently because severe hypotension can occur

A.B.D.F.Assist the woman into a modified Sim's position or upright position with back curved for administration of the block; Alternate her position from side to side every hour; Assist the woman to urinate every 2 hours during labor to prevent bladder distention; Assess blood pressure frequently because severe hypotension can occur Rationale: spinal headache is rare because the dura is not punctured; using a combined anesthetic analgesic and reducing dosage can allow a woman to push when the time is right

A number of major hormones are necessary for health reproduction to occur. These hormones are produced by a number of different organs. The anterior pituitary gland is responsible for producing (select all that apply) A. Follicle stimulating hormone (FSH) B. Luteinizing hormone (LH) C. Gonadotropin-releasing hormone (GnRH) D. Oxytocin E. Prolactin

A.B.E. Follicle stimulating hormone (FSH); Luteinizing hormone (LH); Prolactin Rationale: FSH and LH are both produced by the anterior pituitary gland. Both of these hormones assist in the stimulation and maturation of the ovarian follicle. Prolactin is also produced by the anterior pituitary and is required for milk production (lactogenesis) to occur. GnRH is produced by the hypothalamus and stimulates the release of FSH and LH. Oxytocin is produced by the posterior pituitary gland and is responsible for stimulating uterine contractions during birth

The nurse is caring for a child from a Middle Eastern family. Which interventions should the nurse include in planning care? (Select all that apply) A. Include the father in the decision making B. Ask for a dietary consult to maintain religious dietary practices C. Plan for a male nurse to care for a female patient D. Ask the housekeeping staff to interpret if needed E. Allow time for prayer

A.B.E. Include the father in the decision making; Ask for a dietary consult to maintain religious dietary practices; Allow time for prayer Rationale: The man is typically the head of the household in Muslim families. So the father should be included in all decision making. Muslims do not eat pork and do not use alcohol. Many are vegetarians. The dietitian should be consulted for dietary preferences. Compulsory prayer is practiced several times throughout the day. The family should not be interrupted during prayer, and treatments should not be scheduled during this time. Muslim women often prefer a female health care provider because of laws of modesty; therefore the female patient should not be assigned a male nurse. A housekeeping staff member should not be asked to interpret. When interpreters are used, the should be of the same country and religion, if possible, because of regional differences and hostilities

Chromosomes are composed of genes, which are composed of DNA. Abnormalities are either numerical or structural in nature. Which abnormalities are structural? Select all that apply A. Part of a chromosome is missing B. The material within a chromosome is rearranged C. One or more sets of chromosomes are added D. Entire single chromosome is added E. Two chromosomes adhere to each other

A.B.E. Part of a chromosome is missing; The material within a chromosome is rearranged; Two chromosomes adhere to each other Rationale: It is critical to remember that characteristics of structural abnormalities include part of a chromosome missing or added, rearrangement of material within chromosomes, two chromosomes that adhered to each other, and fragility of a specific site on the X chromosome The addition of a single chromosome (trisomy), the deletion of a single chromosome (monosomy), and one or more added sets of chromosomes (polyploidy) are numerical abnormalities.

An integrative health care approach implies which of the following? (select all that apply) A. The focus is on the whole person B. Conventional Western modalities of treatment are not included C. The beliefs, values, and desires of the patient in terms of health and health care are respected D. Patient autonomy is limited in terms of choosing alternative therapies E. The patient's disease complex is the primary consideration when choosing treatment approaches

A.C. The focus is on the whole person; The beliefs, values, and desires of the patient in terms of health and health care are respected Rationale: Conventional Western modalities are included. patient autonomy and decision making is encouraged, and the whole patient and not just the disease process is the primary consideration

A woman is admitted with a diagnosis of hyperemesis gravidarum. The nurse is assessing for deficient fluid and signs of dehydration. (Select all that apply.) A. Decreased urinary output B. Urine specific gravity of 1.015 C. Non-elastic skin turgor D. Constipation

A.C.D. Decreased urinary output; Non-elastic skin turgor; Constipation Rationale: Decreased urinary output, non-elastic skin turgor, and constipation are signs of dehydration. Specific gravity with dehydration runs greater than 1.025. A specific gravity of 1.015 would be diluted urine.

Women with cardiomyopathy have no underlying heart disease, but symptoms of cardiac decompensation appear during the last weeks of pregnancy or from 2 to 20 weeks postpartum. The following are symptoms of congestive heart failure. (Select all that apply.) A. Dyspnea B. Headache C. Weakness D. Heart palpitations

A.C.D. Dyspnea; Weakness; Heart palpitations Rationale: Cardiomyopathy in the peripartum or postpartum period is a rare condition exclusively associated with pregnancy after exclusion of other causes. The symptoms are those of congestive heart failure: dyspnea, edema, weakness, chest pain, heart palpitations. Cardiomyopathy may suddenly appear in a woman who has been healthy. Headache is not a symptom of congestive heart failure.

Along with gas exchange and nutrient transfer, the placenta produces many hormones necessary for normal pregnancy. These include (select all that apply) A. Human chorionic gonadotropin (hCG) B. Insulin C. Estrogen D. Progesterone E. Testosterone

A.C.D. Human chorionic gonadotropin (hCG); estrogen; progesterone Rationale: hCG causes the corpus luteum to persist and produce the necessary estrogens and progesterone for the first 6 to 8 weeks. Estrogens cause enlargement of the womans uterus and breasts; cause growth of the ductal system in the breasts; and as term approaches, play a role in the initiation of labor. Progesterone causes the endometrium to change, providing early nourishment. Progesterone also protects against spontaneous abortion by suppressing maternal reactions to fetal antigens and reduces unnecessary uterine contractions. Other hormones produced by the placenta include hCT, hCA, and a number of growth factors. Human placental lactogen promotes normal nutrition and growth of the fetus and maternal breast development for lactation. This hormone decreases maternal insulin sensitivity and utilization of glucose, making more glucose available for fetal growth. If a Y chromosome is present in the male fetus, hCG causes the fetal testes to secrete testosterone necessary for the normal development of male reproductive structures

A nurse is teaching a class of young women in a drug rehabilitation program about the risks associated with using illicit drugs during pregnancy. Which of the following statements if made by the students indicate that they understood the nurse's instructions? (Select all that apply) A. My baby won't get enough oxygen if I smoke marijuana B. Cocaine can make my baby grow too big and make my pregnancy last too long C. Cocaine can make the placenta separate from my baby too early D. If I use heroin during pregnancy my baby will go through withdrawal after birth E. Methamphetamine exposure can make my baby's head too big F. My baby might get birth defects, especially in the heart and lungs, if I use PCP

A.C.D. My baby won't get enough oxygen if I smoke marijuana; Cocaine can make the placenta separate from my baby too early; If I use heroin during pregnancy my baby will go through withdrawal after birth Rationale: cocaine reduces fetal growth and increases the risk for preterm labor and birth; babies exposed to methamphetamines can have smaller heads; the major concern with using phencyclidine is polydrug abuse and neuorbehavioral effects on the neonate

Signs and symptoms of maternal hypoglycemia include the following. (Select all that apply.) A. Shakiness (tremors) B. Dry mouth, excessive thirst C. Sweating D. Headache

A.C.D. Shakiness (tremors); Sweating; Headache Rationale: Signs and symptoms include shakiness (tremors), sweating, headache, and also pallor; cold, clammy skin, disorientation, irritability, hunger, and blurred vision. Dry mouth and excessive thirst are symptoms of hyperglycemia not hypoglycemia.

The nurse is providing discharge information for a family who recently moved from China and who say they understand English quite well. Which approaches would be best for the nurse to use for effective teaching? (Select all that apply.) A. Address the oldest family member first before beginning any teaching. B. Use "yes" and "no" questions for easiest feedback from the family. C. Have the family members explain what has been said. D. Have the family members demonstrate a skill if one will need to be done in the home. E. Avoid staring at the family members directly in the eye. F. Speak clearly, but a little louder, so they can understand the language better.

A.C.D.E. Address the oldest family member first before beginning any teaching; Have the family members explain what has been said; Have the family members demonstrate a skill if one will need to be done in the home; Avoid staring at the family members directly in the eye. Rationale: The elders of the family are highly respected and should be addressed initially. Other family members should be encouraged to learn and participate. Even people who have been in the United States for many years often do not feel competent in English. People from Southeast Asia avoid prolonged eye contact, which they consider rude. It is better to glance at their face and eyes periodically. The nurse should avoid "yes" or "no" questions and have the family explain or demonstrate understanding of the teaching provided. Speaking louder does not help someone understand the language better.

In some Middle Eastern and African culture, female genital mutilation is a prerequisite for marriage. Women who now live in North America need care from nurses who are knowledgeable about the procedure and comfortable with the abnormal appearance of her genitalia. When caring for this woman, the nurse can formulate a diagnosis with the understanding that the woman may be at risk for (Select all that apply) A. Obstructed labor B. Increased signs of pain response C. laceration D. hemorrhage E. infection

A.C.D.E. Obstructed labor; laceration; hemorrhage; infection Rationale: The woman is at risk for all of these complications. Female genital mutilation, cutting, or circumcision involves removal of some or all of the external female genitalia. The labia majora are often stitched together over the vaginal and urethral opening as part of this practice. Enlargement of the vaginal opening may be performed before or during the birth. The woman is unlikely to give any verbal or nonverbal signs of pain. This lack of response does not indicate lack of pain. In fact, the pelvic examinations are likely to be very painful because the introitus is so small and inelastic scar tissue makes thee area especially sensitive. A pediatric speculum may be necessary, and the patient should be made as comfortable as possible

External electronic fetal monitoring will be used for a woman just admitted to the labor unit in active labor. Guidelines the nurse should follow when implementing this form of monitoring would be which of the following (select all that apply) A. Use Leopold maneuvers to determine the correct placement of the ultrasound transducer B. Assist woman to maintain a dorsal recumbent position to ensure accurate monitor tracings for evaluation C. Reposition the tocotransducer, cleanse abdomen, and reapply gel when the fetus changes position D. Tell the woman she can perform effleurage along the sides of her abdomen E. Palpate the fundus to estimate the intensity of uterine contractions

A.C.D.E. Use Leopold maneuvers to determine the correct placement of the ultrasound transducer; Reposition the tocotransducer, cleanse abdomen, and reapply gel when the fetus changes position; Tell the woman she can perform effleurage along the sides of her abdomen; Palpate the fundus to estimate the intensity of uterine contractions Rationale: the tocotransducer is always placed over the fundus but the ultrasound transducer, which requires the use of gel, should be repositioned when the fetus moves or as needed; a woman's position should be changed even though it may mean repositioning the transducers

A vaginal examination during labor reveals the following information, -1, 75%, 3cm. An accurate interpretation of this data would include which of the following (Select all that apply) A. Attitude: flexed B. Station: 3cm below the ischial spines C. Presentation: cephalic D. Lie: longitudinal E. Effacement: 75% complete F. Dilation: 9cm more to reach full dilation

A.C.D.E. station is 1cm above the ischial spines (-1); 7cm more to reach full dilation at 10cm

A patient who has undergone a D&C for early pregnancy loss is likely to be discharged the same day. The nurse must ensure that vital signs are stable, that bleeding has been controlled, and that the woman has adequately recovered from the administration of anesthesia. In order to promote an optimal recovery, discharge teaching should include (Select all that apply) A. Iron supplementation B. Resumption of intercourse at 6 weeks post procedure C. Referral to a support group if necessary D. Expectation of heavy bleeding for at least 2 weeks E. Emphasizing the need for rest

A.C.E. Iron supplementation; Referral to a support group if necessary; Emphasizing the need for rest Rationale: The woman should be advised to consume a diet high in iron and protein. For many women, iron supplementation also is necessary. Acknowledge that the patient has experienced a loss, albeit early. She can be taught to expect mood swings and possibly depression. Referral to a support group, clergy, or professional counseling may be necessary. Discharge teaching should emphasize the need for rest. Nothing should be placed in the vagina for 2 weeks postprocedure. This includes tampons and vaginal intercourse. The purpose of this recommendation is to prevent infection. Should infection occur, antibiotics may be prescribed. The patient should expect a scant, dark discharge for 1 to 2 weeks. Should heavy, profuse, or bright bleeding occur she should be instructed to contact her provider

Percutaneous umbilical cord sampling (PUBS), also called cordocentesis, involves the aspiration of fetal blood from the umbilical cord for prenatal diagnosis or therapy. Major indications include (Select all that apply) A. Rh disease B. fetal well being C. Infection D. Lung maturity E. Karyotyping

A.C.E. Rh disease; infection; karyotyping Rationale: These are all indications for PUBS. Another indication is for diagnosis or disorders that require fetal blood for testing. Although used to develop a karyotype (chromosome evaluation) other DNA analysis techniques might also be utilized. NST or BPP are used to determine fetal well being. An amniocentesis is done in order to determine lung maturity

A 20 year old woman tells the nurse that she performs breast self examinations (BSE) on a regular basis. The nurse evaluates the woman's understanding of BSE and ability to perform the technique correctly. Which of the following actions by the woman indicate that she needs further instructions regarding BSE (select all that apply)? A. Performs BSE every month on the first day of her menstrual cycle B. Observes the size of her breasts, the direction of her nipples, and appearance of her skin including the presence of dimpling when looking at her breasts in the mirror C. Lies down on her bed and puts a pillow under the shoulder of the breast that she is going to palpate; then she places the arm on that side under her head D. Uses the tips of her four fingers to palpate her breast E. Palpates her breast, using an overlapping circular pattern around her entire breast F. Palpates her breasts and up into her axilla while taking a shower

A.D. Performs BSE every month on the first day of her menstrual cycle; Uses the tips of her four fingers to palpate her breast Rationale: the woman should perform BSE every month at the end of menstruation when the breasts are the least tender; she should use the finger pads of her three middle fingers for palpation

A nurse is instructing a group of primigravid women about the onset of labor. Which of the following signs could the women observe preceding the onset of their labors (Select all that apply) A. Urinary frequency B. Weight gain of 2g C. Quickening D. Energy surge E. Bloody show F. Shortness of breath

A.D.E. quickening refers to the woman's first perception of fetal movement at 16 to 18 weeks of gestation; urinary frequency, lightening, weight loss of 0.5 to 1 kg occur to signal that the onset of labor is near; backache, stronger Braxton Hicks and blood show are also noted; shortness of breath is relieved once lightening occurs reducing pressure on the diaphragm

A woman is 35 weeks' pregnant during her clinic visit. She complains of numerous vaginal infections during the pregnancy. She tells the nurse, "I'm afraid I have diabetes, because I have some infections." The best response by the nurse would be A. "Diabetes is a possibility. I will set you up for testing." B. "A vaginal infection is a symptom of diabetes, but it also is a problem with normal pregnancies due to the changes in your vaginal area." C. "Itching is a problem with pregnancies and it makes you think you have an infection. The physician can order you some cream to help with the itching and pain." D. "This seems to be a concern with all of our patients today."

B. "A vaginal infection is a symptom of diabetes, but it also is a problem with normal pregnancies due to the changes in your vaginal area." Rationale: During pregnancy, the glycogen levels of the vaginal area increase. This favors the growth of yeast-causing infections. Diabetes is a possibility, but there are other considerations that need to be assessed first. Vulva itching is not a common problem with pregnancy. These symptoms should be investigated for the cause and treated.

When doing genetic counseling with a pregnant woman, the nurse will need to do additional teaching if the patient states A. "Because my husband and I are not Jewish or French-Canadian, the chance that our baby has Tay-Sachs is less." B. "Both my husband and I are B blood type, so our baby will have to be B type also." C. "My husband is color blind; if we have a little girl, she probably will not be affected, but if I have a little boy, he may be affected." D. "Since both my husband and I carry the gene for cystic fibrosis, my baby has a 50% chance of also being a carrier for the gene."

B. "Both my husband and I are B blood type, so our baby will have to be B type also." Rationale: B blood type is dominant; however, both parents may have an "O" recessive gene that can be passed on to a child. Families of Jewish descent, French-Canadians, and those with roots in Eastern Europe have a higher incident of Tay-Sachs. Color blindness is an X-linked recessive disorder; girls tend to be carriers, and boys will develop the problem. Cystic fibrosis is an autosomal recessive disorder, so if both parents are carriers, then there is a 50% chance that an offspring will be a carrier.

If a woman's menstrual cycle began on June 2 and normally lasts 28 days, ovulation would most likely occur on June A. 10 B. 16 C. 21 D. 29

B. 16 Rationale: June 10 would just be 8 days into the cycle and too early for ovulation. Ovulation occurs approximately 12 to 14 days after the beginning of the menstrual period in a 28 day cycle. Ovulation normally occurs approximately 14 days before the beginning of the next period. June 21 would be 18 days into the cycle. Ovulation should have already occurred at this point. June 29 would be 27 days into the cycle and almost time for the next period

The breastfeeding woman whose recommended prepregnant caloric intake was 2000 calories per day needs how many calories per day to meet her current needs? A. 2300 B. 2500 C. 2750 D. 3000

B. 2500 Rationale: 2300 calories is not enough calories to meet her needs. The increase for a breastfeeding mother is 500 calories above her recommended prepregnant caloric intake. 2750 calories may be too many calories and may lead to weight gain. 3000 calories is too many for this mother and will lead to weight gain

When evaluating the external fetal monitoring tracing of a woman whose labor is being induced, the nurse identifies signs of persistent late deceleration patterns and begin intrauterine resuscitation interventions. Which of the following reflects that the appropriate interventions were implemented in the recommended order of priority? 1. Increase rate of maintenance intravenous solution 2. Palpate uterus for tachysystole 3. Discontinue Pitocin infusion 4. Change maternal position to a lateral position; then elevate her legs if woman is hypotensive 5. Administer oxygen at 8 to 10L/min with a tight face mask A. 2,1,5,4,3 B. 4,1,2,3,5 C. 5,3,4,1,2 D. 4,5,1,2,3

B. 4,1,2,3,5 Change maternal position to a lateral position; then elevate her legs if woman is hypotensive Increase rate of maintenance intravenous solution Palpate uterus for tachysystole Discontinue Pitocin infusion Administer oxygen at 8 to 10L/min with a tight face mask

A woman has been admitted to the birthing unit with a diagnosis of spontaneous abortion. She has increased bleeding and is having her pads weighed to estimate the blood loss. The weight of an unused pad is 1.5 grams, the pads used between 7 AM and 9 AM weigh 4.5, 6.5, 10, 15, and 11.5 grams. What is the estimated blood loss? A. 20 mL B. 40 mL C. 60 mL D. Unable to determine with the information provided

B. 40 mL Rationale: When weighing pads or linen, a difference of 1 gram of weight equals approximately 1 mL of fluid volume. There was a 40-gram difference in the pads, and therefore a 40-mL blood loss

Which pregnant adolescent is most at risk for a nutritional deficit during pregnancy? A. A 15 year old of normal height and weight B. A 17 year old who is 10 pounds underweight C. A 16 year old who is 10 pounds overweight D. A 16 year old of normal height and weight

B. A 17 year old who is 10 pounds underweight Rationale: A 15 year old has special nutritional needs during pregnancy, but she is not at the highest risk for deficiency. The adolescent who is pregnant and underweight is most at risk, because she is already deficient in nutrition and must now supply the nutritional intake for both herself and her fetus. An overweight pregnant teen is at risk for deficiency, but is not the highest risk. Being underweight is the most risky because she is already deficient. A 16 year old has special nutritional needs during pregnancy, but she is not the highest risk for deficiency

Which woman would be most likely to seek prenatal care? A. A 15 year old who tells her friends, I don't believe I'm pregnant B. A 20 year old who is in her first pregnancy and has access to a free prenatal clinic C. A 28 year old who is in her second pregnancy and abuses drugs and alcohol D. A 30 year old who is in her fifth pregnancy and delivered her last infant at home with the help of her mother and sister

B. A 20 year old who is in her first pregnancy and has access to a free prenatal clinic Rationale: Being in denial about the pregnancy will prevent her from seeking health care. The patient who acknowledges the pregnancy early, has access to health care, and has no reason to avoid health care is most likely to seek prenatal care. Substance abusers are less likely to seek health care. Some women see pregnancy and delivery as a natural occurrence and do not seek health care

During a home visit, the mother of a 1 week old infant son tells the nurse that she is very concerned about whether her baby is getting enough breast milk. The nurse would tell this mother that at 1 week of age a well nourished newborn should exhibit which of the following? A. Weight gain sufficient to reach his birthweight B. A minimum of three bowel movements each day C. Approximately 10 to 12 wet diapers each day D. Breast feeding at a frequency of every 4 hours or about 6 times each day

B. A minimum of three bowel movements each day Rationale: birth weight is regained in 10 to 14 days; 6 to 8 wet diapers are expected at this time; should be fed every 2 to 3 hours for a total of 8 to 10 times a day.

The nurse is working in an OB/GYN office, where part of her duties include obtaining a patient's history and performing an initial assessment. Which woman is likely to be referred for genetic counseling after her first visit? A. A pregnant woman who will be 40 years older when her infant is born B. A woman whose partner is 38 years of age C. A patient who carries a Y linked disorder D. An anxious woman with a quadruple screening result

B. A woman whose partner is 38 years of age Rationale: Pregnant women who will be 35 years of age or older when the infant is born should be referred to a genetic counselor. Men who father children after the age of 40 should also be sent for referral. Men who father children in their 5th decade or later are more likely to have offspring with a new autosomal dominant mutation. Women who are carriers of X linked disorders should be referred for genetic counseling. Women do not carry a Y chromosome. Pregnant women with abnormal prenatal screening results, such as triple or quadruple screening, or suspicious ultrasound findings may require referral to other health care specialists. This patient simply requires further support and encouragement

Which complaint by a patient at 35 weeks of gestation requires additional assessment? A. Shortness of breath when climbing stairs B. Abdominal pain C. Ankle edema in the afternoon D. Backache with prolonged standing

B. Abdominal pain Rationale: Shortness of breath is an expected finding by 35 weeks. Abdominal pain may indicate preterm labor or placental abruption. Ankle edema in the afternoon is a normal finding at this stage or pregnancy. Backaches while standing is a normal finding during the later stages of pregnancy.

Fertilization of the ovum takes place in which part of the fallopian tube? A. Interstitial portion B. Ampulla C. Isthmus D. Infundibulum

B. Ampulla Rationale: The ampulla is the wider middle part of the tube lateral to the isthmus and is where fertilization occurs. The interstitial portion runs into the uterine cavity and lie within the uterine wall. The isthmus is the narrow part of the tube adjacent to the uterus. If fertilization occurs in these sections, it is too close to the site of implantation, and the fertilized ovum would not have time to properly develop prior to implantation. The infundibulum is the wide, funnel-shaped terminal end of the tube toward the ovaries. If fertilization occurred in this section, the fertilized ovum could travel out into the abdominal cavity.

Fertilization of the ovum takes place in which part of the fallopian tube? A. Interstitial portion B. Ampulla C. Isthmus D. Infundibulum

B. Ampulla Rationale: The interstitial portion runs into the uterine cavity. This area is too close to the uterine body for fertilization to occur; it would lead to improper placement for implantation. The ampulla is the wider middle part of the fallopian tube lateral to the isthmus and is where fertilization occurs. The isthmus is the narrowest portion of the tube. The infundibulum is the end of the tube that opens into the abdominal cavity. Fertilization at this area may lead to an abdominal pregnancy

A woman with severe preeclampsia is being treated with bed rest and intravenous magnesium sulfate. The drug classification of this medication is A. tocolytic B. Anticonvulsant C. Antihypertensive D. Diuretic

B. Anticonvulsant Rationale: A tocolytic drug does slow the frequency and intensity of uterine contractions but is not used for that purpose in this scenario. Anticonvulsant drugs act by blocking neuromuscular transmission and depress the central nervous system to control seizure activity. Decreased peripheral blood pressure is a therapeutic response (side effect) of the anticonvulsant magnesium sulfate. Diuresis is a therapeutic response to magnesium sulfate

The purpose of initiating contractions in a CST is to A. Determine the degree of fetal activity B. Apply a stressful stimulus to the fetus C. identifying fetal acceleration patterns D. increase placental blood flow

B. Apply a stressful stimulus to the fetus Rationale: The NST and biophysical profiles look at fetal movements. The CST involves recording the response of the FHR to stress induced by uterine contractions. The NST looks at fetal heart accelerations with fetal movements.The CST records the fetal response to stress. It does not increase placental blood flow

With regard to the structure and function of the placenta, the maternity nurse should be aware that: A. As the placenta widens, it gradually thins to allow easier passage of air and nutrients B. As one of its early functions, the placenta acts as an endocrine gland C. The placenta is able to keep out most potentially toxic substances, such as cigarette smoke, to which the mother is exposed D. Optimal blood circulation is achieved through the placenta when the woman is lying on her back or standing

B. As one of its early functions, the placenta acts as an endocrine gland Rationale: The placenta widens until week 20 and continues to grow thicker. The placenta produces four hormones necessary to maintain the pregnancy. Toxic. substances such as nicotine and carbon monoxide readily cross the placenta into the fetus. Optimal circulation occurs when the woman is lying on her side.

A women's health nurse practitioner is going to perform a pelvic exam on a female patient. Which of the following nursing actions would be least effective in enhancing the patient's comfort and relaxation during the examination? A. Encourage the patient to ask questions and express feelings and concerns before and after the examination B. Ask the patient questions as the examination is performed C. Allow the patient to keep her shoes and socks on when placing her feet in the stirrups D. Instruct the patient to place her hands over her diaphragm and take deep, slow breaths

B. Ask the patient questions as the examination is performed Rationale: do not ask questions during the examination because it may distract the patient, interfering with relaxation measures she may be using; only offer explanations as needed during the examination

A woman has just been admitted with a diagnosis of hyperemesis gravidarum. She has been unable to retain any oral intake and as a result has lost weight and is exhibiting signs of dehydration with electrolyte imbalance and acetonuria. The care management of this woman would include which of thee following? A. Administering labetalol to control nausea and vomiting B. Assessing the woman's urine for ketones C. Avoiding oral hygiene until the woman is able to tolerate fluids D. Providing small frequent meals consisting of bland foods and warm fluids together once the woman begins to respond to treatment

B. Assessing the woman's urine for ketones Rationale: labetalol is a beta blocker used for hypertension; oral hygiene is important when NPO and after vomiting episodes to maintain the integrity of oral mucosa; taking fluids between, not with, meals reduces nausea, thereby increasing tolerance for oral nutrition

Oogenesis, the process of egg formation, begins during fetal life in the fenmale. Which statement related to ovum formation is correct? A. Two million primary oocytes will mature B. At birth, all ova are contained in the female ovaries C. The oocytes complete their division during fetal life D. Monthly, at least two oocytes mature

B. At birth, all ova are contained in the female ovaries Rationale: Only 400 to 500 ova will mature during the approximately 35 years of a womans reproductive life. All of the cells that may undergo meiosis in a womans lifetime are contained in the ovaries at birth. The primary oocytes begin their first meiotic division during fetal life but remain suspended until puberty. Every month, one primary oocyte matures and completes meiotic division yielding two unequal cells

A nurse observes that parents discuss rules with their children when the children do not agree with the rules. Which style of parenting is being displayed? A. Authoritarian B. Authoritative C. Permissive D. Disciplinarian

B. Authoritative Rationale: A parent who expects children to follow rules without questioning is using an authoritarian parenting style. A parent who discusses the rules with which children do not agree is using an authoritative parenting style. A parent who does not consistently enforce rules and allows the child to decide whether he or she wishes to follow rules is using a permissive parenting style. A disciplinarian style would be similar to the authoritarian style

Three servings of milk, yogurt, or cheese plus two servings of meat, poultry, or fish will adequately supply the recommended amount of protein for the pregnant woman. Many clients are concerned about the increased levels of mercury in fish and may be afraid to include this source of nutrients in their diet. Sound advice by the nurse to assist the client in determining which fish is safe to consume includes: A. Canned white tuna is a preferred choice B. Avoid shark, swordfish, and mackerel C. Fish caught in local waterways is the safest D. Salmon and shrimp contain high levels of mercury

B. Avoid shark, swordfish, and mackerel Rationale: High levels of mercury can harm the developing nervous system of the fetus. It is essential for the nurse to assist the patient in understanding the differences between numerous sources of this product. A pregnant patient can eat as much as 12 ounces a wek of canned light tuna; however, canned white, albacore, or tuna steaks contain higher levels of mercury and should be limited to no more than 6 ounces per week. As a precaution the pregnant patient should avoid eating all of these as well as the less common tilefish. This is a common misconception. Pregnant women and mothers of young children should check with local advisories about the safety of fish caught by families and friends in nearby bodies of water. If no information is available, these fish sources should be avoided, limited to less than 6 ounces, or the only fish consumed that week. Commercially caught fish that is low in mercury includes salmon, shrimp, pollock, or catfish. The pregnant patient may eat up to 12 ounces per week

A breastfeeding woman asks the nurse about what birth control she should use during the postpartum period. Which is the best recommendation for a safe, yet effective method during the first 6 weeks after birth? A. Combination oral contraceptive that she used before she was pregnant B. Barrier method using a combination of a condom and spermicide foam C. Resume using the diaphragm she used prior to getting pregnant D. Complete breastfeeding- baby only receives breast milk for nourishment

B. Barrier method using a combination of a condom and spermicide foam Rationale: a combination hormonal contraceptivee could decrease the milk supply if given before lactation is well established during the first 6 weeks after birth; after 6 weeks, a progestin only contraceptive could be used because it is the least likely hormonal contraceptive to affect lactation; even complete breastfeeding is not considered to be reliable method because ovulation can occur unexpectedly even before the first menstrual period; diaphragm used before pregnancy would have to be checked to see whether it fits properly before the woman uses it again

In many settings, the nurses perform nonstress tests (NST), conduct an initial assessment of the woman and begin necessary interventions for nonreassuring results. These nursing procedures are accomplished after additional education and training under established protocols. Which nursing action is necessary when preparing the patient for the NST? A. Position the woman on her left side B. Be certain the woman is seated comfortably in a reclining chair C. Encourage the woman to drink one liter of water prior to the test D. Place conduction gel on the womans abdomen with one belt

B. Be certain the woman is seated comfortably in a reclining chair Rationale: The woman should be seated in a reclining chair or in a semi-Fowler position if on a bed or stretcher. To correctly position the pregnant patient for an NST, the nurse should place the woman in a reclining chair. or in semi-Fowler position with a lateral tilt. This will optimize uterine perfusion and prevent supine hypotension. This is the appropriate preparation for abdominal ultrasonography. Caregivers sometimes suggest that the women drink juice in order to increase her glucose level and stimulate fetal movement. Although this practice is common, research has not proven it effective (C). Conduction gel should be applied along with two belts. One belt has a doppler transducer to detect the fetal heart rate. The other belt has a tocodynamometer to detect fetal movement or uterine contractions

After induction of a spinal block in preparation for an elective cesarean birth, a woman's blood pressure decreases from 124/76 to 96/60. The nurse's initial action would be to do which of the following? A. Administer a vasopressor intravenously to raise the blood pressure B. Change the woman's position from supine to lateral C. Begin to administer oxygen by mask at 10 to 12L/min D. Notify the woman's health care provider

B. Change the woman's position from supine to lateral Rationale: changing the woman to a lateral position will enhance cardiac output and raise the blood pressure because compression of the abdominal aorta and vena cava is removed; administering oxygen and notifying the health care provider would follow along with increasing intravenous fluid administration; administration of a vasopressor, if ordered by the physician, would follow if the other measures are not sufficient to restore the blood pressure

What characteristic would most likely be found in a Mexican American family? A. Stoicism B. Close extended family C. Considering docile children weak D. Very interested in health promoting lifestyles

B. Close extended family Rationale: Although stoicism may be present in any family, Mexican-American families tend to be more expressive. Most Mexican-American families are very close, and it is not unusual for children to be surrounded by parents, siblings, grandparents, and godparents. It is important to respect this cultural characteristic and to see it as a strength, not a weakness. Considering docile children weak is a characteristic of Native Americans. Although everyone tends now to embrace more health promoting lifestyles, they are more prominent in Anglo-Americans

Which situation best describes a man trying on fathering behaviors? A. Spending more time with his siblings B. Coaching a little League baseball team C. Reading books on newborn care D. Exhibiting physical symptoms related to pregnancy

B. Coaching a little League baseball team Rationale: The man normally will seek closer ties with his father. Interacting with children and assuming the behavior and role of a father best describes a man trying on being a father. Men do not normally read information that is provided in advance. The nurse should be prepared to present the information after the baby is born, when it is more relevant. This is called couvade (D)

The nurse who coordinates and manages a patients care with other members of the health care team is functioning in the role of: A. Teacher B. Collaborator C. Researcher D. Advocate

B. Collaborator Rationale: Education is an essential role of todays nurse. The nurse functions as a teacher during prenatal care, during maternity care, and when teaching parents of children regarding normal growth and development. The nurse collaborates with other members of the health care team, often coordinating and managing the patients care. Care is improved by this interdisciplinary approach as nurses work together with dietitians, social workers, physicians, and others. Nurses contribute to their professional knowledge base by systematically investigating theoretic for practice issues and nursing. A nursing advocate is one who speaks on behalf of another. As the health professional who is closest to the patient, the nurse is in an ideal position to humanize care and to intercede on the patients behalf

In teaching the pregnant adolescent about nutrition, the nurse should A. Emphasize the need to eliminate common teen snack foods, because they are too high in fat and sodium. B. Determine the weight gain needed to meet adolescent growth and add 35 lb. C. Suggest that she not eat at fast-food restaurants, to avoid foods of poor nutritional value. D. Realize that most adolescents are unwilling to make dietary changes during pregnancy.

B. Determine the weight gain needed to meet adolescent growth and add 35 lb. Rationale: Changes in the diet should be kept at a minimum, and snacks should be included. Snack foods can be included in moderation and other foods added to make up for the lost nutrients. Adolescents should gain in the upper range of the recommended weight gain. They also need to gain weight that would be expected for their own normal growth. Eliminating fast foods will make her appear different to her peers. She should be taught to choose foods that add needed nutrients. Adolescents are willing to make changes; however, they still have need to be like their peers

The upper uterus is the best place for the fertilized ovum to implant because it is here that the A. Placenta attaches most firmly B. Developing baby is best nourished C. Uterine endometrium is softer D. Maternal blood flow is lower

B. Developing baby is best nourished Rationale: If the placenta attaches too deeply, it does not easily detach after birth. The uterine fundus is richly supplied with blood and has the thickest endometrium, both of which promote optimal nourishment of the fetus. Softness is not a concern with implantation; attachment and nourishment are the major concerns. The blood supply is rich in the fundus, which allows for optimal nourishment of the fetus

Physiologic anemia often occurs during pregnancy as a result of A. inadequate intake of iron B. Dilution of hemoglobin concentration C. The fetus establishing iron stores D. Decreased production of erythrocytes

B. Dilution of hemoglobin concentration Rationale: Inadequate intake of iron may lead to true anemia. When blood volume expansion is more pronounced and occurs earlier than the increase in red blood cells, the woman will have physiologic anemia, which is the result of dilution of hemoglobin concentration rather than inadequate hemoglobin. If the woman does not take an adequate amount of iron, true anemia may occur when the fetus pulls stored iron from the maternal system. There is an increased production of erythrocytes during pregnancy.

A nulliparous woman is in the active phase of labor and her cervix has progressed to 5cm dilation. The nurse caring for this woman evaluates the external monitor tracing and notes the following: decrease in FHR shortly after onset of several uterine contractions returning to baseline by the end of the contraction; shape is uniform. Based on these findings the nurse should do which of the following? A. Change the woman's position to her left side B. Document the finding on the woman's chart C. Notify the physician D. Perform a vaginal examination to check for cord prolapse

B. Document the finding on the woman's chart Rationale: the pattern described is an early deceleration pattern, which is considered to be benign, reassuring, and requiring no action other than documentation of the finding; it is associated with fetal head compression; changing a woman's position and notifying the physician would be appropriate if nonreassuring signs such as late or variable decelerations were occurring; prolapse of cord is associated with variable decelerations as a result of cord compression

A 40 year old woman at 18 weeks of gestation is having a triple marker test performed. She is obese, and her health history reveals that she is Rh negative. The primary purpose of this test is to screen for A. spina bifida B. Down syndrome C. gestational diabetes D. Rh antibodies

B. Down syndrome Rationale: the triple marker test is used to screen the older pregnant woman for the possibility that her fetus has Down syndrome; serum levels of alpha fetoprotein (AFP), unconjugated estriol, and hCG are measured; maternal serum alpha-fetoprotein alone is the screening test for open neural tube defects such as spina bifida; a 1 hours, 50g glucose test is used to screen for gestational diabetes; amniocentesis and Coombs testing would be used to check for Rh antibodies and sensitization

A woman is expecting her first baby in 7 months. During the nurse's assessment Anna continues to ask questions about changes in her body. The nurse can recommend which type of class to assist the woman with her questions? A. Preconception class B. Early pregnancy class C. Childbirth preparation class D. Parenting class

B. Early pregnancy class Rationale: An early pregnancy class focuses on the first two trimesters. They cover information on adapting to pregnancy, dealing with discomforts, and understanding what to expect. Preconception class is for couples thinking about having a baby. They are designed to help them prepare to have a healthy pregnancy. Childbirth preparation class focuses on preparation for labor and delivery. Parenting classes focus on care of the newborn.

A pictorial tool that can assist the nurse in assessing aspects of family life related to health care is the A. Genogram B. Ecomap C. Life cycle model D. Human development wheel

B. Ecomap Rationale: A genogram (also known as a pedigree) is a diagram that depicts the relationships of family members over generations. An ecomap is a pictorial representation of the family structures and their relationships with the external environment. The life cycle model in no way illustrates a family genogram. This model focuses on stages that a person reaches throughout his or her life. The human development wheel describes various stages of growth and development rather than a family's relationships to each other.

Basic structures of all major body organs are completed during which period of development? A. Zygote B. Embryo C. Fetus

B. Embryo Rationale: The embryonic period is from week 3 until week 8. During this time, the basic structures of the major body organs are formed. During the zygote period, cell division begins and implantation occurs. During the fetal period, major body organs mature.

Maternity nursing care that is based on knowledge gained through research and clinical trials is known as: A. Nurse sensitive indicators B. Evidence based practice C. Case management D. Outcomes management

B. Evidence based practice Rationale: Nurse sensitive indicators are patient care outcomes particularly dependent on the quality and quantity of nursing care provided. Evidence based practice is based on knowledge gained from research and clinical trials. Case management is a practice model that uses a systematic approach to identify specific patients, determine eligibility for care, and arrange access to services. The determination to lower health care costs while maintaining the quality of care has led to a clinical practice model known as outcomes management.

The primary expected outcome for care associated with the administration of magnesium sulfate would be met if the woman exhibits which of the following? A. Exhibits a decrease in both systolic and diastolic BP B. Experiences no seizures C. States that she feels more relaxed and calm D. Urinates more frequently, resulting in a decrease in pathologic edema

B. Experiences no seizures Rationale: magnesium sulfate is a CNS depressant given to prevent seizures.

Which finding in the urine analysis of a pregnant woman is considered a variation of normal? A. Proteinuria B. Glycosuria C. Bacteria in the urine D. Ketones in the urine

B. Glycosuria Rationale: The presence of protein could indicate kidney disease or preeclampsia. Small amounts of glucose may indicate physiologic spilling. Urinary tract infections are associated with bacteria in the urine. An increase in ketones indicates that the patient is exercising too strenuously or has an inadequate fluid and food intake

During which phase of the cycle of violence does the batterer become contrite and remorseful? A. Battering phase B. Honeymoon phase C. Tension building phase D. Increase drug taking phase

B. Honeymoon phase Rationale: During the battering phase violence actually occurs, and thee victim feels powerless. During the honeymoon phase, the battered person wants to believe that the battering will never happen again, and the batterer will promise anything to get back into the home. During the tension building phase, the batterer becomes increasingly hostile, swears, threatens, throws things, and pushes the battered. Often the batterer increases the use of drugs during the tension building phase

The nurse caring for a woman hospitalized for hyperemesis gravidarum should expect that initial treatment involves A. Corticosteroids to reduce inflammation B. IV therapy to correct fluid and electrolyte imbalances C. An antiemetic, such as pyridoxine, to control nausea and vomiting D. Enteral nutrition to correct nutritional defecits

B. IV therapy to correct fluid and electrolyte imbalances Rationale: Corticosteroids have been used successfully to treat refractory hyperemesis gravidarum, but they are not the expected initial treatment for this disorder. Initially, the woman who is unable to down clear liquids by mouth requires IV therapy for correction of fluid and electrolyte imbalances. Pyridoxine is vitamin B6, not an antiemetic. Promethazine, a common antiemetic. may be prescribed. In severe cases of hyperemesis gravidarum, enteral nutrition via a feeding tube may be necessary to correct maternal nutritional deprivation. This is not an initial treatment for this patient

The most important reason for evaluating the pattern of weight gain in pregnancy is to A. prevent excessive adipose tissue deposits B. Identify potential nutritional problems or complications of pregnancy C. Assess the need to limit caloric intake in obese women D. Determine cultural influences on the womans diet

B. Identify potential nutritional problems or complications of pregnancy Rationale: Excessive adipose tissue may occur with excess weight gain, but it is not the reason for monitoring the weight gain pattern.Deviations from the recommended pattern of weight gain may indicate nutritional problems or developing complications. It is important to monitor the pattern of weight gain for developing complications. The pattern of weight gain is not influenced by cultural influences

Which statement made by a lactating woman leads the nurse to believe that the woman might have lactose intolerance? A. I always have heartburn after I drink milk B. If I drink more than a cup of milk, I usually have abdominal cramps and bloating C. Drinking milk usually makes m break out in hives. D. Sometimes I notice that I have bad breath after I drink a cup of milk

B. If I drink more than a cup of milk, I usually have abdominal cramps and bloating Rationale: The woman with lactose intolerance is more likely to experience bloating and cramping, not heartburn. One problem that can interfere with milk consumption is lactose intolerance, which is the inability to digest milk sugar because of a lack of the enzyme lactose in the small intestine. Milk consumption may cause abdominal cramping, bloating, and diarrhea in such people, although many lactose intolerant individuals can tolerate small amounts of milk without symptoms. A woman who breaks out in hives after consuming milk is more likely to have a milk allergy. This woman should be advised to simply brush her teeth after consuming dairy products

A number of cardiovascular system changes occur during pregnancy. Which finding is considered normal for a woman during pregnancy? A. Cardiac output rises by 25% B. Increased pulse rate C. Increased blood pressure D. Decreased red blood cell (RBC) production

B. Increased pulse rate Rationale: Cardiac output increases by 50% with half of this rise occurring in the first 8 weeks gestation. The pulse increases about 15 to 20 beats/minute, which persists to term. In the first trimester, blood pressure usually remains the same as the prepregnancy level, but it gradually decreases up to about 20 weeks of gestation. During the second trimester, both the systolic and diastolic pressures decrease by about 5 to 10 mmHg. Production of RBCs accelerates during pregnancy

When caring for a preterm infant born at 30 weeks of gestation, the nurse should recognize which of the following as the newborn's primary nursing diagnosis A. Risk for infection related to decreased immune response B. Ineffective breathing pattern related to surfactant deficiency and weak respiratory muscle effort C. Ineffective thermoregulation related to immature thermoregulation center D. Imbalanced nutrition: less than body requirements related to ineffective suck and swallow

B. Ineffective breathing pattern related to surfactant deficiency and weak respiratory muscle effort Rationale: although A,C,D, are appropriate and important, respiration with adequate gas exchange takes precedence, especially because adequate surfactant is not produced before 32 weeks of gestation.

Which nursing intervention is necessary before a second trimester transabdominal ultrasound? A. Place the woman NPO for 12 hours B. Instruct the woman to drink 1 to 2 quarts of water C. Administer a soapsuds enema D. Perform an abdominal prep

B. Instruct the woman to drink 1 to 2 quarts of water Rationale: The woman needs a full bladder in order to elevate the uterus, therefore being NPO is not appropriate. When the uterus is still in the pelvis, visualization may be difficult. It is necessary to perform the test when the woman has a full bladder, which. provides a window through which the uterus and its contents can be viewed. A soapsuds enema is not necessary for this procedure. An abdominal prep is not necessary for this procedure

You are a maternal-newborn nurse caring for a mother who just delivered a baby born with Down syndrome. What nursing diagnosis would be the most essential in caring for the mother of this infant? A. Disturbed body image B. Interrupted family processes C. Anxiety D. Risk for injury

B. Interrupted family processes Rationale: Women commonly experience body image disturbances in the postpartum period, but this is unrelated to giving birth to a child with Down syndrome. This mother will likely experience a disruption in the family process related to the birth of a baby with an inherited disorder. Family disruption is common, and the strain of having a child with a serious birth defect may lead to divorce. Siblings may feel neglected because the child with a disorder requires more of their parents time and attention. The mother likely will have a mix of emotions that include anxiety, guilt, and denial, but this is not the most essential nursing diagnosis for this family. This nursing diagnosis is not applicable

A patient in her first trimester complains of nausea and vomiting. She asks, Why does this happen? The nurse's best response is A. It is due to an increase in gastric motility B. It may be due to changes in hormones C. It is related to an increase in glucose levels D. It is caused by a decrease in gastric secretions

B. It may be due to changes in hormones Rationale: Gastric motility decreases during pregnancy. Nausea and vomiting are believed to be caused by increased levels of hormones, decreased gastric motility, and hypoglycemia. Glucose levels decrease in the first trimester. Gastric secretions do decrease, but this is not the main cause of nausea and vomiting.

The karyotype of a person is 47, XY, +21. This person is a: A. Normal male. B. Male with Down syndrome. C. Normal female D. Female with Turner syndrome.

B. Male with Down syndrome. Rationale: A normal male has 46 chromosomes (A). This person is male because his sex chromosomes are XY. He has one extra copy of chromosome 21 (for a total of 47 instead of 46), resulting in Down syndrome (B). A normal female has 46 chromosomes and XX for the sex chromosomes. A female with Turner syndrome has 45 chromosomes; the sex chromosomes have just one X

The nurse who practices in a prenatal clinic understands. that a. major concern of lower socioeconomic groups is to A. Maintain group health insurance on their families B. Meet health needs as the occur C. Practice preventive health care D. Maintain an optimistic view of life

B. Meet health needs as the occur Rationale: Lower socioeconomic groups usually do not have group health insurances. Because of economic uncertainty, lower socioeconomic groups place more emphasis on meeting the needs of the present rather than on future goals. They may value health care, but cannot afford preventive health care. They may struggle for basic needs and often do not see away to improve their situation. It is difficult to maintain optimism

An abortion in which the fetus dies but is retained in the uterus is called ________ abortion. A. Inevitable B. Missed C. Incomplete D. Threatened

B. Missed Rationale: An inevitable abortion means that the cervix is dilating with the contractions. Missed abortion refers to a dead fetus being retained in the uterus. An incomplete abortion means that not all of the products of conception were expelled. With a threatened abortion the woman has cramping and bleeding but not cervical dilation

A patient is sent from the physician's office for assessment because of too little amniotic fluid. The nurse is aware that oligohydramnios can result in A. Excessive fetal urine secretion B. Newborn respiratory distress C. Central nervous system abnormality D. Gastrointestinal blockage

B. Newborn respiratory distress Rationale: Oligohydramnios may be caused by a decrease in urinee secretion. Because an abnormally small amount of amniotic fluid restricts normal lung development, the infant may have inadequate respiratory function after birth, when the placenta no longer performs respiratory function. Excessive amniotic fluid production may occur when the fetus has a central nervous system abnormality. Excessive amniotic fluid production may occur when the gastrointestinal tract prevents normal ingestion of amniotic fluid

Which statement related to changes in the breasts during pregnancy is the most accurate? A. During the early weeks of pregnancy there is decreased sensitivity B. Nipples and areolae become more pigmented C. Montgomery tubercles are no longer visible around the nipples. D. Venous congestion of the breasts is more visible in the multiparous woman

B. Nipples and areolae become more pigmented Rationale: Fullness, heightened sensitivity, tingling and heaviness of thee breasts occur in the early weeks of gestation in response to increased levels of estrogen and progesterone. Nipples and areolae become more pigmented, and the nipples become more erectile and may express colostrum. Montgomery tubercles may be seen around the nipples. These sebaceous glands may have a protective role in that they keep the nipples lubricated for breastfeeding. Venous congestion in the breasts is more obvious in primigravidas

The intrapartum woman sees no need for an admission fetal monitoring strip. If she continues to refuse, what is the first action the nurse should take? A. Consult the family of the woman B. Notify the physician C. Document the refusal in the nurses notes D. Make a referral to the hospital ethics committee

B. Notify the physician Rationale: The patient must be allowed to make choices voluntarily without undue influence or coercion from others. Patients must be allowed to make choices voluntarily without undue influence from others. The physician, especially if unaware of the patients decision, should be notified immediately. The nurse should notify the physician of the refusal of the agencys protocol and document all aspects of the explanations given by the nurse, as well as any instructions from the physician. Documentation is important, but it should not be the first action. Fetal monitoring is not usually considered an ethical problem

A woman has a history of drug use and is screened for hepatitis B during the first trimester. What is an appropriate action? A. Provide a low protein diet B. Offer the vaccine C. Discuss the recommendation to bottle feed her baby D. Practice respiratory isolation

B. Offer the vaccine Rationale: Care is supportive and includes bed rest and a high protein, low fat diet. A person who has a history of high risk behaviors should be offered the hepatitis B vaccine. The first trimester is too early to discuss feeding methods with a woman in the high risk category. Hepatitis B is transmitted through blood

Which family most likely have the most difficulty coping with an ill child? A. A single parent mother who has the support of her parents and siblings B. Parents who have just moved to an area and are living in an apartment while they look for a house C. The family of a child who has had multiple hospitalizations related to asthma and has an adequate relationship with the nursing staff D. A family in which there is a young child and four older married children who live in the area

B. Parents who have just moved to an area and are living in an apartment while they look for a house Rationale: Although only one parent is available, she has the support of her extended family, which will assist her in adjusting to the crisis. Parents in a new environment will have increased stress related to their lack of support system. They have no previous experiences in the setting from which to draw confidence. Because this family had positive experiences in the past, family members can draw from those experiences and feel confident about the setting. This family has an extensive support system that will assist the parents in adjusting to the crisis

The step of the nursing process in which the nurse determines the appropriate interventions for the identified nursing diagnosis is: A. Assessment B. Planning C. Interventions D. Evaluation

B. Planning Rationale: During the assessment phase, data are collected. The third step in the process involves planning care for problems that were identified during assessment. The intervention phase is when the plan of care is carried out. The evaluation phase is determining whether the goals have been met

After admitting a new patient to the pediatric unit, the nurse writes a plan of care. This process of determining outcomes and interventions is which stage of the nursing process? A. Assessment B. Planning C. Implementation D. Evaluation

B. Planning Rationale: During the planning stage, the nurse establishes outcomes and writes nursing interventions. Assessment stage involves collecting the patient data. Implementation stage is the active phase; it is carrying out the plan of care. Evaluation stage determines how well the plan worked.

More than 100 different ethno-cultural groups reside within the United States, and numerous traditional health beliefs are observed among these groups. Traditional beliefs related to the maintenance of health are likely to include A. Avoidance of natural events such as a solar eclipse B. Practicing silence, meditation, and prayer C. Protection of the soul by avoiding envy or jealousy D. Understanding that a hex, spell, or the evil eye may cause illness or injury

B. Practicing silence, meditation, and prayer Rationale: Illness can be prevented by avoiding natural events such as a solar eclipse along with environmental factors such as bad air. Mental and spiritual health is maintained by activities such as silence, meditation, and prayer. Many people view illness as punishment for breaking their religious code and adhere strictly to morals and religious practices to maintain health. Phenomena such as accidental provocation of envy, jealousy, or hate of a friend or acquaintance may cause illness. Agent such as hexes, spells, and the evil eye may strike a person (often a child) and causes injury, illness, or misfortune

A 15-year-old is 7 months' pregnant and comes to the clinic for her first prenatal visit. As the nurse is doing the initial assessment, she is aware that the adolescent is at risk for what complication? A. Fetal congenital defects B. Preeclampsia and anemia C. Polyhydramnios D. Hypothyroidism

B. Preeclampsia and anemia Rationale: Adolescents with poor prenatal care are at increased risk for preeclampsia and anemia during the pregnancy. Being a pregnant adolescent does not put the fetus at higher risk for defects, polyhydramnios, or hypothyroidism.

Which laboratory marker is indicative of disseminated intravascular coagulation (DIC)? A. Bleeding time of 10 minutes B. Presence of fibrin split products C. Thrombocytopenia D. Hyperfibrinogenemia

B. Presence of fibrin split products Rationale: Bleeding time in DIC is normal. Degradation of fibrin leads to the accumulation of multiple fibrin clots throughout the body's vasculature. Low platelets may occur with but are not indicative of DIC because they may result from other coagulopathies. Hyperfibrinogenemia occurs with DIC

Which one of the following nursing actions is most likely to reduce a patient's anxiety and enhance the patient's personal security as it relates to the concept of personal space needs? A. Touching the patient before and during procedures B. Providing explanations when performing tasks C. Making eye contact as much as possible D. Reducing the need for the patient to make decisions

B. Providing explanations when performing tasks Rationale: providing explanations, especially when performing tasks that require close contact, can help to avoid misunderstandings; touching the patient, making eye contact, and taking away the right to make decisions can be interpreted by patients in some cultures as invading their personal space

What data on a patient's health history places her at risk for an ectopic pregnancy? A. Use of oral contraceptives B. Recurrent pelvic infections C. Ovarian cyst 2 years ago D. Heavy menstrual flow of 4 days duration

B. Recurrent pelvic infections Rationale: Oral contraceptives do not increase the risk for ectopic pregnancies. Infection and subsequent scarring of the fallopian tubes prevents normal movement of the fertilized ovum into the uterus for implantation. Ovarian cysts do not cause scarring of the fallopian tubes. This will not cause scarring of the fallopian tubes, which is the main risk factor for ectopic pregnancies

At a previous anepartal visit, the nurse taught a pregnant woman diagnosed with a class II cardiac disorder about measures to use to lower her risk for cardiac decompensation. This woman would demonstrate need for further instruction if she tells the nurse she will use which of the following measures? A. Increase roughage in her diet B. Remain on bed rest, getting out of bed only to go to the bathroom C. Sleep 10 hours every night ad take a short nap after meals D. Will call the nurse immediately if she experiences any pain or swelling in her legs

B. Remain on bed rest, getting out of bed only to go to the bathroom Rationale: bed rest is not required for a woman with a class II designation; she will need to avoid heavy exertion and stop activities that cause fatigue and dyspnea; actions is A,C, and D are all appropriate and recommended for class II

A woman diagnosed with an ectopic pregnancy is given an intramuscular injection of methotrexate. The nurse would tell the woman which of the following? A. Methotrexate is an analgesic that will relieve the dull abdominal pain she is experiencing B. She should avoid alcohol until her primary care provider tells her the treatment is complete C. Follow up blood tests will be required every other month for 6 months after the injection of the methotrexate D. She should continue to take her prenatal vitamin and folic acid to enhance healing

B. She should avoid alcohol until her primary care provider tells her the treatment is complete Rationale: methotrexate destroys rapidly growing tissue, in this case the fetus and placenta, to avoid rupture of the tube and need for surgery; follow up with blood tests is needed for 2 to 8 weeks; alcohol and vitamins containing folic acid increase the risk for side effects with this medication or exacerbating the ectopic rupture

A woman has determined that bottle feeding is the best feeding method for her. Instructions the woman should receive regarding this feeding method should include which of the following? A. provide the infant with supplemental vitamins along with the iron fortified formula B. Sterilize water by boiling, then cool and mix with formula powder or concentrate. C. Expect a 2 week old newborn to drink approximately 30 to 60mL of formula at each feeding. D. Microwave refrigerated formula for about 2 minutes before feeding the newborn

B. Sterilize water by boiling, then cool and mix with formula powder or concentrate. Rationale: supplements are not required when using prepared formulas; a 2 week old infant should consume approximately 90 to 150mL of formula at each feeding; formula should never be heated in the microwave because it could be overheated or unevenly heated

Which of the following objectives would not be considered a priority area for Healthy People 2020? A. Improving nutritional and infant health B. Technologic advances to treat neonates C. Reducing violent or abusive behavior D. Vaccinating 90% of 2 year olds

B. Technologic advances to treat neonates

A woman who is 16 weeks pregnant asks the nurse, "Is it possible to tell by ultrasound if the baby is a boy or girl yet?" The best answer is: A. A baby's sex is determined as soon as conception occurs. B. The baby has developed enough that we can determine the sex by examining the genitals through ultrasound. C. Boys and girls look alike until approximately 20 weeks after conception, and then they begin to look differently D. It might be possible to determine your baby's sex, but the external organs look very similar right now

B. The baby has developed enough that we can determine the sex by examining the genitals through ultrasound. Rationale: This is a true statement, but the external genitalia are similar in appearance until approximately the 12th week. (A) Although gender is determined at conception, the external genitalia of males and females look similar through the 9th week. By the 12th week, the external genitalia are distinguishable as male or female. The external genitalia are similar in appearance until approximately 12 weeks, not 20 weeks. The external genitalia are different at approximately week 12

A 39-year-old is seeing the nurse-midwife for her first prenatal visit. The pregnancy was a surprise—"I thought I was going through the change of life." This is her first pregnancy, and she has no previous health problems. She does not smoke and drank one alcoholic beverage a week until she discovered she was pregnant. Which part of the woman's history shows the highest risk for the fetus developing a chromosomal abnormality such as trisomy 21 and therefore alerts the nurse-midwife to discuss doing genetic studies on the fetus? A. Chances that the woman is perimenopausal B. The fact that the woman is 39 years old C. Drinking alcohol (one drink a week) during early pregnancy D. No preplanning for the pregnancy

B. The fact that the woman is 39 years old Rationale: Maternal age greater than 35 is the highest risk factor for chromosomal abnormalities such as trisomy 21. The low alcohol intake and not planning for the pregnancy are not risk factors for this disorder. The perimenopausal state does not increase the risk of trisomy 21, unless it is related to being more than 35 years old.

The perinatal nurse is giving discharge instructions to a woman, status post suction and curettage secondary to a hydatidiform mole. The woman asks why she must take oral contraceptives for the next 12 months. The best response from the nurse is A. If you get pregnant within 1 year, the chance of a successful pregnancy is very small. Therefore, if you desire a future pregnancy, it would be better for you to use the most reliable method of contraception available B. The major risk to you after a molar pregnancy is a type of cancer that can be diagnosed only by measuring the same hormone that your body produces during pregnancy. If you were to get pregnant, it would make diagnosis of this cancer more difficult C. If you can avoid a pregnancy for the next year, the chance of developing a second moral pregnancy is rare. Therefore, to improve your chance of a successful pregnancy, it is better not to get pregnant at this time. D. Oral contraceptives are the only form of birth control that will prevent a recurrence of a molar pregnancy

B. The major risk to you after a molar pregnancy is a type of cancer that can be diagnosed only by measuring the same hormone that your body produces during pregnancy. If you were to get pregnant, it would make diagnosis of this cancer more difficult Rationale: Women should be instructed to use birth control for 1 year after treatment for a hydatidiform mole. Pregnancy raises hCG levels which increases the risk of choriocarcinoma (A). This is an accurate statement. Beta-hCG levels will be drawn for 1 year to ensure that the mole is completely gone. There is an increased chance of developing choriocarcinoma after the development of a hydatidiform mole. The goal is to achieve a zero hCG level. If the woman were to become pregnant, it may obscure the presence of the potentially carcinogenic cells. The rational for avoiding pregnancy for 1 year is to ensure that carciogenic cells are not present. Any contraceptive method except an IUD is acceptable

A pregnant woman is being discharged from the hospital after placement of a cerclage because of a history of recurrent pregnancy loss secondary to an incompetent cervix. Discharge teaching should emphasize that A. Any vaginal discharge should be reported immediately to her care provider. B. The presence of any contractions, rupture of membranes, or severe perineal pressure should be reported. C. She will need to make arrangements for care at home, because her activity level will be restricted. D. She will be scheduled for a cesarean birth

B. The presence of any contractions, rupture of membranes, or severe perineal pressure should be reported. Rationale: Vaginal bleeding needs to be reported to her primary care provider. Nursing care should stress the importance of monitoring signs and symptoms of preterm labor. Bed rest is an element of care. However, the woman may stand for periods of up to 90 minutes, which allows her the freedom to see her physician. Home uterine activity monitoring may be used to limit the womans need for visits and to safely monitor her status at home. The cerclage can be removed at 37 weeks of gestation (to prepare for a vaginal birth), or a cesarean birth can be planned

A woman who is 6 months pregnant has sought medical attention, saying she fell down the stairs. What scenario would cause an emergency department nurse to suspect that the. woman has been battered? A. The woman and her partner are having an argument that is loud and hostile. B. The woman has injuries on various parts of her body that are in different stages of healing C. Examination reveals a fractured arm and fresh bruises. Her husband asks about her pain D. She avoids making eye contact and is hesitant to answer questions

B. The woman has injuries on various parts of her body that are in different stages of healing Rationale: This is not always an indication of battering. Many times the batterer will be attentive and refuse to leave thee woman's bedside. The battered woman often has multiple injuries in various stages of healing. With battering there are injuries in various stages of healing. It is more normal for the woman fo have a flat affect

The nurse is caring for a dying boy whose religion is Islam (Muslim/Moslem). What information should the nurse use when considering his religion and impending death? A. There are no special rites related to death. B. There are specific practices to be followed. C. The family is expected to wait away from the dying person. D. Ask the family about organ donation.

B. There are specific practices to be followed. Rationale: There are special rites. The nurse should ask the family what should be done, or someone from the person's mosque can be called to assist. Death ritual prescribes the handling of corpse by only family and friends. Burial occurs as soon as possible. Autopsy and organ donation will be opposed by the family.

Which statement is true of multifactorial disorders? A. They may not be evident until later in lie B. They are usually present and detectable at birth C. The disorders are characterized by multiple defects D. Secondary defects are rarely associated with multifactorial disease

B. They are usually present and detectable at birth Rationale: They are usually detectable at birth. Multifactorial disorders result from an interaction between a persons genetic susceptibility and environmental conditions that favor development of the defect. They are characteristically present and detectable at birth. They are usually single isolated defects, although the primary defect may cause secondary defects. Secondary defects can occur with multifactorial disorders

Which maternal condition always necessitates delivery by cesarean section? A. Partial abruptio placentae B. Total placenta previa C. Ectopic pregnancy D. Eclampsia

B. Total placenta previa Rationale: If the mother has stable vital signs and the fetus is alive, a vaginal delivery can be attempted. If the fetus has died, a vaginal delivery is performed. In total placenta previa, the placenta completely covers the cervical os. The fetus would die if a vaginal delivery occurred. The most common ectopic pregnancy is a tubal pregnancy, which is usually detected and treated in the first trimester. Labor can be safely induced if the eclampsia is under control

In terms of the incidence and classification of diabetes, maternity nurses should know that: A. Type 1 diabetes is most common. B. Type 2 diabetes often goes undiagnosed. C. There is only one type of gestational diabetes. D. Type 1 diabetes may become type 2 during pregnancy.

B. Type 2 diabetes often goes undiagnosed. Rationale: Type 2, sometimes called adult onset diabetes, is the most common. Type 2 often goes undiagnosed, because hyperglycemia develops gradually and often is not severe. There are 2 subgroups of gestational diabetes. Type GDM A1 is diet controlled whereas Type 2 GDM A2 is controlled by insulin and diet. People do not go back and forth between type 1 and type 2 diabetes.

Mimicry refers to observing and copying the behaviors of other mothers. An example might be A. Babysitting for a neighbors children B. Wearing maternity clothes before they are needed C. Daydreaming about the newborn D. Imagining oneself as a good mother

B. Wearing maternity clothes before they are needed Rationale: Babysitting other children is a form of role playing where the woman practices the expected role of motherhood. Wearing maternity clothes before they are needed helps the expectant mother feel what its like to be obviously pregnant. Daydreaming is a type of fantasy where the woman tries a variety of behaviors in preparation for motherhood. Imagining herself as a good mother is the womans effort to look for a good role fit. She observes behavior of other mothers and compares them with her own expectations

A woman is 16 weeks pregnant with her first baby. She asks how long it will be before she feels the baby move. The best answer is A. You should have felt the baby move by now B. Within the next month, you should start feeling fluttering sensations C. The baby is moving, but you can't feel it yet D. Some babies are quiet, and you don't. feel them move

B. Within the next month, you should start feeling fluttering sensations Rationale: Because this is her first pregnancy, movement is felt toward the later part of the 17 to 20 weeks. This statement may be alarming to the woman. Maternal perception of fetal movement usually begins 17 to 20 weeks after conception. This is a true statement. The fetus movements are not strong enough to be felt until 17 to 20 weeks; however, this statement does not answer the concern of the woman. Fetal movement should be felt by 17 to 20 weeks. If movement is not felt by the end of that time, further assessment will be necessary

A patient with pregnancy induced hypertension is admitted complaining of pounding headache, visual changes, and epigastric pain. Nursing care is based on the knowledge that these signs indicate A. Anxiety due to hospitalization B. Worsening disease and impending convulsion C. Effects of magnesium sulfate D. Gastrointestinal upset

B. Worsening disease and impending convulsion Rationale: These are danger signs and should be treated. Headache and visual disturbances are due to increased cerebral edema. Epigastric pain indicates distention of the hepatic capsules and often warns that a convulsion is imminent. She has not been started on magnesium as a treatment yet. Also, these are not expected effects of the medication. These are danger signs showing increased cerebral edema and impending convulsion

A patient who is in week 28 of gestation is concerned about her weight gain of 17lb. The nurses best response is A. You should try to decrease your amount of weight gain for the next 12 weeks B. You have gained an appropriate amount for the number of weeks of your pregnancy C. You should not gain any more weight until you reach the third trimester. D. You have not gained enough weight for the number of weeks of your pregnancy

B. You have gained an appropriate amount for the number of weeks of your pregnancy Rationale: The woman has gained the appropriate amount of weight. It would be inappropriate to have her decrease her weight gain. A woman in her 28th week of gestation should have gained between 17 and 20 lb. The normal pattern of weight gain is 2 to 3lb total in the first trimester (by 13 weeks) and 1 lb per week after that. Weight gain needs to be consistent during the last part of pregnancy and should not be suppressed. She has gained an appropriate amount of weight and should not increase thee weight gain

The primary reason for evaluating alpha-fetoprotein (AFP) levels in maternal serum is to determine if the fetus has A. hemophilia B. a neural tube defect C. sickle cell anemia D. a normal lecithin/sphingomyelin (L/S) ration

B. a neural tube defect Rationale: Hemophillia is a genetic defect and is best detected with chromosomal studies such as chorionic villus sampling or amniocentesis. An open neural tube allows a high level of AFP to seep into the amniotic fluid and enter the maternal serum. Sickle cell is a genetic defect and is best detected with chromosomal studies such as chorionic villus sampling or amniocentesis. L/S ratios are determined with an amniocentesis and is usually done in the third trimester

A woman with preeclampsia is being treated with bed rest and intravenous magnesium sulfate. The drug classification of this medication is A. tocolytic. B. anticonvulsant. C. antihypertensive. D. diuretic.

B. anticonvulsant. Rationale: Magnesium sulfate controls seizure activity by blocking neuromuscular transmission and depressing the central nervous system. The drug does slow down the frequency and intensity of uterine contractions, but it is not used for that purpose with preeclampsia. Magnesium relaxes smooth muscle and reduces vasoconstriction, possibly resulting in modest blood pressure reduction. Diuresis is a secondary effect due to increased circulation to the kidneys.

The purpose of initiating contractions in a contraction stress test is to A. determine the degree of fetal activity. B. apply a stressful stimulus to the fetus. C. identify fetal acceleration patterns. D. increase placental blood flow.

B. apply a stressful stimulus to the fetus. Rationale: The contraction stress test involves recording the response of the fetal heart rate to stress induced by uterine contractions.

To prevent GI upset, patients should be instructed to take iron supplements A. on a full stomach B. at bedtime C. after eating a meal D. with milk

B. at bedtime Rationale: Iron supplements are best absorbed if they are taken when the stomach is empty. Taking iron supplements at bedtime may reduce GI upset. Iron supplements are best absorbed if they are taken when the stomach is empty. Iron can be taken at bedtime if abdominal discomfort occurs when it is take between meals. Bran, tea, coffee, milk, and eggs may reduce absorption. Iron can be taken at bedtime if abdominal discomfort occurs when it is taken between meals

Preconception counseling is critical to the outcome of diabetic pregnancies because poor glycemic control before and during early pregnancy is associated with A. frequent episodes of maternal hypoglycemia B. congenital anomalies in the fetus C. polyhydramnios D. Hyperemesis gravidum

B. congenital anomalies in the fetus Rationale: Frequent episodes of maternal hypoglycemia may occur during the first trimester (not before conception) as a result of hormone changes and the effects on insulin production and usage. Preconception counseling is particularly important because strict metabolic control before conception and in the early weeks of gestation is instrumental in decreasing the risks of congenital anomalies. Hydroamnios occurs about 10 times more often in diabetic pregnancies than in nondiabetic pregnancies. Typically, it is seen in the third trimester of pregnancy. Hyperemesis gravidarum may exacerbate hypoglycemic events as the decreased food intake by the mother and glucose transfer to the fetus contribute to hypoglycemia.

Early pregnancy classes offered in the first and second trimesters cover A. phases and stages of labor B. coping with common discomforts of pregnancy C. Methods of pain relief D. Predelivery and postdelivery care of the patient having a cesarean delivery

B. coping with common discomforts of pregnancy Rationale: Phases and stages of labor are taught in childbirth preparation classes. Early pregnancy classes focus on the first two trimesters and cover information on adapting to pregnancy, dealing with early discomforts, and understanding what to expect in the months ahead. Patin control is part of childbirth preparation classes. This is taught in cesarean birth preparation classes

The results of a nonstress test shows three fetal heart rate accelerations with fetal movement that peak at 15 beats per minute above baseline and last 15 seconds. The nurse's next action should be to A. apply acoustic stimulation for 1 second for further testing. B. do nothing. C. continue to test for 40 additional minutes. D. prepare the woman for a contraction stress test.

B. do nothing. Rationale: A reactive sign is at least two fetal heart rate accelerations with or without fetal movement, occurring within a 20-minute period, peaking at least 15 beats per minute above the baseline, and lasting 15 seconds. This is reassuring, and no further testing is necessary. Acoustic stimulation can be used if the fetus is not active. The results given in the base of the question are reassuring, and no other testing is necessary.

While providing care in an obstetric setting, the nurse should understand that postpartum care of the woman with cardiac disease A. is the same as that for any pregnant woman B. includes rest, stool softeners, and monitoring of the effect of activity C. Includes ambulating frequently alternating with active range of motion D. includes limiting visits with the infant to once per day

B. includes rest, stool softeners, and monitoring of the effect of activity Rationale: Care of the woman with cardiac disease in the postpartum period is tailored to thee womans functional capacity. Bed rest may be ordered, with or without bathroom privileges. Bowel movements without stress or strain for the woman are promoted with stool softeners, diet and fluid. The woman will be on bed rest to conserve energy and reduce the strain on the heart. Although the woman may need help caring for the infant, breastfeeding and infant visits are not contraindicated

A woman is admitted to the labor unit in active labor. She informs the nurse that she has had no prenatal care. She has been taking Fioricet (acetaminophen, butalbital, caffeine) for pain throughout the pregnancy. The nurse is aware that this drug is classified as X and therefore A. will not harm the fetus B. is a teratogen and the fetus may be harmed. C. information is not sufficient to determine if the fetus will be harmed. D. will only affect one fetal organ system.

B. is a teratogen and the fetus may be harmed. Rationale: A class X medication means that the drug is well established as being harmful to a fetus and should not be used during pregnancy. Class A drugs have no demonstrated fetal risk. Class B and C drugs suggest possible harm, but no studies have been done on humans to prove safety. Teratogens typically cause more than one defect.

The nurse has been assigned to care for a patient during the night shift. The patient's medication to prevent seizures was due at 6:00 am. At that time, the nurse was involved with another patient and did not administer the medication. At 10am, the patient ambulated to the bathroom, had a seizure, fell, and later developed brain damage as a result of the fall. The nurse can be sued for A. abandonment B. malpractice C. a civil tort D. nothing, the nurse is immune because she was assisting another patient

B. malpractice Rationale: Malpractice has four elements that must be proved: a duty (the nurse was assigned to care for the patient), breach of duty (the nurse did not render care by neglecting the medication), damage (the patient suffered brain damage), proximate cause (brain damage was due to the fall during the seizure). Abandonment would have occurred if the nurse had not provided any care for the patient or if the nurse had walked away from his or her job. A civil tort is a civil wrong or injury. The nurse is not immune if she neglects one patient for another.

A 27 year old woman newly diagnosed with diabetes is admitted to an agency to regulate her medication and receive patient teaching on diabetes. She is assigned a case manager on her first day. To best explain this role to the woman, the nurse states that a case manager will A. decide which patient teaching is necessary for the woman B. manage and collaborate the woman's care to ensure optimal outcomes C. be responsible for reviewing the woman's chart for errors D. decide which method of treatment is most cost effective for the agency

B. manage and collaborate the woman's care to ensure optimal outcomes Rationale: A case manager will focus on both quality and cost outcomes. He or she will coordinate services needed and manage the care collaboratively to ensure optimal outcomes. The case manager will coordinate the patient teaching, not decide which patient teaching is necessary for the woman. The quality assurance team is responsible for reviewing the woman's chart for errors. The case manager is concerned about cost effectiveness, but the main focus is to ensure quality outcomes

A woman is concerned that she has developed numerous nosebleeds during this pregnancy. She feels this is a sign of leukemia and wants to be screened. The nurse's response to the woman should be based on the fact that A. leukemia is a major concern during pregnancy. B. nosebleeds are a common occurrence during pregnancy. C. nosebleeds are rare in pregnancy; therefore further assessment is necessary. D. platelet count decreases significantly during pregnancy.

B. nosebleeds are a common occurrence during pregnancy. Rationale: With the higher levels of estrogen causing increased vascularity in the upper respiratory tract, epistaxis is a common occurrence. Leukemia rates do not increase during pregnancy. There is a slight decrease in the platelet count but within normal range.

The nurse is reviewing the lab reports on a 17-year-old new patient. The gonadotropin-releasing hormone levels are extremely low. The nurse can anticipate that the patient will A. look older than her years. B. not have primary or secondary sexual characteristics. C. have primary, but not secondary, sexual characteristics. D. have adequate levels of follicle-stimulating hormone (FSH) and luteinizing hormone (LH).

B. not have primary or secondary sexual characteristics. Rationale: Gonadotropin-releasing hormone begins increasing as a boy enters puberty, stimulating secretion of LH and FSH from the anterior pituitary. LH and FSH then stimulate secretion of testosterone. With low levels of GNRH, she will look younger, because she has not started to develop primary or secondary sexual characteristics. The level of GNRH increases slowly until it is adequate to stimulate the anterior pituitary to increase its production of follicle-stimulating hormone (FSH) and luteinizing hormone (LH).

The maternity nurse understands that vascular volume increases 40 to 60% during pregnancy to A. compensate for decreased renal plasma flow B. provide adequate perfusion of the placenta C. eliminate metabolic wastes of the mother D. prevent maternal and fetal dehydration

B. provide adequate perfusion of the placenta Rationale: Renal plasma flow increases during pregnancy. The primary function of increased vascular volume is to transport oxygen and nutrients to the fetus via the placenta. Assisting with pulling metabolic wastes from the fetus for maternal excretion is one purpose of the increased vascular volume. This is not the primary reason for the increase in volume.

A woman who is 6 weeks' pregnant is scheduled for an ultrasound. She asks the nurse what can be seen at this stage of the pregnancy. The nurse would be correct if she responded A. the sex of the baby. B. the baby's heartbeat. C. characteristics of the baby's face. D. fetal presentation.

B. the baby's heartbeat. Rationale: The heartbeat is visible when the embryo is 5 mm in length. Fetal sex and details about the baby cannot be seen until later in the pregnancy. Characteristics of the baby's face can be seen on a three-dimensional sonogram later in the pregnancy. Fetal presentation is determined during the second and third trimesters.

A woman told the nurse the doctor had written down that she had experienced quickening. When explaining this to the woman, the nurse uses the knowledge that quickening is A. the cheese-like secretions that cover the fetus. B. the first sensation of fetal movement. C. the production of a surface-active lipid necessary for the neonate to breathe. D. an excessive amount of amniotic fluid.

B. the first sensation of fetal movement. Rationale: The first sensation of fetal movement detected by the woman is called quickening. The cheese-like covering over the fetus is called vernix caseosa. The surface-active lipid that is produced by a fetus is called surfactant. An excessive amount of amniotic fluid is called hydramnios.

The older mother is at greater risk for postpartum hemorrhage because of A. uterine atony. B. uterine myomas. C. anemia. D. infections.

B. uterine myomas. Rationale: Women older than 35 years old have a greater frequency of uterine myomas (fibroids), which predisposes her to postpartum hemorrhage. Uterine atony is a major cause of early postpartum hemorrhage. However, the risk for this type of hemorrhage is the same for all ages. The older woman is not at greater risk for anemia or infections.

Members of the Jehovah's Witness faith are opposed to certain aspects of health care: (Select all that apply.) A. Birth control B. Blood transfusions C. Infant baptism D. Autopsy

B.C. Blood transfusions; infant baptism Rationale: Blood transfusions are not allowed. May accept alternatives to transfusions, such as use of non-blood plasma expanders, careful surgical technique to minimize blood loss, and use of autologous transfusions. Jehovah's Witnesses are prepared to die rather than break God's law. Infant baptism is not performed. Adult baptism is done by immersion. Use of birth control is a personal decision. Autopsy will be decided by persons involved.

The nursing care management of a newborn whose mother is HIV positive would most likely include which of the following? (Select all that apply) A. Isolating the newborn in a special nursery B. Implementing standard precautions immediately after birth C. Telling the mother that she should not breastfeed D. Wearing gloves for routine care measures such as feeding E. Initiating treatment with antiviral medication(s) as soon as the newborn is confirmed to be HIV positive F. Preparing to administer antibiotics to prevent the newborn from developing opportunistic infections

B.C. Implementing standard precautions immediately after birth; Telling the mother that she should not breastfeed Rationale: risk of transmission of HIV is greater than the benefits of breastfeeding; isolation is not required, nor are gloves, for routine care measures; the nurse should be using standard precautions as would be used with all patient; antiviral treatment begins after birth before HIV status is known; opportunistic pulmonary infections are a concern when the infant is diagnosed with HIV infection

Mechanical forces that interfere with normal prenatal development include oligohydramnios and fibrous amniotic bands, A patient at 34 weeks of gestation has reported to the OB triage unit for assessment of oligohydramnios. The nurse assigned to care for this patient is aware that prolonged oligohydramnios may result in (select all that apply) A. Intrauterine limb amputations B. Clubfoot C. Delayed lung development D. Other fetal abnormalities E. Fetal deformations

B.C.D. Club foot; delayed lung development; other fetal abnormalities. Rationale: Oligohydramnios, an abnormally small volume of amniotic fluid, reduces the cushion surrounding the fetus and may result in deformations such as clubfoot. Prolonged oligohydramnios interferes with fetal lung development because it does not allow normal development of the alveoli. Oligohydramnios may not be the primary fetal problem but rather may be related to other fetal anomalies. Fibrous amniotic bands may result from tears in the inner sac of fetal deformation or intrauterine limb amputations. Fibrous bands are usually sporadic and unlikely to recur. Because these bands can cause multiple defects, they may be confused with birth defects from other causes such as chromosome or single gene abnormalities

A woman who is 8 weeks' pregnant calls the physician's office complaining of nausea and vomiting in the mornings. Which suggestions by the nurse would be helpful to decrease the nausea and vomiting? (Select all that apply.) A. Increase the intake of fats in her diet. B. Drink fluids between meals instead of with meals. C. Eat dry crackers before getting up in the morning. D. Eat some cheese before bedtime.

B.C.D. Drink fluids between meals instead of with meals; Eat dry crackers before getting up in the morning; Eat some cheese before bedtime. Rationale: Drinking fluids between meals, eating dry carbohydrates before getting up in the morning, and eating a protein snack before bed may help to decrease nausea in the morning. High-fat foods will increase nausea and should be avoided.

The consequences technique will assist children to learn the direct result of their behavior. This technique can be used with children from toddler age to adolescence. If children learn to understand consequences, they are less likely to repeat the offending behavior. Consequences fall into which categories? (Select all that apply) A. Corporeal B. Natural C. Logical D. Unrelated E. Behavioral

B.C.D. Natural; Logical; Unrelated Rationale: Natural consequences are those that occur spontaneously. For example, a child leaves a toy outside and it is lost. Logical consequences are those that are directly related to misbehavior. If two children are fighting over a toy, the toy is removed and neither child has it. Unrelated consequences are purposely imposed; for example, the child is late for dinner so her or she is not allowed to watch television. Corporeal punishment is not part of this behavioral approach and usually takes the approach of spanking the child. Corporal punishment is highly controversial and is strongly discouraged by the American Academy of Pediatrics. Behavior modification is another disciplinary technique that rewards positive behavior and ignore negative behavior

Approximately 82% of teen pregnancies are unintended. Seventy percent of teens have had sex by their 19th birthday. Factors that contribute to an increased risk for teen pregnancy include (select all that apply) A. High self esteem B. Peer pressure C. Limited access to contraception D. Planning sexual activity E. Lack of role models

B.C.E. Peer pressure; limited access to contraception; lack of role models Rationale: Peer pressure to begin sexual activity is contributing factor towards teenage pregnancy. Limited access to contraceptive devices and lack of accurate information about how to use these devices are also factors. Lack of appropriate role models, desire to alleviate or escape the present situation at home, along with feelings of invincibility, also contribute to teenage pregnancy. Low self esteem and the consequent inability to set limits on sexual activity places the adolescent at risk for teen pregnancy. Ambivalence towards sexuality, and not planning intercourse, are more likely to result in teen pregnancy

Environmental substances known or thought to harm the fetus include A. penicillin B. cocaine C. toxoplasmosis infection D. flu virus E. Zika infection F. Tetracycline

B.C.E.F. cocaine; toxoplasmosis infection; zika infection; tetracycline Rationale: Cocaine use, infections with toxoplasmosis and Zika, and tetracycline use during pregnancy are known or thought to harm the fetus. Penicillin and the flu virus are not.

Which of the following health services represents the primary level of preventive care? (Select all that apply) A. Breast self examination and testicular self examination education programs B. Providing flu immunizations in pharmacies C. A safer sex informational pamphlet provided to adolescents during a health education class D. Blood pressure and cholesterol screening at a health fair E. Instituting a wheelchair, cane, and walked exchange program for persons whose health insurance does not cover these items F. A car- seat fitting fair at local auto dealerships

B.C.F. Providing flu immunizations in pharmacies; A safer sex informational pamphlet provided to adolescents during a health education class; A car- seat fitting fair at local auto dealerships Rationale: A and D are secondary level of prevention and E is tertiary level of prevention

The nurse caring for women in labor should be aware of signs characterizing normal (reassuring) and abnormal (nonreassuring) FHR patterns. Which of the following would be nonreassuring signs? (Select all that apply) A. Moderate baseline variability B. Average baseline FHR of 100beats/min C. Acceleration with fetal movement D. Late deceleration patterns approximately every 3 contractions E. FHR of 155 beats/min between contractions F. Early deceleration patterns

B.D. Average baseline FHR of 100beats/min; Late deceleration patterns approximately every 3 contractions Rationale: The baseline rate should be 110 to 160 beats/min; late deceleration pattern of any magnitude is nonreassuring (abnormal), especially if it is repetitive

Preterm infants are at an increased risk for developing respiratory distress. The nurse should assess for signs that would indicate the newborn is having difficulty breathing. Which of the following are signs of respiratory distress? (Select all that apply) A. Use of abdominal muscles to breathe B. Tachypnea C. Periodic breathing pattern D. Suprasternal retraction E. Nasal flaring F. Acrocyanosis

B.D.E Tachypnea; Suprasternal retraction; Nasal flaring Rationale: retractions, nasal flaring reflect increased effort and work to breathe. A.C.F are all expected findings consistent with efficient respiratory effort in the preterm newborn

Optimal patient care relies on the nurses expertise and clinical judgement; however, critical thinking (a component of nursing judgement) underlies the nursing process. The nurse who uses critical thinking understands that the steps of critical thinking include (select all that apply) A. Therapeutic communication B. Examining biases C. Setting priorities D. Managing data E. Evaluating other factors

B.D.E. Examining biases;Managing data; Evaluating other factors Rationale: The 5 steps of critical thinking include: recognizing assumptions, examining biases, analyzing the need for closure, managing data, and evaluating other factors. Therapeutic communication is a skill that all nurses must have to carry out the many roles expected within the profession; however, it is not one of the steps of critical thinking. Setting priorities is part of the planning phases of the nursing process.

The nurse is preparing cough medication for several patients of various faiths and realizes that it contains alcohol. For which religious groups is alcohol forbidden, requiring the nurse to check if the patient can consume the medication? (Select all that apply.) A. Jewish B. Islam C. Catholic D. Christian Science E. Jehovah's Witness

B.D.E. Islam; Christian Science; Jehovah's Witnesses Rationale: The nurse would need to check with the patient prior to administration. Patients of the Islamic, Christian Science, and Jehovah's Witness religions do not use alcohol. *Incorrect regarding JW's- alcohol in moderation; not total abstinence*

Eating disorders include anorexia nervosa and bulimia. Many women with anorexia have amenorrhea and do not become pregnant where as women with bulimia or subclinical anorexia may become pregnant. These condition(s) are associated with (Select all that apply) A. Food cravings B. Low birth weight C. food aversions D. Electrolyte imbalance E. Small for gestational infants

B.D.E. Low birth weight; electrolyte imbalance; small for gestational infants Rationale: These conditions are associated with electrolyte imbalance, low birth weight, and small for gestational age infants. All women should be asked about eating disorders and nurses should watch for behaviors that may indicate disordered eating. Some women eat normally during pregnancy for the sake of the fetus, but others continue their previous dysfunctional eating patterns during pregnancy or in the early postpartum period. Food cravings and aversions are normal for most women during pregnancy. Women may have a strong preference or strong dislike for certain foods. They're generally not harmful, and some, like aversion to alcohol, may be beneficial

A woman has been diagnosed with mild preeclampsia and will be treated at home. The nurse, in teaching this woman about the treatment regimen for mild preeclampsia, would tell her to which of the following? (Select all that apply) A. Check her respirations before and after taking her oral dose of magnesium sulfate B. Place a dipstick into a clean catch sample of her urine to test for protein C. Reduce her fluid intake to four to five 8 ounce glasses each day D. Do gentle exercises such as hand and feet circles and gently tensing and relaxing arm and leg muscles E. Avoid excessively salty foods F. Maintain strict bed rest in a quiet dimly lighted room with minimal stimuli

B.D.E. Place a dipstick into a clean catch sample of her urine to test for protein; Do gentle exercises such as hand and feet circles and gently tensing and relaxing arm and leg muscles; Avoid excessively salty foods Rationale: Magnesium sulfate is administered intravenously in the hospital with severe preelclampsia; a clean catch, midstream urine specimen should be used to assess urine for protein using a dipstick; fluid intake should be 6 to 8 (8oz) glasses a day along with roughage to prevent constipation; gentle exercise improves circulation and helps preserve muscle tone and a sense of well being; modified bed rest with diversional activities is recommended for mild preeclampsia

A 52 year old woman asks the nurse practitioner about how often she should be assessed for the common health problems women of her age could experience. The nurse would recommend which of the following screening measures (Select all that apply) A. an endometrial biopsy every 2-3 years B. A fecal occult blood test every year C. A mammogram every other year D. Clinical breast examination every year E. Bone mineral density testing every year beginning when she is 55 F. Vision examination every 2-4 years

B.D.F. A fecal occult blood test every year; Clinical breast examination every year; Vision examination every 2-4 years Rationale: endometrial biopsies are not recommended on a routine basis for most women, but women at risk for endometrial cancer should have one done at menopause; mammograms should be done annually for women over 50 years of age and bone mineral density testing beginning at age 65

A woman's preeclampsia has advanced to the severe stage. She is admitted to the hospital and her primary health care provider has ordered an infusion of magnesium sulfate be started. In fulfilling this order the nurse would implement which of the following? (Select all that apply) A. Prepare a loading dose of 2g of magnesium sulfate in 200mL of 5% glucose in water to be given over 15 minutes B. Prepare the maintenance solution by mixing 40g of magnesium sulfate in 1000mL of lactated Ringer's solution C. Monitor maternal vital signs, fetal heart rate (FHR) patterns, and uterine contractions every 2 hours D. Expect the maintenance dose to be approximately 2g/hr E. Report a respiratory rate of 14 breaths or less per minute to the primary health care provider immediately F. Recognize the urinary output should be at least 25-30mL per hour

B.D.F. Prepare the maintenance solution by mixing 40g of magnesium sulfate in 1000mL of lactated Ringer's solution; Expect the maintenance dose to be approximately 2g/hr; Recognize the urinary output should be at least 25-30mL per hour Rationale: the loading dose should be an IV of 4 to 6g diluted in 100mL of intravenous fluid; maternal assessment should occur every 15 to 30 minutes and FHR and UC continuously; respirations should be less than 12

A women's health nurse practitioner is preparing and education presentation on the topic of intimate partner violence (IPV) to a group of women who come to the clinic where she practices. As part of the presentation, she plans to dispel commonly held myths regarding IPV. Which of the following statements represent the facts related to IPV? (Select all that apply) A. Battering almost always affects women who are poor B. Of women who experience battering, 25% are battered by an intimate partner C. Women usually are safe from battering while they are pregnant, although the battering will resume after they have the baby D. Women tend to leave the relationship if the battering is bad E. Counseling may be successful in helping the batterer stop his behavior F. Women do not like to be beaten and will often do anything to avoid a confrontation

B.E.F. Of women who experience battering, 25% are battered by an intimate partner; Counseling may be successful in helping the batterer stop his behavior; Women do not like to be beaten and will often do anything to avoid a confrontation Rationale: IPV can occur in any family; battering frequently begins or escalates during pregnancy; women may stay even if the battering is bad because of fear and financial dependence, and shelters often have long waiting lists.

When assessing a pregnant woman at 28 weeks of gestation who is diagnosed with mitral valve stenosis, the nurse must be alert for signs indicating cardiac decompensation. Signs of cardiac decompensation would include which of the following? (Select all that apply) A. Dry, hacking cough B. Increasing fatigue C. Wheezing with inspiration and expiration D. Bradycardia E. Progressive generalized edema F. Orthopnea with increasing dyspnea

B.E.F. increasing fatigue; progressive generalized edema; orthopnea with increasing dyspnea Rationale: Other signs of cardiac compensation include moist, productive, frequent cough and crackles at bases of lungs; pulse becomes rapid, weak, irregular.

A couple has been counseled for genetic anomalies. They ask you, "What is karyotyping?" Your best response is: A. "Karyotyping will reveal if the baby's lungs are mature." B. "Karyotyping will reveal if your baby will develop normally." C. "Karyotyping will provide information about the gender of the baby and the number and structure of the chromosomes." D. "Karyotyping will detect any physical deformities the baby has."

C. "Karyotyping will provide information about the gender of the baby and the number and structure of the chromosomes." Rationale: The L/S ratio, not karyotyping, reveals lung maturity. Although karyotyping can detect genetic anomalies, the range of normal is non-descriptive. Karyotyping provides genetic information, such as gender and chromosomal structure. Karyotyping is completed by photographing or using computer imaging to arrange chromosomes in pairs for the largest to smallest. The karyotype can then be analyzed. Although karyotyping can detect genetic anomalies, not all such anomalies display obvious physical deformities. The term deformities is a non-descriptive word. Furthermore, physical anomalies may be present that are not detected by genetic studies (e.g. cardiac malformation)

A woman who is 7 months' pregnant states, "I'm worried that something will happen to my baby." The nurse's best response is A. "There is nothing to worry about." B. "The doctor is taking good care of you and your baby." C. "Tell me about your concerns." D. "Your baby is doing fine."

C. "Tell me about your concerns." Rationale: Encouraging the woman to discuss her feelings is the best approach. The nurse should not disregard or belittle the woman's feelings.

Sally comes in for her first prenatal examination. This is her first child. She asks you (the nurse), "How does my baby get air inside my uterus?" The correct response is: A. "The baby's lungs work in utero to exchange oxygen and carbon dioxide." B. "The baby absorbs oxygen from your blood system." C. "The placenta provides oxygen to the baby and excretes carbon dioxide into your bloodstream." D. "The placenta delivers oxygen-rich blood through the umbilical artery to the baby's abdomen."

C. "The placenta provides oxygen to the baby and excretes carbon dioxide into your bloodstream." Rationale: The fetal lungs do not function for respiratory gas exchange in utero. The baby does not simply absorb oxygen from a womans blood system. Blood and gas transport occur through the placenta. The placenta functions by supplying oxygen and excreting carbon dioxide to the maternal bloodstream. The placenta delivers oxygen-rich blood through the umbilical vein, not. artery

The nurse is explaining how to assess edema to the nursing students working on the antepartum unit. Which score indicates edema of lower extremities, face, hands, and sacral area? A. +1 edema B. +2 edema C. +3 edema D. +4 edema

C. +3 edema Rationale: Edema is classified as +1 indicates minimal edema of the lower extremities. Marked edema of the lower extremities is termed +2 edema. Edema of the extremities, face, and sacral area is classified as +3 edema. Generalized massive edema (+4) includes accumulation of fluid in the peritoneal cavity.

While working with the pregnant woman in her first trimester, the nurse is aware that chorionic villus sampling can be performed during pregnancy as early as _____ weeks A. 4 B. 8 C. 10 D. 12

C. 10 Rationale: It is too early to perform this test at 4-8 weeks. Fetal villus tissue can be obtained as early as 10 weeks of gestation and analyzed directly for chromosomal or genetic abnormalities. The test should be performed at 13 weeks, but it can be done as early as 10 weeks

During her pregnancy, a woman with pregestational diabetes has been monitoring her blood glucose level several times a day. Which of the following levels would require further assessment? A. 85mg/dL- before breakfast B. 90mg/dL- before lunch C. 135mg/dL- 2 hours after supper D. 126mg/dL- 1 hour after breakfast

C. 135mg/dL- 2 hours after supper Rationale: a 2 hour postprandial blood glucose should be less than 120mg/dL; choices A, B, and D all fall within the expected normal ranges

Of adolescents who become pregnant, what percentage have had previous births? A. 10% B. 15% C. 19% D. 35%

C. 19% Rationale: 19% of pregnant adolescents have had one or more previous births

If a patient's normal prepregnancy diet contains 45g of protein daily, how many more grams of protein should she consume per day during pregnancy? A. 5 B. 10 C. 26 D. 30

C. 26 Rationale: 5g will not be enough to meet her protein needs during pregnancy. 10g will not be enough extra protein to meet her needs during pregnancy. The recommended intake of protein for the pregnant woman is 71g. 30g is more than is necessary and will add extra calories.

At approximately ________ weeks of gestation, lecithin is forming on the alveolar surfaces, the eyelids open, and the fetus measures approximately 27cm crown to rump and weighs approximately 1110g A. 20 B. 24 C. 28 D. 30

C. 28 Rationale: These milestones would not be completed by 20-24 weeks of gestation. These are all milestones that occur at 28 weeks. These specific milestones will be reached as early as 28 weeks, not 30 weeks of gestation

Elective abortion is considered an ethical issue because: A. Abortion law is unclear about a womans constitutional rights B. The Supreme Court ruled that life begins at conception C. A conflict exists between the rights of the woman and the rights of the fetus D. It requires third party consent

C. A conflict exists between the rights of the woman and the rights of the fetus Rationale: Abortion laws are unclear concerning a womans constitutional rights. The Supreme Court has not ruled on when life begins. Elective abortion is considered an ethical dilemma because two opposing courses of action are available. The belief that induced abortion is a private choice is in conflict with the belief that elective pregnancy termination is taking a life. Abortion does not require third party consent

Which 16 year old is most likely to experience secondary amenorrhea? A. A girl who is 5ft 2in, 130lb B. A girl who is 5ft 9in, 150lb C. A girl who is 5ft, 7in, 96lb D. A girl who is 5ft 4in, 120lb

C. A girl who is 5ft, 7in, 96lb Rationale: This girls (5'2, 130lb) body mass index (BMI) is sufficient to assist with sex hormone production. A low BMI (or body fat) is a risk factor for secondary amenorrhea. This girls (5'9, 150lb) BMI is sufficient to assist with sex hormone production. Because of her height and low body weight, a female who is 5'7 and 96lb is at risk of developing secondary amenorrhea, which occurs in women who are thin and have a low percentage of body fat. Fat is necessary to make sex hormones that stimulate ovulation and menstruation. This girls (5'4, 120lb) body fat is sufficient to assist with sex hormone production. Low BMIs are a risk factor for secondary amenorrhea

Which situation reflects a potential ethical dilemma for the nurse? A. A nurse administers analgesics to ta patient with cancer as often as the physicians order allows B. A neonatal nurse provides nourishment and care to a newborn who has a defect that is incompatible with life C. A labor nurse, whose religion opposes abortion, is asked to assist with an elective abortion D. A postpartum nurse provides information about adoption to a new mother who feels she cannot adequately care for her infant

C. A labor nurse, whose religion opposes abortion, is asked to assist with an elective abortion Rationale: There is no element of conflict for the nurse; therefore a dilemma does not exist (A/B/D). A dilemma exists in this situation because the nurse is being asked to assist with a procedure that she or he believes is morally wrong. The other situations do not contain elements of conflict for the nurse

A woman taking magnesium sulfate has respiratory rate of 10 breaths/min. In addition to discontinuing the medication, the nurse should A. vigorously stimulate the woman B. Instruct her to take deep breaths C. Administer calcium gluconate D. Increase her IV fluids

C. Administer calcium gluconate Rationale: Stimulation will not increase the respirations. This will not be successful in reversing the effects of the magnesium sulfate. Calcium gluconate reverses the effects of magnesium sulfate. Increasing her IV fluids will not reverse the effects of the medication

A patient states, My breasts are so small, I don't think I will be able to breastfeed. The nurse's best response is A. It may be difficult, but you should try anyway B. You can always supplement with formula C. All women have approximately the same amount of glandular tissue to secrete milk D. The ability to produce breast milk depends on increased levels of estrogen and progesterone

C. All women have approximately the same amount of glandular tissue to secrete milk Rationale: The size of the breasts does not ensure success or failure in breastfeeding. Supplementation decreases the production of breast milk by decreasing stimulation. Stimulation of the breast, not the size of the breast, are responsible for milk production. Increased levels of estrogen decrease the production of milk by affecting prolactin

Alternative and complementary therapaies: A. Replace conventional Western modalities of treatment B. Are used by only a small number of American adults C. Allow for more patient autonomy D. Focus primarily on the disease an individual is experiencing

C. Allow for more patient autonomy Rationale: Alternative and complementary therapies are part of an integrative approach to health care. An increasing number of American adults are seeking alternative and complementary health care options. Many popular alternative healing modalities offer human centered care based on philosophies that recognize the value of the patients input and honor the individual beliefs, values, and desires. Alternative healing modalities offer a holistic approach to health, focusing on the whole person and not just the disease

A nurse is caring for a child with the religion of Christian Science. What intervention should the nurse include in the care plan for this child? A. Ofer iced tea to the child who is experiencing deficient fluid volume B. Inform the spiritual care department that the child has been admitted to the hospital C. Allow parents to sign a form opting out of routine immunizations D. Ask parents whether the child has been baptized

C. Allow parents to sign a form opting out of routine immunizations Rationale: Coffee and tea are declined as a drink. When a Christian Science believer is hospitalized, a parent or patient may request that a Christian Science practitioner be notified as opposed to the hospital assigned clergy. Christian Science believers seek exemption from immunizations but obey legal requirements. Baptism is not a ceremony for the Christian Science religion

As a result of changes in health care delivery and funding, a current trend seen in the pediatric setting is: A. Increased hospitalization of children B. Decreased number of children living in poverty C. An increase in ambulatory care D. Decreased use of managed care

C. An increase in ambulatory care Rationale: Hospitalization for children has decreased. Health care delivery has not altered the number of children living in poverty. One effect of managed care has been that pediatric health care delivery has shifted dramatically from the acute care setting to the ambulatory setting. One of the biggest changes in health care has been the growth of managed care. The number of hospital beds being used has decreased as more care is given in outpatient settings and in the home. The number of children living in poverty has increased over the last decade. Managed care has increased in order to control cost

A student nurse is planning to communicate with her postpartum patient using an interpreter. Her instructor should provide further guidance if the student does which of the following? A. Chooses a female interpreter from the woman's country of origin B. Stops periodically during the interaction to ask the interpreter how things are going C. Asks questions while looking at the interpreter D. Gathers culturally appropriate learning aids and reading materials to used during the interaction

C. Asks questions while looking at the interpreter Rationale: the student should be looking at the woman while speaking and asking questions through the interpreter; a normal tone of voice should be used

Which nursing intervention is correctly written? A. Encourage turning, coughing, and deep breathing B. Force fluids as necessary C. Assist to ambulate for 10 minutes at 8AM, 2PM, and 6PM D. Observe interaction with infant

C. Assist to ambulate for 10 minutes at 8AM, 2PM, and 6PM Rationale: This intervention does not state how often this procedure should be done (A). Force fluids is not specific; it does not state how much. Interventions may not be carried out unless they are detailed and specific (C). This intervention is not detailed and specific (D).

To enhance the accuracy of the Papanicolaou (Pap) test, the nurse should instruct the patient to do which of the following? A. Schedule the test just prior to the onset of menses B. Stop taking birth control pills for 2 days before the test C. Avoid intercourse for 24 to 48 hours before the test D. Douche with a specially prepared antiseptic solution the night before the test

C. Avoid intercourse for 24 to 48 hours before the test Rationale: women should not use vaginal medications or douche for 24 to 48 hours before the test; OCPs can continue; the best time for the test is midcycle

The nurse is caring for a newborn whose mother had gestational diabetes. His estimated gestational age is 41 weeks, and his birthweight is 4800g. When assessing this newborn, the nurse should be alert for which of the following? A. Fracture of the femur B. Hypercalcemia C. Blood glucose level less than 40mg/dL D. Signs of a congenital heart defect

C. Blood glucose level less than 40mg/dL Rationale: fracture from trauma is more common in the upper body (e.g humerus, clavicle); hypocalcemia is common; the newborn of a gestational diabetic mother is more likely to experience congenital anomalies sucj as heart defects

What is the only known cure for preeclampsia? A. Magnesium sulfate B. Antihypertensive medications C. Delivery of the fetus D. Low-dose aspirin of 81 mg/day

C. Delivery of the fetus Rationale: If the fetus is viable and near term, delivery is the only known "cure" for preeclampsia. Magnesium sulfate and antihypertensive medications do not cure preeclampsia. Low-dose aspirin appears to have a modest preventive effect in women at high risk but little effect in low-risk women.

What is the only known cure for preeclampsia? A. Magnesium sulfate B. Antihypertensive medications C. Delivery of the fetus D. Administration of ASA every day of the pregnancy

C. Delivery of the fetus Rationale: Magnesium sulfate is one of the medications used to treat but not to cure preeclampsia. Antihypertensive medications are used to lower the dangerously elevated blood pressures in preeclampsia and eclampsia. If the fetus is viable and near term, delivery is the only known cure for preeclampsia. Low doses of ASA (60 to 80mg) have been administered to women at high risk for developing preeclampsia.

A student nurse's association is planning a health fair that will emphasize secondary level prevention activities. Which of the following activities would their faculty adviser tell them to eliminate because it does not fit into their stated focus? A. Teaching participants how to check their radial pulse for rate and regularity B. Blood glucose monitoring C. Demonstration of relaxation measures to manage stress D. Breast models that participants can use to learn breast palpation techniques

C. Demonstration of relaxation measures to manage stress Rationale: it represents primary level of prevention

What is the purpose of amniocentesis for the patient hospitalized at 34 weeks with pregnancy induced hypertension? A. Identification of abnormal fetal cells B. Detection of metabolic disorders C. Determination of fetal lung maturity D. Identification of sex of the fetus

C. Determination of fetal lung maturity Rationale: Identification of abnormal cells is done during the early portion of the pregnancy. The test is done in the early portion of the pregnancy if the metabolic disorder is genetic. During the third trimester, amniocentesis is most often performed to determine fetal lung maturity. In pregnancy induced hypertension, preterm delivery may be necessary because of changes in placental perfusion. Amniocentesis is done early in the pregnancy to do genetic studies and determine the sex

A woman is seeing her primary physician for complaints of frequent nosebleeds. She states she thought she was pregnant about 3 months ago, but her periods started and the symptoms disappeared. The health care provider should be alert for what complication of a missed abortion? A. Anemia B. Infertility C. Disseminated intravascular coagulation D. Thrombocytopenia

C. Disseminated intravascular coagulation Rationale: Disseminated intravascular coagulation is a major complication of a missed abortion. Bleeding may occur from any area, such as gums, nose, and cuts. Anemia, infertility, and thrombocytopenia are not complications of missed abortion.

To relieve a leg cramp, the patient should be instructed to: A. massage the affected muscle. B. stretch and point the toe. C. Dorsiflex the foot D. Apply a warm pack

C. Dorsiflex the foot Rationale: Since she is prone to blood clots in the legs, massaging the affected leg muscle is contraindicated. Pointing the toes will contract the muscle and not relieve the pain. Dorsiflexion of the foot stretches the leg muscle and relieves the painful muscle contraction. Warm packs can be used to relax thee muscle, but more immediate relief is necessary, such as dorsiflexion of the foot

A Native American woman has lactose intolerance. In doing patient teaching, the nurse should instruct the woman to include what foods in her diet to increase calcium intake? A. Milk, yogurt, cheeses B. Meat, legumes, fresh fruit C. Dried pinto beans, dark-green leafy vegetables, nuts D. She needs to get her calcium requirements through supplementation.

C. Dried pinto beans, dark-green leafy vegetables, nuts Rationale: Dried pinto beans, dark-green leafy vegetables, and nuts are substitutes for dairy products. The woman will not be able to tolerate milk and dairy products due to her lactose intolerance. Legumes can be used to increase calcium intake, but meat and fresh fruit are poor choices. Calcium supplementation may be necessary, but it is best to first attempt to obtain the woman's needs through dietary intake.

Metabolic changes throughout pregnancy that affect glucose and insulin in the mother and the fetus are complicated but important to understand. Nurses should know that A. insulin crosses the placenta to the fetus only in the first trimester, after which the fetus secretes its own B. Women with insulin dependent diabetes are prone to hyperglycemia during the first trimester, because they are consuming more sugar C. During the second and third trimesters, pregnancy exerts a diabetogenic effect that ensures an abundant supply of glucose for the fetus D. Maternal insulin requirements steadily decline during pregnancy

C. During the second and third trimesters, pregnancy exerts a diabetogenic effect that ensures an abundant supply of glucose for the fetus Rationale: Insulin never crosses the placenta; the fetus starts making its own around the tenth week. As a result of normal metabolic changes during pregnancy, insulin dependent women are prone to hypoglycemia (low levels). Pregnant women develop increased insulin resistance during the second and third trimesters. Maternal insulin requirements may double or quadruple by the end of pregnancy

Which suggestion is appropriate for the pregnant woman who is experiencing nausea and vomiting? A. Eat only three meals a day so the stomach is empty between meals B. Drink plenty of fluids with each meal C. Eat dry crackers or toast before arising in the morning D. Drink coffee or orange juice immediately on arising in the morning

C. Eat dry crackers or toast before arising in the morning Rationale: Instruct the woman to eat five to six small meals rather than three full meals peer day. Nausea is ore intense when the stomach is empty. Fluids should be taken separately from meals. Fluids overstretch the stomach and may precipitate vomiting. This will assist with the symptoms of morning sickness. It is also important for the woman to arise slowly. Coffee and orange juice stimulate acid formation in the stomach. It is best to suggest eating dry carbohydrates when rising in the morning

To manage her diabetes appropriately and ensure a good fetal outcome, the pregnant woman with diabetes. will need to alter her diet by A. Eating six small meals per day B. Reducing carbohydrates in her diet C. Eating her meals and snacks on a fixed schedule. D. Increasing her consumption of protein

C. Eating her meals and snacks on a fixed schedule. Rationale: It is more important to have a fixed meal schedule than equal division of food intake. Approximately 45%of the food eaten should be in the form of carbohydrates. Having a fixed meal schedule will provide the woman and the fetus with a steadier blood sugar level, provide better balance with insulin administration, and help prevent complications.

A woman is currently pregnant; she has a 5 year old son and a 3 year old daughter. She had one other pregnancy that terminated at 8 weeks. Her gravida and para are A. Gravida 3 para 2 B. Gravida 4 para 3 C. Gravida 4 para 2 D. Gravida 3 para 3

C. Gravida 4 para 2 Rationale: Because she is currently pregnant, she is classified as a gravida 4; the pregnancy that was terminated at 8 weeks is classified as an abortion. Gravida 4 is correct, but she is a para 2. The pregnancy that was terminated at 8 weeks is classified as an abortion. She has had four pregnancies, including the current one (gravida 4). She had two pregnancies that terminated after 20 weeks (para 2). The pregnancy that terminated at 8 weeks is classified as an abortion. Since she is currently pregnant, she is classified as a gravida 4, not a 3. The para is correct

Centering pregnancy is an example of an alternative model of prenatal care. Which. statement accurately applies to the centering model of care? A. Group sessions begin with the first prenatal visit B. At each visit blood pressure, weight, and urine dipsticks are obtained by the nurse. C. Eight to 12 women are placed in gestational age cohort groups D. Outcomes are similar to traditional prenatal care

C. Eight to 12 women are placed in gestational age cohort groups Rationale: Group sessions begin at 12 to 16 weeks of gestation and end with an early postpartum visit. Prior to group sessions, the patient has an individual assessment, physical examination, and history. At the beginning of each group meeting, patients measure their own BP, weight and urine dips and enter these in their record. Fetal heart rate assessment and fundal height are obtained by the nurse. Gestational age cohorts comprise the groups, with approximately 8 to 12 women in each group. This group remains intact throughout the pregnancy. Individual follow up visits are scheduled as needed. Results evaluating this approach have been very promising. In a recent study of adolescent patients, there was a decrease in LBW infants and an increase in breastfeeding rates

The United States ranks 25th in infant mortality rates of the world. Which factor has a significant impact on decreasing the mortality rate of infants? A. Resolving all language and cultural differences B. Enrolling the pregnant woman in the Medicaid program by the 8th month of pregnancy. C. Ensuring early and adequate prenatal care D. Providing more womens shelters

C. Ensuring early and adequate prenatal care Rationale: Language and cultural differences are not infant mortality issues but must be addressed to improve overall healthcare. Medicaid provides health care for poor pregnant women, but the process may take weeks to take effect. The 8th month is too late to apply and receive benefits for this pregnancy. Because preterm infants form the largest category of those needing expensive intensive care, early pregnancy intervention is essential for decreasing infant mortality rates. This is especially important for women in high risk groups, such as racial minorities, teenagers, and those living in poverty. The women in shelters have the same difficulties in obtaining health care as do other poor people, particularly lack of transportation and inconvenient hours of the clinics.

Which term refers to a shared cultural, social, and linguistic heritage? A. Beliefs B. Culture C. Ethnicity D. Socialization

C. Ethnicity Rationale: Ethnicity is an affiliation of a set of persons who share a unique cultural, social, religious, and linguistic heritage. Beliefs are attitudes that can be shared, not only within but also across ethnic groups. Culture is a pattern of beliefs and values that are learned and shared from generation to generation and guides the thinking, decisions, and actions of a group of people. Socialization is the process by which individuals learn the roles that are expected of them.

Families who deal effectively with stress exhibit which behavior pattern? A. Focus on family problems B. Feel weakened by stress C. Expect that some stress is normal D. Feel guilty when the stress exists

C. Expect that some stress is normal Rationale: Healthy families focus on family strengths rather than on the problems and know that stress is temporary and may be positive. If families are dealing effectively with stress, then weakening of the family unit should not occur. Healthy families recognize that some stress is normal in all families. Because some stress is normal in all families, feeling guilty is not reasonable. Guilt only immobilizes the family and does not lead to resolution of the stress.

A 54-year-old woman is experiencing symptoms of a urinary tract infection and needs to seek health care. Which advanced practice nurse would be the best choice for this woman? A. Certified nurse-midwife B. Clinical nurse specialist C. Family nurse practitioner D. Pediatric nurse practitioner

C. Family nurse practitioner Rationale: A family nurse practitioner is prepared to care for people of all ages. They can assess, diagnosis, and treat patients. A certified nurse-midwife provides care during pregnancy, childbirth, and postpartum. A clinical nurse specialist functions as a clinical leader but does not provide primary care. A pediatric nurse practitioner provides primary care to children.

In caring for a pregnant woman with sickle cell anemia, the nurse is aware that signs and symptoms of sickle cell crisis include: A. Anemia. B. Endometritis. C. Fever and pain. D. Urinary tract infection.

C. Fever and pain. Rationale: Women with sickle cell anemia are not iron deficient. Therefore routine iron supplementation, even that found in prenatal vitamins should be avoided in order to prevent iron overload. Women with. sickle cell trait usually are at greater risk for postpartum endometritis (uterine wall infection); however, this is not likely to occur in pregnancy and is not a sign of crisis. Women with sickle cell anemia have recurrent attacks (crisis) of fever and pain, most often in the abdomen, joints, and extremities. These attacks are attributed to vascular occlusion when RBCs assume the characteristic sickled shape. Crises are usually triggered by dehydration, hypoxia, or acidosis. The women are at an increase risk for UTIs; however, this is not an indication of sickle cell crisis

Which step in the nursing process identifies the basis or cause of the patients problem? A. Intervention B. Expected outcome C. Nursing diagnosis D. Evaluation

C. Nursing diagnosis Rationale: Interventions are actions taken to meet thee problem. Expected outcome is a statement of the goal. A nursing diagnosis states the problem and its cause (related to). Evaluation determines whether the goal has been met

The labor of a pregnant woman with preeclampsia is going to be induced. Before initiating the Pitocin infusion, the nurse reviews the womans latest laboratory test findings, which reveal a low platelet count, an elevated aspartate transaminase (AST) level, and a falling hematocrit. The nurse notifies the physician, because the lab results are indicative of A. eclampsia B. Disseminated intravascular coagulation C. HELLP syndrome D. Rh compatibility

C. HELLP syndrome Rationale: Eclampsia is determined by the presence of seizures. DIC is a potential complication associated with HELLP syndrome. HELLP syndrome is a laboratory diagnosis for a variant of severe preeclampsia that involves hepatic dysfunction characterized by hemolysis (H), elevated liver enzymes (EL), and low platelets (LP). These are not clinical indications of Rh incompatibility

While reviewing the dietary intake documentation of a 7 year old Asian boy with a fractured femur, the nurse notes that he consistently refuses to eat the food on his tray. What assumption is most likely accurate? A. He is a picky eater B. He needs less food because he is on bed rest C. He may have culturally related food preferences. D. He is probably eating between meals and spoiling his appetite

C. He may have culturally related food preferences. Rationale: Although the child may be a picky eater, the key point is that he is from a different culture. The foods he is being served may seem strange to him. Nutrition plays an important role in healing. Although the energy the child expends has decreased while on bed rest, he has increased needs for good nutrition. When cultural differences are noted, food preferences should always be obtained. A child will often refuse to eat unfamiliar foods. Although the nurse should determine whether the child is eating the food the family has brought from home, the more important point is to determine whether he has food preferences

A pregnant woman with a cardiac disorder will begin anticoagulant therapy to prevent clot formation. In preparing the woman for this treatment measure, the nurse would expect to teach the woman about self administration of which of the following medication? A. Furosemide B. Propranolol C. Heparin D. Warfarin

C. Heparin Rationale: furosemide is a diuretic; propranolol is a beta blocker that is used to manage hypertension and tachycardia; although warfarin is an anticoagulant, it can cross the placenta and affect the fetus, whereas heparin, which is a large molecule, does not

A 36 year old divorcee with a successful modeling career finds. out that her 18 year old married daughter is expecting her first child. What is a major factor in determining how the woman will respond to becoming a grandmother? A. Her career B. Being divorced C. Her age D. age of the daughter

C. Her age Rationale: Career responsibilities may have demands that make the grandparents not as accessible, but it is not a major factor in determining the womans response to becoming a grandmother. Being divorced is not a major factor that determines adaptation of grandparents. Age is a major factor in determining the emotional response of prospective grandparents. Young grandparents may not be happy with the stereotype of grandparents as being old. The age of the daughter is not a major factor that determines adaptation of grandparents. The age of the grandparent is a major factor

A woman who is older than 35 years may have difficulty achieving pregnancy primarily because: A. personal risk behaviors influence fertility B. She has used contraceptives for an extended time C. Her ovaries may be affected by the aging process D. Prepregnancy medical attention is lacking

C. Her ovaries may be affected by the aging process Rationale: The older adult participates in fewer risk behaviors than the younger adult. The problem is the age of the ovaries, not the past use of contraceptives. Once the mature woman decides to conceive, a delay in becoming pregnant may occur because of the normal aging of the ovaries. Prepregnancy medical care is available and discouraged

A nurse instructed a female patient regarding vulvar self examination (VSE). Which of the statements made by the patient will require further instruction? A. I will perform this examination at least once a month, especially if I change sexual partners or am sexually active B. I will become familiar with how my genitalia look and feel so I will be able to detect changes C. I will use the examination to determine when I should get medications at the pharmacy for infections D. I will wash my hands thoroughly before and after I examine myself

C. I will use the examination to determine when I should get medications at the pharmacy for infections Rationale: Self examination should not be used for self diagnosis but rather to detect early changes and seek guidance of a health care provider if changes are noted

What is the primary role of practicing nurses in the research process? A. Designing research studies B. Collecting data for other researchers C. Identifying researchable problems D. seeking funding to support research studies

C. Identifying researchable problems Rationale: Designing research studies is only one factor of the research process. Data collection is one factor of research. Nursing generates and answers its own questions based on evidence within its unique subject area. Financial support is necessary to conduct research, but it is not the primary role of the nurse in the research process.

When helping the mother, father, and other family members actualize the loss of an infant, the nurses should A. Use the words lost or gone rather than dead or died B. Make sure the family understands that it is important to name the baby C. If the parents choose to visit with the baby, apply lotion to the baby and wrap the infant in a pretty blanket D. Set a firm time for ending the visit with the baby so that the parents know when to let go

C. If the parents choose to visit with the baby, apply lotion to the baby and wrap the infant in a pretty blanket Rationale: Nurses must use dead and died to assist the bereaved in accepting reality. Although naming the baby can be helpful, it is important not to create the sense that parents have to name the baby. If fact, some cultural taboos and religious rules prohibit the naming of an infant who has died. Presenting the baby in a nice way stimulates the parents senses and provides pleasant memories of their baby. Parents need different time periods with their baby to say goodbye. Nurses need to be careful not to rush the process

A nurse assigned to a child does not know how to perform a treatment that has been prescribed for the child. What should the nurses first action be? A. Delay the treatment until another nurse can do it B. Make the childs parents aware of the situation C. Inform the nursing supervisor of the problem D. Arrange to have the child transferred to another unit

C. Inform the nursing supervisor of the problem Rationale: The nurse could endanger the child by delaying the intervention until another nurse is available. Telling the childs parents would most likely increase their anxiety and will not resolve the difficulty. If a nurse is not competent to perform a particular nursing task, the nurse must immediately communicate this fact to the nursing supervisor or physician. Transfer to another unit delays needed treatment and would create unnecessary disruption for the child and family

When providing care to the prenatal patient, the nurse understands that pica is A. intolerance of milk products B. iron deficiency anemia C. Ingestion of nonfood substances D. episodes of anorexia and vomiting

C. Ingestion of nonfood substances Rationale: This is termed lactose intolerance (A). Pica may produce iron deficiency anemia if proper nutrition is decreased. The practice of eating substances not normally thought of as food is called pica. Clay or dirt and solid laundry starch are the substances most commonly ingested. Pica is not related to anorexia and vomiting

When teaching the pregnant woman with class II heart disease, the nurse should A. Advise her to gain at least 30lb B. Explain the importance of a diet high in calcium C. Instruct her to avoid strenuous activity D. Inform her of the need to limit fluid intake

C. Instruct her to avoid strenuous activity Rationale: Weight gain should be kept at a minimum with heart disease. Iron and folic acid intake is important to prevent anemia. Activity may need to be limited so that cardiac demand does not exceed cardiac capacity. Fluid intake should not be limited during pregnancy. She may also be put on a diuretic. Fluid intake is necessary to prevent fluid deficits

An experienced maternity nurse needs to teach a new mother about bottle feeding. The mother is 25 years old and has a 2-year-old that she also bottle fed. She has been in the United States for 1 year and has a limited understanding of the English language. What factors will negatively influence the learning process and will cause the nurse to alter her teaching techniques? A. Developmental level and previous experiences B. Physical environment C. Language and culture D. Organization and skill of the teacher

C. Language and culture Rationale: The new mother is from a different culture and has English as a second language. The nurse will need to alter her teaching methods to accommodate these two factors to ensure learning. At 25 years of age, the mother has the developmental ability to learn. She has previous experience with bottle feeding. These two factors will increase the likelihood of successful teaching-learning. The physical environment is not addressed in the root of the question. The nurse is experienced in teaching new mothers about bottle feeding.

As a nurse in labor and delivery, you are caring for a Muslim woman during the active phase of her labor. You note that when you touch her, she quickly draws away. You should A. Continue to touch her as much as you need to while providing care B. Assume that she does not like you and decrease your time with C. Limit touching to a minimum, as this may not be acceptable in her culture D. Ask the charge nurse. to reassign you to another patient

C. Limit touching to a minimum, as this may not be acceptable in her culture Rationale: By continuing to touch her, the nurse is showing disrespect for her cultural beliefs. A Muslims response to touch does not reflect like or dislike. Touching is an important component of communication in various cultures, but if the patient appears to find it offensive, the nurse should respect her cultural beliefs and limit touching her. This reaction may be offensive to the patient

Family centered maternity care developed in response to : A. Demands by physicians for family involvement in childbirth B. The Sheppard-Towner Act of 1921 C. Parental requests that infants be allowed to remain with them rather than in a nursery D. Changes in pharmacologic management of labor

C. Parental requests that infants be allowed to remain with them rather than in a nursery Rationale: Family centered care was a request by parents, not physicians. The Sheppard-Towner Act provided funds for state managed programs for mothers and children. As research began to identify the benefits of early extended parent infant contact, parents began to insist that the infant remain with them. This gradually developed into the practice of rooming in and finally to family centered maternity care. The changes in pharmacologic management of labor were not a factor in family centered maternity care

In describing the size and shape of the nonpregnant uterus to a patient, the nurse would say it is approximately the size and shape of a A. Cantoloupe B. Grapefruit C. Pear D. Large orange

C. Pear Rationale: A cantaloupe is too large and the wrong shape for the uterus. A grapefruit is too large for the nonpregnant uterus, and the uterus is larger at the upper end and tapers down. The nonpregnant uterus is approximately 7.5 5.0 2.5 cm, which is close to the size and shape of a pear. An orange may be the appropriate size but not the appropriate shape

A woman has just decided to become a vegetarian and is 7 months' pregnant. The nurse knows that her patient teaching concerning protein foods has been successful when the woman chooses which foods to help satisfy her protein needs? A. Green salad with walnuts B. Fresh fruit salad with almonds C. Pinto beans with cornbread D. Chicken with rice

C. Pinto beans with cornbread Rationale: The combination of legumes and grains will give a complete protein that contains all of the essential amino acids. The walnuts are an incomplete protein, and the combination of vegetables will not make them complete. The almonds are an incomplete protein, and the combination of fruits will not make them complete. The chicken is a complete protein but is not included on a vegetarian diet.

The parent of a child who has had numerous hospitalizations asks the nurse for advice because her child has been having behavior problems at home and in school. In discussing effective discipline, what is an essential component? A. All children display some degree of acting out and this behavior is normal B. The child is manipulative and should have firmer limits set on her behavior C. Positive reinforcement and encouragement should be used to promote cooperation and the desired behaviors D. Underlying reasons for rules should be given and the child should be allowed to decide which rules should be followed

C. Positive reinforcement and encouragement should be used to promote cooperation and the desired behaviors Rationale: Behavior problems should not be disregarded as normal. It would be incorrect to assume the child is being manipulative and should have firmer limits set on her behaviors. Using positive reinforcement and encouragement to promote cooperation and desired behaviors is one of the three essential components of effective discipline. Providing the underlying reasons for rules and giving the child a choice concerning which rules to follow constitute a component of permissive parenting and are not considered an essential component of effective discipline

The purpose of the ovum zona pellucida is to A. Make a pathway for more than one sperm to reach the ovum B. Allow the 46 chromosomes from each gamete to merge C. Prevent multiple sperm from fertilizing the ovum D. Stimulate the ovum to being mitotic cell division

C. Prevent multiple sperm from fertilizing the ovum Rationale: Once sperm has entered the ovum, the zona pellucida changes to prevent other sperm from entering. Each gamete (sperm and ovum) has only 23 chromosomes. There will be 46 chromosomes when they merge. Fertilization causes the zona pellucida to change its chemical composition so that multiple sperm cannot fertilize the ovum. Mitotic cell division begins when the nuclei of the sperm and ovum unite

Which factor is known to increase the risk of gestational diabetes mellitus?: A. Underweight before pregnancy B. Maternal age younger than 25 years C. Previous birth of large infant D. Previous diagnosis of type 2 diabetes

C. Previous birth of large infant Rationale: Obesity (BMI of 30 or greater) creates a higher risk for gestational diabetes. A woman younger than 25 generally is not at risk for gestational diabetees mellitus. Previous birth of a large infant suggests gestational diabetes mellitus. The person with type 2 diabetes mellitus already is a diabetic and will continue to be so after pregnancy. Insulin may be required during pregnancy because oral hypoglycemia drugs are contraindicated during pregnancy

The parents of a young child ask the nurse for suggestions on how to discipline. When discussing the use of "timeouts," what approach should the nurse recommend? A. Send the child to his or her room if the child has one. B. Use 5 minutes per year of age for this discipline. C. Selecting an area that is safe and non-stimulating, such as a hallway. D. Try another approach if the child cries, refuses, or is more disruptive

C. Selecting an area that is safe and non-stimulating, such as a hallway. Rationale: The area must be non-stimulating and safe. The child becomes bored in this environment and then changes behavior to rejoin activities. The child's room may have toys and other equipment and activities that may negate the effect of being separated from the family activities. In addition, the place where the child sleeps should retain positive association and should not be used as a site of discipline. The general rule is 1 minute per year of age, not 5 minutes per year. When the child engages in such behavior, the child should be reminded that the timeout begins when the child quiets, rather than allowing the child to manipulate his way out of discipline.

The maternity nurse should have a clear understanding of the correct use of a clinical pathway. One characteristic of clinical pathways is that they: A. Are developed and implemented by nurses B. Are used primarily in the pediatric setting C. Set specific time lines for sequencing interventions D. Are part of the nursing process

C. Set specific time lines for sequencing interventions Rationale: Clinical pathways are developed by multiple healthcare professionals and reflect interdisciplinary interventions. They are used in multiple settings and for patients throughout the life span. Clinical pathways measure outcomes of patient care. Each pathway outlines specific time lines for sequencing interventions. The steps of the nursing process are assessment, diagnosis, planning, intervention, and evaluation

A pregnant woman who abuses cocaine admits to exchanging sex for her drug habit. This behavior puts her at greater risk for A. Depression of the central nervous system B. Hypotension and vasodilation C. Sexually transmitted diseases D. Postmature death

C. Sexually transmitted diseases Rationale: Cocaine is a central nervous system stimulant. Cocaine causes hypertension and vasoconstriction. Sex acts exchanged for drugs place the woman at increased risk for sexually transmitted diseases because of multiple partners and lack protection. Premature delivery of the infant is one of the most common problems associated with cocaine use during pregnancy

The fastest growing group of homeless people is A. Men and women preparing for retirement B. Migrant workers C. Single women and their children D. Intravenous (IV) substance abusers

C. Single women and their children Rationale: Most people contemplating retirement have made provisions. Migrant workers may seek health care only when absolutely necessary; however, not all are homeless. Pregnancy and birth, especially for a teenager, are contributing factors for being homeless. Not all substance abusers are homeless

In general, healthy families are able to adapt to changes within the family unit, however, some factors add to the usual stress experienced by any family. The nurse is in a unique position to assess the child for symptoms of neglect. Which high risk family situation places the child at the greatest risk for being neglected? A. Marital conflict and divorce B. Adolescent parenting C. Substance abuse D. A child with special needs

C. Substance abuse Rationale: Although divorce is traumatic to children, research has shown that living in a home filled with conflict is also detrimental. In this situation, conflict may arise and young children may be unable to verbalize their distress; however, the child is not likely to be neglected. Teenage parenting often has a negative effect on the health and social outcomes of the entire family. Adolescent girls are at risk for a number of pregnancy complications, are unlikely to attain a high level of education, and are more likely to be poor. Parents who abuse drugs or alcohol may neglect their children because obtaining and using the substance(s) may have a stronger pull on the parents than the care of their children. When a child is born with a birth defect or has an illness that requires special care, the family is under additional stress. These families often suffer financial hardship as health insurance benefits quickly reach their maximum

A patient who is 7 months pregnant states, I'm worried that something will happen to my baby. The nurses best response is A. There is nothing to worry about B. The doctor is taking good care of you and your baby C. Tell me about your concerns D. Your baby is doing fine

C. Tell me about your concerns Rationale: This statement is belittling the patients concerns (A). This statement is belittling the patients concerns by telling her she would not worry. Encouraging the client to discuss her feelings is the best approach. Women during their third trimester need reassurance that such fears are not unusual in pregnancy. This statement disregards the patients feelings and treats them as unimportant

A pregnant woman in her first trimester tells her nurse midwife that although she does drink during pregnancy, she does so only on the weekend and only a little bit. What should the nurse's initial response be to this woman's comment? A. You need to realize that use of alcohol in any amount will result in your child being mentally retarded B. I am going to refer you to a counseling center for women with alcohol problems C. Tell me what you mean by drinking a little on the weekend D. Antioxidants can reduce the effect of alcohol exposure for your baby. I will tell you what you can take

C. Tell me what you mean by drinking a little on the weekend Rationale: it is essential to first determine what she drinks and how much; once that is established, then intervention regarding alcohol use that is individualized for her can occur

A woman in her first trimester of pregnancy can expect to visit her physician every 4 weeks so that A. she develops trust in the health care team B. Her questions about labor can be answered C. The condition of the expectorant mother and fetus can be monitored D. Problems can be eliminated

C. The condition of the expectorant mother and fetus can be monitored Rationale: Developing a trusting relationship should be established during these visits, but that is not the primary reason. Most women do not have questions concerning labor until the last trimester of the pregnancy. This routine allows monitoring of maternal health and fetal growth and ensures that problems will be identified early. All problems cannot be eliminated because of prenatal visits, but they can be identified

A patient at 24 weeks of gestation says she has a glass of wine with dinner every evening. The nurse will counsel her to eliminate all alcohol intake, becauase A. A daily consumption of alcohol indicates a risk for alcoholism B. She will be at risk for abusing other substances as well C. The fetus is placed at risk for altered brain growth D. The fetus is at risk for multiple organ anomalies

C. The fetus is placed at risk for altered brain growth Rationale: This is not a major risk for the infant (A).This has not been proven (B). The brain grows more rapidly in the third trimester and is most vulnerable to alcohol exposure during this time. The major concerns are mental retardation, learning disabilities, high activity level, and short attention span

In practical terms regarding genetic health care, nurses should be aware that A. Genetic disorders equally affect people of all socioeconomic backgrounds, races, and ethnic groups B. Genetic health care is more concerned with population than individuals C. The most important of all nursing functions is providing emotional support to the family during counseling D. Taking genetic histories is the province of large universities and medical centers

C. The most important of all nursing functions is providing emotional support to the family during counseling Rationale: Although anyone may have a genetic disorder, certain disorders appear more often in certain ethnic and racial groups. Genetic health care is highly individualized, because treatments are based on the phenotypic responses of the individual. Nurses should be prepared to help with a variety of stress reactions from a couple facing the possibility of a genetic disorder. Individual nurses at any facility can take a genetic history, although larger facilities may have better support services

Which patient situation fails to meet the first requirement of informed consent? A. The patient does not understand the physicians explanations B. The physician gives the patient only a partial list of possible side effects and complications C. The patient is confused and disoriented D. The patient signs a consent form because her husband tells her to

C. The patient is confused and disoriented Rationale: Understanding is an important element of the consent, but first the patient has to be competent to sign. Full disclosure of information is an important element of the consent, but first the patient has to be competent to sign. The first requirement of informed consent is that the patient must be competent to make decisions about health care. Voluntary consent is an important element of the consent, but first the patient has to be competent to sign

What is a major barrier to health care for teen mothers? A. The hospital/clinic is within walking distance of the girl's home. B. The institution is open days, evenings, and Saturday by special arrangement. C. The teen must be prepared to see a different nurse and/or physician at every visit. D. The health care workers have a positive attitude.

C. The teen must be prepared to see a different nurse and/or physician at every visit. Rationale: Whenever possible, the teen should be scheduled to see the same nurses and practitioners for continuity of care. Having a clinic within walking distance with flexible hours and staff with positive attitudes are all interventions that eliminate barriers to health care.

What is a major barrier to health care for teen mothers? A. The hospital/clinic is within walking distance of the girls home B. The institution is open days, evenings and Saturday by special arrangement C. The teen must be prepared to see a different nurse or doctor or both at every visit. D. The health care workers have a positive attitude

C. The teen must be prepared to see a different nurse or doctor or both at every visit. Rationale: If the hospital/clinic were within walking distance of the girls home, it would prevent the teen from missing appointments because of transportation problems. If the institution were open days, evenings and Saturday by special arrangement, this availability would be helpful for teens who work, go to school, or have other time of day restrictions. Scheduling conflicts are a major barrier to health care. Whenever possible, the teen should be scheduled to see the same nurses and practitioners for continuity of care. A negative attitude is unfortunate, because it discourages families that would benefit most from consistent prenatal care

Which setting for childbirth allows the least amount of parent infant contact? A. Labor/delivery/recovery/postpartum room B. Birth center C. Traditional hospital birth D. Home birth

C. Traditional hospital birth Rationale: The labor/delivery/recovery/postpartum room setting allows increased parent-infant contact. Birth centers are set up to allow an increase in parent-infant contact. In the traditional hospital setting, the mother may see the infant for only short feeding periods, and the infant is cared for in a separate nursery. Home births allow and increase in parent-infant contact

Ultrasonography is an important and safe technique used in antepartum surveillance. It provides critical information related to fetal activity, gestational age and fetal well being. Which statement regarding ultrasonography during pregnancy is most accurate? A. Ultrasonography uses infared technology to create an image B. Ultrasonography is only utilized as an adjunct to more invasive tests C. Ultrasonography is not harmful to the fetus D. Ultrasonography is not a component of biophysical profile testing

C. Ultrasonography is not harmful to the fetus Rationale: Ultrasonography uses sound waves to create an image. As an adjunct to more invasive tests, ultrasonography can provide visual guidance for increased safety. It can be done as a standalone test. Most women look forward to the results of this test, which causes no harm to the fetus. Ultasonography is a component of biophysical profile testing.

Methotrexate is recommended as part of the treatment plan for which obstetric complication? A. Complete hydatidiform mole B. Missed abortion C. Unruptured ectopic pregnancy D. Abruptio placentae

C. Unruptured ectopic pregnancy Rationale: Methotrexate is not indicated or recommended as a treatment option for a complete hydatidiform mole. Methotrexate is not indicated or recommended as a treatment option for missed abortions. Methotrexate. is an effective, nonsurgical treatment option for hemodynamically stable women whose ectopic pregnancy is unruptured and less than 4cm in diameter. Methotrexate is not indicated or recommended as a treatment option for abruptio placentae

How can a woman avoid exposing her fetus to teratogens? A. Update her immunizations during the first trimester of pregnancy B. Use saunas and hot tubs during the winter months only C. Use only class A drugs during her pregnancy D. Use alcoholic beverages only in the first and third trimesters of pregnancy

C. Use only class A drugs during her pregnancy Rationale: Immunizations, such as rubella, are contraindicated in pregnancy. Maternal hyperthermia is an important teratogen. In well-controlled studies, class A drugs have demonstrated no fetal risk. Alcohol is an environmental substance known to be teratogenic

A new breastfeeding mother asks the nurse how to prevent nipple soreness. The nurse tells this woman that the key to preventing sore nipples would be which of the following? A. Limiting the length of breastfeeding to no more than 10 minutes on each breast until the milk comes in B. Applying lanolin to each nipple and areola after each feeding C. Using correct breastfeeding technique D. Using nipple shells to protect the nipples and areola between feeding

C. Using correct breastfeeding technique Rationale: limiting length of feeding does not protect the nipples and areola; B. and D. are correct actions but not the most important

During prenatal teaching it is important for the nurse to inform the patient about danger signs in pregnancy. Which sign need to be reported immediately to the health care provider? A. Clear mucous vaginal discharge B. Frequent urination C. Vaginal bleeding D. Backache that occurs after standing for a long period

C. Vaginal bleeding Rationale: Vaginal bleeding during pregnancy needs to be reported immediately. It may be an indication of several complications of pregnancy, such as placenta previa or abruptio placenta. Mucous discharge may increase during pregnancy and is considered normal. Frequent urination is common during the first trimester and later in the third trimester. Backaches are the most common complaint during the third trimester.

When addressing the questions of a newly pregnant woman, the nurse can explain that the certified nurse midwife is qualified to perform: A. Regional anesthesia B. Cesarean deliveries C. Vaginal deliveries D. Internal versions

C. Vaginal deliveries Rationale: Regional anesthesia must be performed by a physician. Cesarean deliveries must be performed by a physician. The nurse midwife is qualified to deliver infants vaginally in uncomplicated pregnancies. Internal versions must be performed by a physician

Nursing intervention for the pregnant diabetic is based on the knowledge that the need for insulin A. increases throughout pregnancy and the postpartum period B. Decreases throughout pregnancy and the postpartum period C. Varies depending on the stage of gestation D. Should not change because the fetus produces its own insulin

C. Varies depending on the stage of gestation Rationale: Insulin needs decrease during the first trimester, when nausea, vomiting, and anorexia are a factor. Insulin needs during the second and third trimesters, when the hormones of pregnancy create insulin resistance in maternal cells. The insulin needs change during the pregnancy

A pregnant woman's diet consists almost entirely of whole grain breads and cereals, fruits, and vegetables. The nurse should be most concerned about this woman's intake of A. Calcium B. Protein C. Vitamin B12 D. Folic acid

C. Vitamin B12 Rationale: Depending on the woman's food choices this diet may be adequate in calcium. Protein needs can be sufficiently met by a vegetarian diet. This diet is consistent with that followed by a strict vegetarian (vegan). Vegans consume only plant products. Because vitamin B12 is found in foods of animal origin, the diet is deficient in vitamin B12. The nurse should be more concerned with the woman's intake of vitamin B12 due to her dietary restrictions. Folic acid can be met by enriched bread products

A woman is admitted through the emergency room with a medical diagnosis or ruptured ectopic pregnancy. The primary nursing diagnosis at this time would be: A. acute pain related to irritation of the peritoneum with blood B. risk for infection related to tissue trauma C. deficient fluid volume related to blood loss associated with rupture of the uterine tube D. anticipatory grieving related to unexpected pregnancy outcome

C. deficient fluid volume related to blood loss associated with rupture of the uterine tube Rationale: A,B,D are appropriate nursing diagnoses, but deficient fluid is the most immediate concern, placing the woman's well being at greatest risk

A 35 year old woman has an amniocentesis performed to find out whether her baby has a chromosome defect. Which statement by this patient indicates that she understands her situation? A. The doctor will tell me if I should have an abortion when the tests results come back B. I know support groups exist for parents who have a baby with birth defects, but we have plenty of insurance to cover what we need C. When all the lab results come back, my husband and I will make a decision about the pregnancy D. My mother must not find out about all this testing. If she does, she will think I'm having an abortion

C. When all the lab results come back, my husband and I will make a decision about the pregnancy Rationale: The final decision about the future of the pregnancy lies with the patient only. Support groups are extremely important for parents of a baby with a defect. Insurance will help cover expenses, but the defect also takes a toll on the emotional, physical, and social aspects of the parents lives. The final decision about genetic testing and the future of the pregnancy lies with the patient. She will involve only those people whom she chooses in her decisions. An amniocentesis is performed to detect chromosomal defects. Many women have this done to prepare and educate themselves for the baby's arrival. The woman should also be assured that her care is confidential

To resolve family conflict, it is necessary to have open communication, accurate perception of the problem and a(n) A. Intact family structure B. Arbitrator C. Willingness to consider the view of others D. Balance in personality types

C. Willingness to consider the view of others Rationale: The structure of a family may affect family dynamics, but it is still possible to resolve conflict without an intact family structure if all of the ingredients of conflict resolution are present. Conflicts can be resolved without the assistance of an arbitrator. Without the willingness of the members of a group to consider the views of others, conflict resolution cannot take place. Most families have diverse personality types among their members. This diversity may make conflict resolution more difficult but should not impede it as long as the ingredients of conflict resolution are present

Which response by the nurse is most appropriate to this statement: This test isn't my idea, but my husband insists? A. Don't worry. Everything will be fine B. Why don't you want to have this test C. You're concerned about having this test D. It's your decision

C. You're concerned about having this test Rationale: This is false reassurance and does not deal with the concerns expressed by the woman (A). The woman may not be able to answer why questions. It may also make her defensive. The nurse should clarify the statement and assist the patient in exploring here feelings about the test. This is a closed statement and does not encourage the woman to express her feelings

When comparing threatened abortion to inevitable abortion, inevitable abortion has A. increased cramping. B. backache. C. cervical dilation. D. spotting.

C. cervical dilation. Rationlae: Abortion becomes inevitable when the membranes rupture and the cervix dilates. Cramping and backache may increase as the abortion progresses, but cervical dilation is the proof that it is inevitable. Active bleeding may be present and may be heavy.

In planning care for a new patient in the prenatal clinic, the nurse is aware that various test are available to screen for fetal abnormalities. One that is used early in pregnancy is A. maternal blood test for folic acid levels. B. testing fetal blood for metabolic disorders. C. chorionic villus sampling. D. maternal complete blood cell count.

C. chorionic villus sampling. Rationale: Chorionic villus sampling is done in the first trimester. The chromosomes will be analyzed for fetal abnormalities. Folic acid intake is extremely important early in the pregnancy to prevent fetal abnormalities. However, blood testing for folic acid levels and CBC will not screen for fetal abnormalities. Testing for metabolic disorders is done on the newborn.

The nurse informs the 36-year-old primigravida that she may have a multiple marker screening test performed because she is at risk for A. gestational diabetes. B. preeclampsia. C. chromosomal anomalies of the fetus. D. preterm labor.

C. chromosomal anomalies of the fetus. Rationale: The fetus of a mature woman is at increased risk for chromosomal anomalies. A multiple marker screening is used to screen for specific types of chromosomal anomalies. The older woman is at increased risk for pregnancy complications, but the multiple marker screening does not test for diabetes, preeclampsia, or preterm labor.

It is important for the nurse to understand that the levator ani is a(n) A. imaginary line that divides the true and false pelvis B. basin shaped structure at the lower end of the spine C. collection of three pairs of muscles D. division of the fallopian tube

C. collection of three pairs of muscles Rationale: The linea terminalis is the imaginary line that divides the false pelvis from the true pelvis. The basin shaped structure at the lower end of the spine is the bony pelvis. The levator ani is a collection of three pairs of muscles that support internal pelvic structures and resist increases in intraabdominal pressure. The fallopian tube divisions are the interstitial portion, isthmus, ampulla, infundibulum

In teaching about weight gain during pregnancy, the nurse should include A. the pregnant woman should gain a total of 25 pounds regardless of her beginning weight. B. the woman who is overweight prior to pregnancy should not gain any weight during the pregnancy. C. during the third trimester, the pregnant woman should gain about 1 pound of weight per week. D. a young adolescent pregnant woman should gain less than the recommended weight gain.

C. during the third trimester, the pregnant woman should gain about 1 pound of weight per week. Rationale: The pattern of weight gain is as important as the total increase. The general recommendation for the third trimester is 1 pound per week. Weight gain during pregnancy differs according to the beginning weight. A woman who is overweight should gain 15 to 25 pounds; a woman who is underweight should gain 28 to 40 pounds. Weight gain is important during pregnancy even for a woman who is overweight. An adolescent pregnant woman needs to gain weight toward the top end of the recommended weight gains. She needs to provide not only for the fetus but also for her own growth.

When doing an initial assessment on a newly diagnosed pregnant woman, she tells the nurse, "In my younger days, I did some stupid things and had different types of STDs and once had a pelvic inflammatory disease." The nurse is aware that the woman is at risk for A. spontaneous abortion. B. hypertension. C. ectopic pregnancy D. infections.

C. ectopic pregnancy Rationale: STDs and PID both may cause scar tissue in the fallopian tubes. This will prevent the fertilized ovum from traveling into the uterus, and therefore it will implant in the tube. The patient is not at increased risk of spontaneous abortion, hypertension, or infections.

When assisting with the collection of a specimen for chromosome analysis, the nurse must A. provide collection tubes that have no preservatives. B. place the collection tubes on ice immediately. C. ensure that the cells in the specimen stay alive. D. observe the length of time required for clot formation.

C. ensure that the cells in the specimen stay alive. Rationale: Specimens for chromosome analysis must contain living cells, because chromosomes are visible microscopically only in living dividing cells.

A woman has been diagnosed with iron deficiency anemia. The nurse knows that patient teaching about increasing iron intake has been effective if the woman chooses A. boiled eggs, whole grain toast, and milk. B. baked chicken, pinto beans, and iced tea. C. green salad with broccoli, black beans, and strawberries. D. broiled fish, fried potatoes, and green beans.

C. green salad with broccoli, black beans, and strawberries. Rationale: Green leafy vegetables and legumes are rich in iron. The strawberries are rich in vitamin C, which increases the absorption of iron. Boiled eggs and whole grain bread are rich in iron, but milk will decrease the absorption of iron and should not be taken at the same time. Meats and legumes are rich in iron; tea will decrease the absorption of iron and should not be taken at the same time. Broiled fish, fried potatoes, and green beans are not rich in iron.

A nurse manager of a prenatal clinic should recognize that the most significant barrier encountered by pregnant women in accessing health care would be which of the following? A. Lack of transportation to the clinic B. Child care responsibilities C. Inability to pay D. Deficient knowledge related to the benefits of prenatal care

C. inability to pay Rationale: Although the other options are important, inability to pay, which relates to inadequate or no health care insurance, is the most significant barrier

A woman tells the nurse she is 16 weeks' pregnant. During the assessment, the nurse measures the fundus of the uterus to be at the umbilicus. The nurse correctly interprets the comparison of the dates with the measurements to be A. not comparable. B. congruent. C. incongruent. D. irrelevant.

C. incongruent. Rationale: The fundus should be at the umbilicus by 20 weeks. At 16 weeks, it is normally midway between the symphysis pubis and the umbilicus. The two sets of data do not match, and more assessment is necessary. From 16 to 18 weeks until 36 weeks, the fundal height, measured in centimeters, is approximately equal to the gestational age of the fetus in weeks.

The nurses role in diagnostic testing is to provide A. advice to the couple B. assistance with decision making C. information about the tests D. reassurance about fetal safety

C. information about the tests Rationale: The nurses role is to inform, not advise. Decision making should always lie with the couple involved. The nurse should provide information so that the couple can make an informed decision. Ensuring fetal safety is not possible with all the diagnostic testing. To offer this is to give false reassurance to the parents. The nurse can inform the couple about potential problems so they can make an informed decision

When teaching a pregnant woman with class II heart disease, the nurse should A. advise her to gain at least 30 pounds. B. explain the importance of a diet high in sodium. C. instruct her to avoid strenuous activity. D. inform her of the need to limit fluid intake.

C. instruct her to avoid strenuous activity. Rationale: Activity may need to be limited so that cardiac demand does not exceed cardiac capacity. Weight gain is kept low in order to reduce the workload of the heart. Sodium intake should be adequate. Low-sodium diets are recommended at times to prevent congested heart failure. Fluids do not need to be limited.

Following administration of fentanyl (Sublimaze) IV for labor pain, a woman's labor progresses more rapidly than expected. The physician orders that a stat dose of nalaxone (Narcan) 1mg be administered intravenously to the woman to reverse respiratory depression in the newborn after its birth. In fulfilling this order the nurse would: A. question the route because this medication should be administered orally B. recognize that the dose is too low C. monitor maternal condition for possible side effects D. observe the woman for bradycardia and lethargy

C. monitor maternal condition for possible side effects Rationale: administration of Narcan is given to reverse effects of sedation as the client is experiencing potential symptoms and/or in this case delivery is imminent and the provider wants to counteract possible fetal effects as a result of maternal narcotic administration. The nurse should continue to monitor maternal condition for possible side effects of the medication.

To evaluate the woman's learning about performing infant care, the nurse should A. demonstrate infant care procedures. B. allow the woman to verbalize the procedure. C. observe the woman as she performs the procedure. D. routinely assess the infant for cleanliness.

C. observe the woman as she performs the procedure. Rationale: Demonstration is an excellent teaching method, but not an evaluation method. During verbalization of the procedure, the nurse may not pick up on techniques that are incorrect. It is not the best tool for evaluation. The woman's ability to perform the procedure correctly under the nurse's supervision is the best method of evaluation. This will not ensure that the proper procedure is carried out. The nurse may miss seeing unsafe techniques being used (D).

To increase the absorption of iron in a pregnant woman, the iron preparation should be given with A. milk B. Tea C. orange juice D. coffee

C. orange juice Rationale: The calcium and phosphorous in milk decrease iron absorption. Tannin in the tea reduces the absorption of iron. A vitamin C source may increase the absorption of iron. A decreased intake of caffeine is recommended in pregnancy

The classic sign of placenta previa is the sudden onset of A. heavy bleeding in the first half of pregnancy. B. severe cramping in the latter half of pregnancy. C. painless, uterine bleeding in the latter half of pregnancy. D. rupture of the bag of waters.

C. painless, uterine bleeding in the latter half of pregnancy. Rationale: The classic sign of placenta previa is the sudden onset of painless, uterine bleeding in the latter half of pregnancy Bleeding occurs when the placental villi are torn from the uterine wall, resulting in hemorrhage from the uterine vessels. Bleeding is typically painless because it does not occur in a closed cavity and therefore does not cause pressure on adjacent tissue. Rupture of the bag of waters is not a sign of placenta previa.

A pregnant woman at 32 weeks of gestation comes to the emergency room because she has begun to experience bright red vaginal bleeding. She reports that she is experiencing no pain. The admission nurse suspects: A. abruptio placentae B. disseminated intravascular coagulation C. placenta previa D. preterm labor

C. placenta previa Rationale: the clinical manifestations of placenta previa are described; dark red bleeding with pain is characteristic or abruptio placente; massive bleeding from many sites is associated with DIC; bleeding is not a sign of preterm labor

As part of preparing a 24 year old woman at 42 weeks of gestation for a nonstress test, the nurse would: A. tell the woman to fast for 8 hours before the test B. explain that the test will evaluate how well her baby is moving inside her uterus C. show her how to indicate when her baby moves D. attach a spiral electrode to the presenting part to determine FHR patterns

C. show her how to indicate when her baby moves Rationale: food/fluid is not restricted before the test; the test will evaluate thee response of the fetal heart rate (FHR) to fetal movement- acceleration is expected; external not internal monitoring is used

Alterations in hormonal balance and mechanical stretching are responsible for several changes in the integumentary system during pregnancy. Stretch marks often occur on the abdomen and breasts. These are referred to as A. Chloasma B. Linea nigra C. striae gravidarum D. Angiomas

C. striae gravidarum Rationale: Chloasma is a facial melasma also known as a mask of pregnancy. This condition is manifested by a blotchy, hyperpigmentation of the skin over the cheeks, nose and forehead especially in dark complexioned women. Linea nigra is a pigmented line extending from the symphysis pubis to the top of thee fundus in the midline. Striae gravidarum or stretch marks appear in 50 to 90% of pregnant women during the second half of pregnancy. They most often occur on thee breasts and abdomen. This integumentary alteration is the result of separation within the underlying connective (collagen) tissue. Angiomas and other changes also may appear

During a contraction stress test, four contractions lasting 45 to 55 seconds were recorded in a 10 minute period. A late deceleration was noted during the third contraction. The nurse conducting the test would document that the result is: A. negative B. positive C. suspicious D. unsatisfactory

C. suspicious Rationale: A suspicious result is recorded when late decelerations occur with less than 50% of the contractions; a negative test result is recorded when no late decelerations occur during at least three uterine contractions lasting 40 to 60 seconds each, within a 10 minute period; a positive test is recorded when there are persistent late decelerations with more than 50% of the contractions; unsatisfactory is the result recorded when there is a failure to achieve adequate uterine contractions.

A laboring woman's temperature is elevated as a result of an upper respiratory infection. The FHR pattern that reflects maternal fever would be A. diminished variability B. Variable decelerations C. tachycardia D. early decelerations

C. tachycardia Rationale: The FHR increases as the maternal core body temperature rises; therefore, tachycardia would be the pattern exhibited; it is often a clue of intrauterine infection because maternal fever is often the first sign; diminished variability reflects hypoxia and variable decelerations are characteristic of cord compression; early decelerations are characteristic of head compression and are not considered an abnormal pattern

A woman who is 6 weeks' pregnant is in for her prenatal appointment and asks the nurse when the sex of the baby can been determined by ultrasound. The nurse bases her answer on the knowledge that A. the sex cannot be determined by ultrasound; an amniocentesis must be done. B. the sex of the baby can be determined now by ultrasound. C. the external genitalia look similar in both males and females until about 9 weeks of gestation. D. the external genitalia will look different at 6 weeks, so an ultrasound can be done at that time.

C. the external genitalia look similar in both males and females until about 9 weeks of gestation. Rationale: The external genitalia starts to change at about 9 weeks of gestation. Prior to that time, males and females look similar and it is not possible to determine the sex from ultrasound. Sex of a fetus can be determined by ultrasound after the 9th week.

Implantation of the zygote should occur in the upper portion of the uterus. This is the best area for the growing fetus and placenta for all of these reasons except A. the upper uterus is richly supplied with blood. B. the upper uterus lining is thick. C. the upper uterus is supplied with the beginnings of the umbilical cord. D. the muscle fibers of the upper uterus prevent blood loss post delivery.

C. the upper uterus is supplied with the beginnings of the umbilical cord. Rationale: The umbilical cord develops from the zygote. The upper uterus is rich with blood. This is optimal for fetal gas exchange and nutrition. The uterine lining is thicker in the upper portion. This prevents the placenta from attaching so deeply that it does not easily detach after birth. The muscle fibers of the upper portion of the uterus are strong interlacing fibers that will clamp down over the site of placental separation, preventing excessive bleeding.

When measuring the BP to ensure consistency and to facilitate early detection of BP changes consistent with gestational hypertension, the nurse should: A. place the woman in a supine position B. allow the woman to rest for at least 15 minutes before measuring her BP C. use the same arm for each BP measurement D. use a proper sized cuff that covers at least 50% of her upper arm

C. use the same arm for each BP measurement Rationale: The woman should be seated or in a lateral position, she should rest for 5 to 10 minutes, and the cuff should cover 80% of the upper arm

A woman is in active labor and about to deliver a baby girl (diagnosed by ultrasound). The woman's husband has hemophilia A. The nursery nurse planning to care for the infant needs to be aware that she A. will be a hemophilic and therefore must be prepared for bleeding problems. B. will have a 50% chance of being a carrier but will not have the disorder. C. will be a carrier. D. will neither be a carrier nor have the disorder.

C. will be a carrier. Rationale: Hemophilia is an X-linked recessive disorder. The father will pass the disorder to 100% of their daughters, and the daughters will become carriers.

Changes occur as a woman progresses through labor. Which of the following maternal adaptations would be expected during labor? (Select all that apply) A. Increase in both systolic and diastolic blood pressure during uterine contractions in the first stage of labor B. Decrease in white blood cell count C. Slight increase in heart rate during the first and second stages of labor D. Decrease in gastric motility leading to nausea and vomiting during the first stage of labor E. Hypoglycemia F. Proteinuria up to 1+

C.D.E.F. systolic blood pressure increases with uterine contractions in the first stage, whereas both systolic and diastolic blood pressure increase during contractions in the second stage; WBC increases

A newly married 25 year old woman has been smoking since she was a teenager. She has come to the women's health clinic for a checkup before she begins trying to get pregnant. The woman demonstrates a need for further instruction about the effects of smoking on reproduction and health when she makes which of the following statements (Select all that apply) A. Smoking can interfere with my ability to get pregnant B. My husband also needs to stop smoking because secondhand smoke can have an adverse effect on my pregnancy and the development of the baby C. Smoking can make my pregnancy last longer than it should D. Smoking can reduce the amount of calcium in my bones E. Smoking will mean I will experience menopause at an older age than my friends who do not smoke

C.E. Smoking can make my pregnancy last longer than it should; Smoking will mean I will experience menopause at an older age than my friends who do not smoke Rationale: smoking is associated with preterm birth, not postterm pregnancies; smoking can reduce the age for menopause

When caring for a pregnant woman who is Hispanic, the nurse should recognize that the woman may be guided by which of the following cultural beliefs and practices during her pregnancy? (Select all that apply) A. Warm breezes can be dangerous during pregnancy B. Prenatal care should begin very early in pregnancy C. Milk should be avoided because it results in big babies and difficult births D. Drinking chamomile tea should be avoided because it can cause preterm labor E. The advice of both mother and mother in law should be followed during pregnancy F. Pelvic examinations should not be performed by a male health care provider

C.E.F. Milk should be avoided because it results in big babies and difficult births; The advice of both mother and mother in law should be followed during pregnancy; Pelvic examinations should not be performed by a male health care provider Rationale: air movement is considered dangerous; prenatal care should begin late in pregnancy; chamomile tea is thought to ensure effective labor

A pregnant womans diet may not meet her need for folates. A good source of this nutrient is A. Chicken B. Cheese C. Potatoes D. Green leafy vegetables

D. Green leafy vegetables Rationale: Chicken is an excellent source of protein, but it is poor in folates. Cheese is an excellent source of calcium, but it is poor in folates. Potatoes contain carbohydrates and vitamins and minerals, but are poor in folates. Sources of folates include green leafy vegetables, whole grains, fruits, liver, dried peas, and beans

A 27-year-old is in her third trimester. She gives the nurse the following 24-hour dietary recall:Breakfast: 1 cup cereal with 1 cup skim milk, ½ cup orange juiceLunch: 1 ounce ham and 2 slices whole wheat bread and 1 cup yogurtSupper: 3 ounces broiled chicken breast, ½ cup green beans, whole wheat rollSnack: 2 ounces cheese with 8 crackersWhen comparing this dietary recall with MyPlate, which snacks should the nurse suggest to the woman to meet her dietary deficits? A. 1 cup cottage cheese with fresh fruit B. Grilled cheese sandwich C. Yogurt with walnuts, crackers D. 2 cups salad greens with 3 ounces lean turkey, broccoli, and kidney beans

D. 2 cups salad greens with 3 ounces lean turkey, broccoli, and kidney beans Rationale: The woman is low on protein, vitamin A source, and vegetables. The broccoli and dark green salad greens are a good vitamin A source, plus they satisfy the need for vegetables. The turkey and kidney beans satisfy the protein needs. She has taken in more than the three servings of dairy products and whole grains that she needs. She is lacking in fruits and vegetables.

Health teaching during routine prenatal care includes providing patients with the recommended weight gain during pregnancy. For a woman with a single fetus who begins pregnancy at a normal weight, the amount is _____lb. A. 10 to 15 B. 15 to 20 C. 37 to 50 D. 28-40

D. 28-40 Rationale: A 10lb weight gain is not sufficient to meet the needs of the pregnancy. A 15 to 20 lb weight gain is recommended for women who are overweight before the pregnancy. A 37-50lb weigh gain is recommended for women who are carrying twins. When women with a multifetal pregnancy gain the recommended amount of weight, they are less likely to deliver prior to 32 weeks and infants are more likely to weigh more than 2500gram. A weight gain of 28-40lb is believed to reduce intrauterine growth retardation that may result from inadequate nutrition and also allows variations in individual needs. No precise weight gain is appropriate for every woman

Some of the embryo's intestines remain within the umbilical cord during the embryonic period because the: A. umbilical cord is much larger at this time than it will be at the end of the pregnancy. B. Intestines begin their development within the umbilical cord C. Nutrient content of the blood is higher in this location. D. Abdomen is too small to contain all the organs while they are developing

D. Abdomen is too small to contain all the organs while they are developing Rationale: The intestines remain within the umbilical cord only until approximately week 10. Intestine begin their development within the umbilical cord, but only because the liver and kidneys occupy most of the abdominal cavity. All the intestines are within the abdominal cavity around week 10. Blood supply is adequate in all areas. Intestines stay in the umbilical cord for approximately 10 weeks because they are growing faster than the abdomen. The abdominal contents grow more rapidly than the abdominal cavity, so part of their development takes place in the umbilical cord. By 10 weeks, the abdomen is large enough to contain them

The nurse is planning care for a patient with an ethnic background different from the nurse's. What approach by the nurse is best during the planning? A. Strive to keep ethnic background from influencing health needs. B. Encourage continuation of ethnic practices in the hospital setting. C. Attempt, in a nonjudgmental way, to change ethnic beliefs. D. Adapt the patient's ethnic practices as possible to the patient's health situation.

D. Adapt the patient's ethnic practices as possible to the patient's health situation. Rationale: Whenever possible, nursing care should facilitate the integration of ethnic practices into caring for the patient and supporting the patient's health needs. The ethnic background is part of the individual; it would be very difficult to eliminate the influence of the ethnic background. The ethnic practices need to be evaluated within the context of the health care setting to determine whether they are conflicting. The ethnic background is part of the individual; it would be very difficult to eliminate the influence of the ethnic background.

What routine nursing assessment is contraindicated in the patient admitted with suspected placenta previa? A. Monitoring FHR and maternal vital signs B. Observing vaginal bleeding or leakage of amniotic fluid C. Determining frequency, duration, and intensity of contractions D. Determining cervical dilation and effacement

D. Determining cervical dilation and effacement Rationale: Monitoring FHR and maternal vital signs is a necessary part of the assessment for this woman. Monitoring for bleeding and rupture of membranes is not contraindicated with this woman. Monitoring contractions is not contraindicated with this woman. Vaginal examination of the cervix may result in perforation of the placenta and subsequent hemorrhage

An Rh-negative woman (2-2-0-0-2) just gave birth to a healthy term Rh-positive baby boy. The direct and indirect Coomb's test results are both negative. The nurse should do which of the following? A. Prepare to administer Rho(D) immune globulin (RhoGAM) to the newborn within 24 hours of his birth B. Observe the newborn closely for signs of jaundice appearing within 24 hours of birth C. Recognize the RhoGAM is not needed because both Coomb's test results are negative D. Administer Rho(D) immune globulin intramuscularly to the woman within 72 hours of her baby's birth

D. Administer Rho(D) immune globulin intramuscularly to the woman within 72 hours of her baby's birth Rationale: RhoGAM should be administered to the mother within 72 hours of birth; pathologic jaundice is unlikely because Coombs' test results indicate that antibodies have not been formed to destroy the newborn's RBCs; RhoGAM is given to prevent formation of antibodies; it would not be given if antibodies have already been formed, as indicated by a positive Coombs' test

Which statement is true about the characteristics of a healthy family? A. The parents and children have rigid assignments for all the family tasks B. Young families assume the total responsibility for the parenting tasks, refusing any assistance C. The family is overwhelmed by the significant changes that occur as a result of childbirth. D. Adults agree on the majority of basic parenting principles

D. Adults agree on the majority of basic parenting principles Rationale: Healthy families remain flexible in their role assignments. Members of a healthy family accept assistance without feeling guilty. Healthy families can tolerate irregular sleep and meal schedules, which are common during the months after childbirth. Adults in a healthy family communicate with each other so that minimal discord occurs in areas such as discipline and sleep schedules

The function of the cremaster muscle in men is to A. Aid in voluntary control in excretion of urine B. Entrap blood in the penis to produce an erection C. Assist with transporting sperm D. Aid in temperature control of the testicles

D. Aid in temperature control of the testicles Rationale: The urinary meatus aids in controlling the excretion of urine. Entrapment of blood in the penis is a result of its spongy tissue. Seminal fluid assists with transporting sperm. A cremaster muscle is attached to each testicle. Its function is to bring the testicle close to the body to warm it or allow it to fall away from the body to cool it, thus promoting normal sperm production

What situation is most conducive to learning? A. a teacher who speaks very little Spanish is teaching a class of Latino students. B. A class is composed of students of various ages and educational backgrounds C. An auditorium is being used as a classroom for 300 students D. An Asian nurse provides nutritional information to a group of pregnant Asian women

D. An Asian nurse provides nutritional information to a group of pregnant Asian women Rationale: The ability to understand the language in which teaching is done determines how much the patient learns. Patients for who English is not their primary language may not understand idioms, nuances, slang terms, informal usage of words, or medical words. The teacher should be fluent in the language of the student. Developmental levels and educational levels influence how a person learns best. In order for the teacher to best present information, it is best for the class to be of the same levels. A large class is not conducive to learning. It does not allow for questions, and the teacher is not able to see nonverbal cues from the students to ensure understanding. A patient's culture influences the learning process; thus a situation that is most conducive to learning is one in which the teacher has knowledge and understanding of the patient's cultural beliefs

The nurse observes that when an 8 year old boy enters the playroom, he often causes disruption by taking toys from other children. The nurse's best approach for this behavior is to A. Ban the child from the playroom B. Explain to the other children in the playroom that he is very ill and should be allowed to have the toys C. Approach the child in his room and ask, Would you like it if the other children took your toys from you D. Approach the child in his room and state, I am concerned that you are taking the other children's toys. It upsets them and me

D. Approach the child in his room and state, I am concerned that you are taking the other children's toys. It upsets them and me Rationale: Banning the child from the playroom will not solve the problem. The problem is his behavior, not the place where he exhibits it. Illness is not a reason for a child to be undisciplined. When the child recovers, the parents will have to deal with a child who . is undisciplined and unruly. Children should not be made to feel guilty and have their self esteem attacked. By the nurses using I rather than the you message, the child can focus on the behavior. The child and the nurse can begin to explore why the behavior occurs.

Which question by the nurse will most likely promote sharing of sensitive information during a genetic counseling interview? A. How many people in your family are mentally retarded or handicapped? B. What kinds of defects or diseases seem to run in the family? C. Did you know that you can always have an abortion if the fetus is abnormal? D. Are there any family members who have learning or development problems?

D. Are there any family members who have learning or development problems? Rationale: This question assumes that there are genetic problems that result in intellectual disability in the family. Some individuals may find these terms offensive. Some individuals may not be aware of which diseases are genetically linked an may not answer the question accurately. This is taking the decision away from the parents. They are seeking counseling in order to prevent problems, not to find out what do to if there is a problem. The nurse should probe gently by using lay oriented terminology rather than direct questions or statements

A woman is trying to calm her fussy baby daughter in preparation for feeding. She exhibits a need for further instruction if she does which of the following? A. Swaddles the baby B. Dims light in the room and turns off the television C. Gently rocks the baby and talks to her in a low voice D. Attempts to get the baby to latch on immediately

D. Attempts to get the baby to latch on immediately Rationale: mother should be encouraged to let the newborn begin to suck on her clean finger until the baby begins to calm down then switch to the breast; A.B. and C. are al appropriate actions to calm a fussy baby

A laboring woman's uterine contractions are being internally monitored. When evaluating the monitor tracing, which of the following findings would be a source of concern and require further assessment? A. Frequency every 2 1/2 to 3 minutes B. duration of 80 to 85 seconds C. Intensity during a uterine contraction of 55 to 80 mmHg D. Average resting pressure of 25 to 30 mmHg

D. Average resting pressure of 25 to 30 mmHg Rationale: the average resting pressure should be 10mmHg; A, B, C are all findings within expected ranges

What should the nurse expect to be problematic for a family whose religious affiliation is Jehovah's Witness? A. Immunizations B. Autopsy C. Organ donation D. Blood transfusion

D. Blood transfusion Rationale: Christian Science believers may seek exemption from immunizations. Believers in Islam are opposed to organ donation. Jehovah's Witness believers can make individual decisions about autopsy. Jehovah's Witness believers are opposed to blood transfusions. They may accept alternatives to transfusions, such as non blood plasma expanders

Which of the following statements about injuries in childhood is false? A. Male deaths outnumber female deaths B. After 1 year of age, death rates in children are less than for infants C. Children ages 5 to 14 years have the lowest rate of death D. Caucasians have more deaths for all ages than Hispanics

D. Caucasians have more deaths for all ages than Hispanics

Between 6 and 10 days after conception, the trophoblast secretes enzymes that enable it to burrow into the endometrium until the entire blastocyte is covered. This is termed implantation. Tiny projections then develop out of the trophoblast and extend into the endometrium. These projections are referred to as A. Decidua basalis B. Decidua capsularis C. Decidua vera D. Chorionic villi

D. Chorionic villi Rationale: The deciduas basalis is the portion of the endometrium where the chorionic villi tap into the maternal blood vessels. The deciduas capsularis is the portion of the endometrium that covers the blastocyte. The portion of the endometrium that lines the rest of the uterus is called deciduas vera. These villi are vascular processes that obtain oxygen and nutrients from the maternal bloodstream and dispose of carbon dioxide and waste products into the maternal blood.

As a powerful central nervous system stimulant, which of these substances can lead to miscarriage, preterm labor, placental separation (abruption), and stillbirth? A. Heroin B. Alcohol C. PCP D. Cocaine

D. Cocaine Rationale: Heroin is an opiate. It's use in pregnancy is associated with preeclampsia, intrauterine growth restriction, miscarriage, premature rupture of membranes, infections, breech presentation, and preterm labor. The most serious effect of alcohol use in pregnancy is FAS. The major concerns regarding PCP use in pregnant women are its association with polydrug abuse and the neurobehavioral effects on the neonate. Cocaine is a powerful CNS stimulant. Effects on pregnancy associated with cocaine use include abrupto placentae, preterm labor, precipitous birth, stillbirth

The major source of nutrients in the diet of a pregnant woman should be composed of A. simple sugars B. Fats C. Fiber D. Complex carbohydrates

D. Complex carbohydrates Rationale: The most common simple carbohydrate is table sugar, which is a source of energy but does not provide any nutrients. Fats provide 9kcal in each gram, in contrast to carbohydrates and proteins, which provide only 4kcal in each gram. Fiber is supplied mainly by the complex carbohydrates. Complex carbohydrates supply the pregnant woman with vitamins, minerals, and fiber

Which factor is most important in diminishing maternal/fetal/neonatal complications in a pregnant woman with diabetes? A. The woman's stable emotional and psychological status B. Evaluation of retinopathy by an ophthalmologist C. Total protein excretion and creatinine clearance within normal limits D. Degree of glycemic control before and during the pregnancy

D. Degree of glycemic control before and during the pregnancy Rationale: The occurrence of complications can be greatly diminished by maintaining normal blood glucose levels before and during the pregnancy. Evaluating the woman's emotional status, ophthalmic examinations, and assessment of renal status are an important part of prenatal care, but not the most important factors to reduce complications.

The maternal task that begins in the first trimester and continues throughout the neonatal period is called A. seeking safe passage for herself and her baby B. Securing acceptance of the baby by others C. Learning to give of herself D. Developing attachment with the baby

D. Developing attachment with the baby Rationale: This is a task that ends with delivery. During this task the woman seeks health care and cultural practices. This process continues throughout pregnancy as the woman reworks relationships. This task occurs during pregnancy as the woman allows her body to give space to the fetus. She continues with giving to others in the form of food or presents. Developing attachment (strong ties of affection) to the unborn baby begins in early pregnancy when the woman accepts that she is pregnant. By the second trimester, the baby becomes real and feelings of love and attachment surge

Pregnant women who are obese are more likely to develop one or both of these two most frequently reported maternal risk factors. These factors include which of the following combinations? A. Premature labor and infection B. Hemorrhage and hypertension associated with pregnancy C. Infection and diabetes D. Diabetes and hypertension associated with pregnancy

D. Diabetes and hypertension associated with pregnancy Rationale: Obesity is associated with an increased risk for hypertension and diabetes

Which statement should a counselor make when telling a couple about thee prenatal diagnosis of genetic disorders? A. Diagnosis can be obtained promptly through most hospital laboratories B. Common disorders can quickly be diagnosed through blood tests C. A comprehensive evaluation will result in an accurate diagnosis D. Diagnosis may be slow and could be inconclusive

D. Diagnosis may be slow and could be inconclusive Rationale: Some tests must be sent to a special laboratory, which requires a longer waiting period for results. There is no quick blood test available at this time to diagnose genetic disorders. Despite a comprehensive evaluation, a diagnosis may never be established. Even the best offers at diagnosis do not always yield the information needed to counsel patients. They may require many visits over several weeks

When caring for a pregnant woman with suspected cardiomyopathy, the nurse must be alert for signs and symptoms of cardiac decompensation which include A. a regular heart rate and hypertension B. An increased urinary output, tachycardia, and dry cough C. shortness of breath, bradycardia, and hypertension D. Dyspnea, crackles; and an irregular weak pulse

D. Dyspnea, crackles; and an irregular weak pulse Rationale: These symptoms are not generally associated with cardiac decompensation. Of these symptoms, only tachycardia is indicative of cardiac decompensation. OF these symptoms, only dyspnea is indicative of cardiac decompensation. Signs of cardiac decompensation include dyspnea, crackles; an irregular, weak, rapid pulse; rapid respirations; a moist, frequent cough; generalized edema; increasing fatigue; and cyanosis of the lips and nail beds

An appropriate nursing measure when a baby has an unexpected anomaly is to A. remove thee baby from the delivery area immediately B. Tell the parents that the baby has to go to the nursery immediately C. Inform the parents immediately that something is wrong D. Explain the defect and show the baby to the parents as soon as possible

D. Explain the defect and show the baby to the parents as soon as possible Rationale: The parents should be able to touch and hold the baby as soon as possible (A) This would raise anxiety levels of the parents; they should be told about the defect and allowed to see the baby (B) They should be told immediately, but they should be told about the defect and allowed to see the infant. (C) Parents experience less anxiety when they are told about the defect as early as possible and are allowed to touch and hold the baby

The role of the pediatric nurse is influenced by trends in health care. What is the current trend in child health care? A. Primary focus on treatment of disease or disability B. National health care planning on a distributive or episodic basis C. Accountability to professional codes and international standards D. Focus on health promotion- the provision of care designed to keep people healthy and prevent illness

D. Focus on health promotion- the provision of care designed to keep people healthy and prevent illness Rationale: This is the current focus of health care in which nursing plays a major role. The treatment of disease or disability is traditionally the role of the physician. National health care planning is not a major trend. Accountability to professional codes and international standards is an established responsibility, not a trend

Which clinical sign is not included in the classic symptoms of preeclampsia? A. Hypertension B. Edema C. Proteinuria D. Glycosuria

D. Glycosuria Rationale: The first indication of preeclampsia is usually an increase in the maternal blood pressure. The first sign noted by the pregnant woman is a rapid weight gain and edema of the hands and face. Proteinuria usually develops later than the edema and hypertension. Spilling glucose into the urine is not one of the three classic symptoms of preeclampsia.

While you are assessing the vital signs of a pregnant woman in her third trimester, the patient complains of feeling faint, dizzy, and agitated. Which nursing intervention is appropriate? A. Have the patient stand up and retake her blood pressure B. Have the patient sit down and hold her arm in a dependent position C. Have the patient lie supine for 5 minutes and recheck her blood pressure on both arms D. Have the patient turn to her left side and recheck her blood pressure in 5 minutes

D. Have the patient turn to her left side and recheck her blood pressure in 5 minutes Rationale: Pressures are significantly higher when the patient is standing. This option causes. an increase in systolic and diastolic pressures. The arm should bee supported at the same level of the heart. The supine position may cause occlusion of the vena cava and descending aorta, creating hypotension. Blood pressure is affected by positions during pregnancy. The supine position may cause occlusion of thee vena cava and descending aorta. Turning the pregnant woman to a lateral recumbent position alleviates pressure on the blood vessels and quickly corrects supine hypotension

While the vital signs of a pregnant woman in her third trimester are being assessed, the woman, who is lying supine, complains of feeling faint, dizzy, and agitated. Which nursing intervention is appropriate? A. Have the patient stand up; retake her blood pressure. B. Have the patient sit down and hold her arm in a dependent position. C. Have the patient lie supine for 5 minutes; recheck her blood pressure on both arms. D. Have the patient turn to her left side; recheck her blood pressure in 5 minutes.

D. Have the patient turn to her left side; recheck her blood pressure in 5 minutes. Rationale: Blood pressure is affected by positions during pregnancy. The supine position may cause occlusion of the vena cava and descending aorta. Turning the pregnant woman to a lateral recumbent position alleviates pressure on the blood vessels and quickly corrects supine hypotension. Having the patient stand up would cause an increase in systolic and diastolic pressures. Having the patient hold her arm in a dependent position will cause a false reading.

According to Friedman's classifications, providing such physical necessities as food, clothing, and shelter is the _____ family function A. Economic B. Socialization C. Reproductive D. Health care

D. Health care Rationale: The economic function provides resources but is not concerned with health care and other basic necessities. The socialization function teaches the child cultural values. The reproductive function is concerned with ensuring family continuity. Physical necessities such as food, clothing, and shelter are considered part of health care

A 32 year old primigravida is admitted with a diagnosis of ectopic pregnancy. Nursing care is based on the knowledge that A. Bed rest and analgesics are the recommended treatment B. She will be unable to conceive in the future. C. A D&C will be performed to remove the products of conception D. Hemorrhage is a major concern

D. Hemorrhage is a major concern Rationale: The recommended treatment is to remove the pregnancy before hemorrhaging. If the tube must be removed, her fertility will decrease but she will not be infertile. A D&C is done on the inside of the uterine cavity. The ectopic is located within the tubes. Severe bleeding occurs if the fallopian tube ruptures

A woman has delivered twins. The first twin was stillborn, and the second is in the intensive care nursery and is recovering quickly from respiratory distress. The woman is crying softly and says, I wish my baby could have lived. What is the most therapeutic response? A. Don't be sad. At least you have one healthy baby B. How soon do you plan to have another baby? C. I have a friend who lost a twin and she's doing just fine now D. I am so sorry about your loss. Would you like to talk about it

D. I am so sorry about your loss. Would you like to talk about it Rationale: This is denying the loss of the other infant (A). This is denying the loss of the infant and her grief and belittling her feelings(B). This is belittling her feelings (C). The nurse should recognize the woman's grief and its significance

To determine the cultural influence on a patient's diet, the nurse should first: A. Evaluate the patient's weight gain during pregnancy B. Assess the socioeconomic status of the patient C. Discuss the four food groups with the patient D. Identify the food preferences and methods of food preparation common to that culture

D. Identify the food preferences and methods of food preparation common to that culture Rationale: Evaluating a patients weight gain during pregnancy should be included for all patients, not just from those who are culturally different. The socioeconomic status of thee patient may alter the nutritional intake, but not the cultural influence. Teaching the food groups to the patient should come after assessing food preferences. Understanding the patients food preferences and how she prepares food will assist the nurse in determining whether the patients culture is adversely affecting her nutritional intake

The most basic information a maternity nurse should have concerning conception is: A. Ova are considered fertile 48 to 72 hours after ovulation B. Sperm remain viable in the woman's reproductive system for an average of 12 to 24 hours C. Conception is achieved when a sperm successfully penetrates the membrane surrounding the ovum D. Implantation in the endometrium occurs 6 to 10 days after conception

D. Implantation in the endometrium occurs 6 to 10 days after conception Rationale: Ova are considered fertile for approximately 24 hours after ovulation. Sperm remain viable in the woman's reproductive system for an average of 2 to 3 days. Penetration of the ovum by the sperm is called fertilization. Conception occurs when the zygote, the first cell of the new individual, is formed. After implementation, the endometrium is called the decidua

In which situation is dilation and curettage (D&C) indicated? A. Complete abortion at 8 weeks B. Incomplete abortion at 16 weeks C. Threatened abortion at 6 weeks D. Incomplete abortion at 10 weeks

D. Incomplete abortion at 10 weeks Rationale: If all the products of conception have been passed (complete abortion), a D&C is not used. D&C is used to remove the product of conception from the uterus and can be done safely until week 14 of gestation. If the pregnancy is still viable (threatened abortion), a D&C is not used. D&C is used to remove the products of conception from the uterus and can be used safely until week 14 of gestation

The most dangerous effect on the fetus of a mother who smokes cigarettes while pregnant is A. Genetic changes and anomalies B. Extensive central nervous system damage C. Fetal addiction to the substance inhaled D. Intrauterine growth restriction

D. Intrauterine growth restriction Rationale: Cigarettes normally will not cause genetic changes. Cigarettes normally will not cause extensive central nervous system damage. Addiction is not a normal concern with a neonate. The major consequences of smoking tobacco during pregnancy are low birth weight infants, prematurity, and increased perinatal loss

Nurses should be aware of the strengths and limitations of various biochemical assessments during pregnancy, including that A. Chorionic villus sampling (CVS) is becoming more popular because it provides early diagnosis B. Screening for maternal serum alpha-fetoprotein (MSAFP) levels is recommended between 10 and 12 weeks of gestation in order to give parents time to consider options C. Percutaneous umbilical blood sampling (PUBS) is one of the multiple marker screen tests for Down syndrome D. MSAFP is a screening tool only; it identifies candidates for more definitive procedures

D. MSAFP is a screening tool only; it identifies candidates for more definitive procedures Rationale: CVS does provide a rapid result, but it is declining in popularity because of advances in noninvasive screening techniques. MSAFP screening is recommended for all pregnant women. Screening is recommended between 15 and 20 weeks of gestation. Abnormal findings give parents time to have additional tests done. This is correct, MSAFPis a screening tool, not a diagnostic tool. Further diagnostic testing is indicated after an abnormal MSAFP

A placenta previa in which the placental edge just reaches the internal os is called A. Total B. Partial C. Complete D. Marginal

D. Marginal Rationale: With a total placenta previa the placenta completely covers the os. With a partial previa the lower border of the placenta is within 3cm of the internal cervical os, but does not completely cover the os. A complete previa is termed total. The placenta completely covers the internal cervical os. A placenta previa that does not cover any part of the cervix is termed marginal

Infants exposed to rubella during the first trimester are at risk for which complication? A. Preterm birth B. Sepsis C. Meningitis D. Microcephaly

D. Microcephaly Rationale: If maternal infection occurs during this time, about 90% of fetuses will have CRS. Hearing loss, mental retardation, cataracts, cardiac defects, growth restriction, and microcephaly are common fetal complications. Rubella exposure is not associated with preterm birth, neonatal sepsis, or meningitis.

What form of heart disease in women of childbearing years usually has a benign effect on pregnancy? A. Cardiomyopathy B. Rheumatic heart disease C. Congenital heart disease D. Mitral valve prolapse

D. Mitral valve prolapse Rationale: Cardiomyopathy produces congestive heart failure during pregnancy. Rheumatic heart disease can lead to heart failure during pregnancy. Some congenital heart diseases will produce pulmonary hypertension or endocarditis during pregnancy. Mitral valve prolapse is a benign condition that is usually asymptomatic

Which diagnostic test evaluates the effect of fetal movement on fetal heart activity? A. Contraction stress test (CST) B. Sonography C. Biophysical profile D. Nonstress test (NST)

D. Nonstress test (NST) Rationale: CST evaluates the fetal reaction to contractions. Sonographic examinations visualize the fetus and are done for various reasons. The biophysical profile evaluates fetal status using many variables. An NST evaluates the ability of the fetal heart to accelerate either spontaneously or in association with fetal movement

When making a home visit, it is essential that the nurse use appropriate infection control measures. Of the measures listed below, which one is the most important? A. Including personal protective equipment in the home care bag B. Designating a dirty area with a trash bag to collect soiled equipment and supplies C. Wearing clean vinyl gloves for procedures that involve touching the patient D. Performing hand hygiene using either soap and running water or a self drying antiseptic solution

D. Performing hand hygiene using either soap and running water or a self drying antiseptic solution Rationale: although A and B are appropriate, they are not the most important; gloves are not needed unless the possibility of contacting body substances is present

The nurse should teach breastfeeding mothers about breast care measures to preserve the integrity of the nipples and areola. Which of the following should the nurse include in these instructions? A. Cleanse nipples and areola twice a day with mild soap and water B. Apply vitamin E cream to nipples and areola at least four times each day before a feeding C. Insert plastic lined pads into the bra to absorb leakage and protect clothing D. Place a nipple shell into the bra if the nipples are sore

D. Place a nipple shell into the bra if the nipples are sore Rationale: no soap should be used because it could dry the areola and increase the risk for irritation; vitamin E should not be used because it is a fat soluble vitamin that the infant could ingest when breastfeeding; lanolin or colostrum/milk are the preferred substances to bee applied to the area; plastic liners can trap moisture and lead to sore nipples

A woman had a chorionic villus sampling procedure. Prior to discharge the nurse should teach her to report what symptom that may be an indication of a complication? A. Lack of fetal movement B. Frequent urination C. Nausea and vomiting D. Vaginal bleeding or passage of amniotic fluid

D. Vaginal bleeding or passage of amniotic fluid Rationale: Vaginal bleeding or passage of amniotic fluid suggests possible miscarriage and should be reported. Chorionic villus sampling is done between 10 and 12 weeks of gestation. Fetal movement is not felt until about 16 weeks. Frequent urination, nausea, and vomiting are common symptoms of pregnancy during the first trimester and not an indication of procedural complications.

Glucose metabolism is profoundly affected during pregnancy because: A. Pancreatic function in the islets of Langerhans is affected by pregnancy B. The pregnant woman uses glucose at a more rapid rate than the nonpregnant woman C. The pregnant woman increases her dietary intake significantly D. Placental hormones are antagonistic to insulin, resulting in insulin resistance

D. Placental hormones are antagonistic to insulin, resulting in insulin resistance Rationale: Pancreatic functioning is not affected by pregnancy. The glucose requirements differ because of the growing fetus. The pregnant woman should increase her intake by. 200 calories a day. Placental hormones, estrogen, progesterone, and human placental lactogen (HPL) create insulin resistance. Insulin is also broken down more quickly by the enzyme placental insulinase.

Which nursing interventions are written correctly? A. Ambulate daily. B. Assist with breastfeeding. C. Force fluids. D. Provide 100 mL of fluids of choice every 2 hours while awake.

D. Provide 100 mL of fluids of choice every 2 hours while awake. Rationale: Nursing interventions are to be specific. The other examples are not specific. Vague interventions may be confusing and the level of care may be altered.

An NST in which two or more fetal heart rate (FHR) accelerations of 15 beats per minute (BPM) or more occur with fetal movement in a 20 minute period is termed A. Nonreactive B. Positive C. Negative D. Reactive

D. Reactive Rationale: A nonreactive result means that the heart rate did not accelerate during fetal movement. A positive result is not used with an NST. The CST uses positive as a result term. A negative result is not used with an NST. The CST uses negative as a result term. The NST Is reactive (normal) when two or more FHR accelerations of at least 15bpm (each with a duration of at least 15 seconds) occur in a 20 minute period

Mortality rates are calculated from a: A. survey of physicians B. sample of hospital records C. registry of all deaths D. Sample death certificates

D. Sample death certificates

While teaching an early pregnancy class, the nurse explains that the morula is a A. Fertilized ovum before mitosis begins B. Flattened disc shaped layer of cells within a fluid filled sphere C. Double layer of cells that becomes the placenta D. Solid ball composed of the first cells formed after fertilization

D. Solid ball composed of the first cells formed after fertilization Rationale: The fertilized ovum is called the zygote. This is the embryonic disc. it will help develop into the baby. The placenta is formed when two layers of cells: the trophoblast, which is the other portion of the fertilized ovum, and deciduas, which is the portion of the uterus where implantation occurs. The morula is so named because it resembles a mulberry. It is a solid ball of 12 to 16 cells that develops after fertilization

A woman with preeclampsia has a seizure. The nurse's primary duty during the seizure is to: A.Insert an oral airway B. Suction the mouth to prevent aspiration C.Administer oxygen by mask D. Stay with the patient and call for help

D. Stay with the patient and call for help Rationale: Insertion of an oral airway during seizure activity is no longer the standard of care. The nurse should attempt to keep the airway patent by turning the patients head to the side to prevent aspiration.; Once the seizure has ended, it may be necessary to suction the patient's mouth. Oxygen would be administered after the convulsion has ended. If a patient becomes eclamptic, the nurse should stay with her and call for help. Nursing actions during a convulsion are directed towards ensuring a patent airway and patient safety

The traditional diet of Asian women includes little meat or dairy products and may be low in calcium and iron. The nurse can help the woman increase her intake of these foods by A. Emphasizing the need for increased milk intake during pregnancy B. Suggesting she eat more hot foods during pregnancy C. Telling her husband that she must increase her intake of fruits and vegetables for the baby's sake. D. Suggest she eat more tofu, bok choy, and broccoli

D. Suggest she eat more tofu, bok choy, and broccoli Rationale: Since milk products are not part of this woman's diet, it should be respected and other alternatives offered. Also, lactose intolerance is common. Pregnancy is considered hot; therefore, the woman would eat cold foods. Fruits and vegetables are cold foods and should be included in the diet. In family dynamics, however, the husband does not dictate to the wife in this culture. The diet should be improved by increasing foods acceptable to the woman. These foods are common in the Asian diet and are good sources of calcium and iron

The Women, Infants, and Children (WIC) program provides: A. Well-child examinations for infants and children living at the poverty level B. Immunizations for high risk infants and children C. Screening for infants with developmental disorders D. Supplemental food supplies to low income women who are pregnant or breastfeeding

D. Supplemental food supplies to low income women who are pregnant or breastfeeding Rationale: Medicaids early and periodic Screening, Diagnosis, and Treatment program provides for well child examinations and for treatment of any medical problems diagnosed during such check ups. Children in the WIC program are often linked with immunizations, but that is not the primary focus of the program. Public law 99-457 provides financial incentives to states to establish comprehensive early intervention services for infants and toddlers with, or at risk for, developmental disabilities. WIC is a federal program that provides supplemental food supplies to low-income women who are pregnant or breastfeeding and to their children until age 5 years

In most states, adolescents who are not emancipated minors must have the permission of their parents before: A. Treatment for drug abuse B. Treatment for sexually transmitted diseases C. Accessing birth control D. Surgery

D. Surgery Rationale: Most states allow minors to obtain treatment for drug or alcohol abuse without parental consent. Most states allow minors to obtain treatment for STDs without parental consent. In most states, minors are allowed access to birth control without parental consent. If a minor receives surgery without proper informed consent, assault and battery charges against the care provider can result. This does not apply to an emancipated minor (a minor child who has the legal competency of an adult because of circumstances involving marriage, divorce, parenting of a child, living independently without parents, or enlistment in the armed services).

Which factor significantly contributed to the shift from home births to hospital births in the early 20th century? A. Puerperal sepsis was identified as a risk for labor and delivery B. Forceps were developed to facilitate difficult births C. The importance of early parental-infant contact was identified D. Technologic developments became available to physicians

D. Technologic developments became available to physicians Rationale: Puerperal sepsis has been a known problem for generations. In the late 19th century, Semmelweis discovered how it could be prevented with improved hygienic practices. The development of forceps to help physicians facilitate difficult births was a strong factor in the decrease of home births and increase of hospital births. Other important discoveries included chloroform, drugs to initiate labor, and the advancement of operative procedures such as cesarean birth. Unlike home births, early hospital births hindered bonding between parents and their infants. Technological developments were available to physicians, not lay midwives

The doctor has ordered meperidine (Demerol) 25mg IV q2-3hr prn for pain associated with labor. In fulfilling this order, the nurse should consider which of the following? A. Abstinence syndrome will occur if the woman is opioid dependent B. Dosage of the analgesic is too high for IV administration, necessitating a new order C. Maternal respiratory depression is more likely to occur than when compared with morphine D. The newborn should be observed for respiratory depression if birth occurs within 4 hours of the dose

D. The newborn should be observed for respiratory depression if birth occurs within 4 hours of the dose Rationale: as a pure opioid agonist, meperidine will not cause abstinence syndrome; a 25mg dose is appropriate for IV administration; this medication is a potent opioid agonist analgesic; while it can cause respiratory depression, that is more likely to occur with morphine

While assessing her patient, what does the nurse interpret a positive sign of pregnancy? A. Fetal movement felt by the woman. B. Amenorrhea C. Breast changes D. Visualization of fetus by ultrasound

D. Visualization of fetus by ultrasound Rationale: Fetal movement is a presumptive sign of pregnancy. Amenorrhea is a presumptive sign of pregnancy. Breast changes are a presumptive sign of pregnancy. The only positive signs of pregnancy are auscultation of fetal heart tones, visualization. of the fetus by ultrasound, and fetal movement felt by the examiner

Family centered care (FCC) describes safe, quality care that recognizes and adapts to both the physical and psychosocial need of the family. Which nursing practice coincides with the principles of FCC? A. The newborn is returned to the nursery at night so that the mother can receive adequate rest before discharge B. The father is encouraged to go home after the baby is delivered C. All patients are routinely placed on a fetal monitor D. The nurses assignments includes both mom and baby and increases the nurses responsibility for education

D. The nurses assignments includes both mom and baby and increases the nurses responsibility for education Rationale: In this model the infant usually stays with the mother in the labor/delivery/recovery (LDR) room throughout her hospital stay. The father or other primary support person is encouraged to stay with the mother and infant, and many facilities provide beds so that they can remain throughout the night. In this modes, the nurse uses selective technology rather than routine procedures. This includes electronic fetal monitoring and IV therapy. Family centered care (FCC) increases the responsibility of nurses. In addition to the physical care provided, nurses assume a major role in teaching, counseling, and supporting families

Which goal is most appropriate for the collaborative problem of wound infection? A. The patient will have a temperature of 98.6F within 2 days B. The patients fluid intake will be maintained at 1000mL per 8 hours C. The patient will not exhibit further signs of infection D. The patient will be monitored to detect therapeutic response to antibiotic therapy

D. The patient will be monitored to detect therapeutic response to antibiotic therapy Rationale: Monitoring a patients temperature is an independent nursing role. Intake and output is an independent nursing role. Monitoring for complications is an independent nursing role. In a collaborative problem, the goal should be nurse oriented and reflect the nursing interventions of monitoring or observing. In collaborative problems, other team members are involved for other duties, such as prescribing antibiotics

A woman who delivered her third child yesterday has just learned that her two school aged children have contracted chickenpox. What should the nurse tell her. A. The womans two children should be treated with acyclovir before she goes home from the hospital B. The baby will acquire immunity from the woman and will not be susceptible to chickenpox C. The children can visit their mother and sibling in the hospital as planned but must wear gowns and masks D. The woman must make arrangements to stay somewhere other than her home until the children are no longer contagious

D. The woman must make arrangements to stay somewhere other than her home until the children are no longer contagious Rationale: Acyclovir is used to treat varicella pneumonia. The baby is already born and has received immunities. If the mother never had chickenpox, she cannot transmit thee immunities to the baby. Varicella infection occurring in a newborn may be life threatening. Varicella (chickenpox) is highly contagious. Although the baby inherits immunity from the mother, it would not be safe to expose either the mother or the baby

Which statement is true regarding the quality assurance or incident report? A. The report assures the legal department that no problems exists B. Reports are a permanent part of the patients chart C. The nurses notes should contain, Incident report filed, and copy placed in chart D. This report is a form of documentation of an event that may result in legal action

D. This report is a form of documentation of an event that may result in legal action Rationale: The report is a warning to the legal department to be prepared for a potential legal action. Incident reports are not a part of the patient's chart. Incident reports are not mentioned in the nurse's notes. Documentation on the chart should include all factual information regarding the womans condition that would be recorded in any situation. Incident reports are not mentioned in the nurse's notes. The nurse completes an incident report when something occurs that might result in a legal action against the clinic or hospital or is a variance from the standard of care

One danger in using nonfood supplementation of nutrients is A. Increased absorption of all vitamins B. Development of pregnancy induced hypertension (PIH) C. Increased caloric intake D. Toxic effects on the fetus

D. Toxic effects on the fetus Rationale: Supplements do not have better absorption than natural vitamins and minerals. There is no relationship between supplements and PIH. Supplements do not contain calories. The use of supplements in addition to food may increase the intake of some nutrients to doses much higher than the recommended amounts. Overdoses of some nutrients have been shown to cause fetal defects

When explaining twin conception, the nurse points out that dizygotic twins develop from A. A single fertilized ovum and are always of the same sex B. A single fertilized ovum and may be the same sex of different sexes C. Two fertilized ova and are the same sex D. Two fertilized ova and may be the same sex or different sexes

D. Two fertilized ova and may be the same sex or different sexes Rationale: A single fertilized ovum that produces twins is called monozygotic. Monozygotic twins are always the same sex. Dizygotic twins are from fertilized ova and may or may not be the same sex. Dizygotic twins are two different zygotes, each conceived from a single ovum and a single sperm. They may be both male, both female, or one male and one female

Which vitamins or minerals can lead to congenital malformations of the fetus if taken in excess by the mother? A. Zinc B. Vitamin D C. Folic acid D. Vitamin A

D. Vitamin A Rationale: Zinc is vital to good maternity and fetal health and is highly unlikely to be consumed in excess. Vitamin A, taken in excess, causes a number of problems. An analog of Vitamin A appears in prescribed acne medications, which must not be taken during pregnancy. Vitamin D is vital to good maternity and fetal health and is highly unlikely to be consumed in excess. Vitamin A, taken in excess, causes a number of problems. An analog of vitamin A appears in prescribed acne medications, which must not be taken during pregnancy. Folic acid is vital to good maternity and fetal health and is highly unlikely to be consumed in excess. Vitamin A, taken in excess, causes a number of problems. An analog of vitamin A appears in prescribed acne medications, which must not be taken during pregnancy. Zinc, vitamin D, and folic acid are vital to good maternity and fetal health and are highly unlikely to be consumed in excess. Vitamin A, taken in excess, causes a number of problems. An analog of vitamin A appears in prescribed acne medications, which must not be taken during pregnancy

In her birth plan, a woman requests that she be allowed to use the new whirlpool bath during labor. When implementing this woman's request the nurse would do which of the following? A. Assist the woman to maintain a reclining position in the tub B. Tell the woman that she will need to leave the tub as soon as her membranes rupture. C. Limit her to no longer than 1 hour in the tub D. Wait to initiate the bath until the woman has entered the active phase of her labor

D. Wait to initiate the bath until the woman has entered the active phase of her labor Rationale: woman can and should change her position while in the bath, using lateral and hand and knees when indicated; as long as amniotic fluid is clear or only slightly meconium tinged, a whirlpool can continue; there is no limit to the time she can spend in the water- she can stay as long as she wishes

A pregnant couple has formulated a birth plan and is reviewing it with the nurse at an expectant parents class. Which aspect of their birth plan would be considered unrealistic and require further discussion with the nurse? A. My husband and I have agreed that my sister will be my coach since he becomes anxious with regard to medical procedures and blood. He will be nearby and check on me every so often to make sure everything is ok B. Wee pan to use the techniques taught in the Lamaze classes to reduce the pain experienced during labor C. We want the labor and birth to take place in a birthing room. My husband will come in the minute the baby is born D. We do not want the fetal monitor used during labor, since it will interfere with movement and doing effleurage

D. We do not want the fetal monitor used during labor, since it will interfere with movement and doing effleurage Rationale: This is an acceptable request for a laboring woman. Using breathing techniques to alleviate pain is a realistic part of a birth plan. Not all fathers are able to be present during the birth, however, this couple has made a realistic plan that works for their specific situation. Since monitoring is essential to assess fetal well being, it is not a factor that can be determined by the couple. The nurse should fully explain its importance. The option for intermittent electronic monitoring could be explored if this is a low risk pregnancy and as long as labor is progressing normally. The birth plan is a tool with which parents can explore their childbirth options; however, the plan must be viewed as tentative

Which pregnant woman should restrict her weight gain during pregnancy? A. Woman pregnant with twins B. Woman in early adolescence C. Woman shorter than 62 inches or 157cm D. Woman who was 20 pounds overweight before pregnancy

D. Woman who was 20 pounds overweight before pregnancy Rationale: A higher weight gain in twin gestations may help prevent low birth weights. Adolescents need to gain weight toward the higher acceptable range, which will provide for their own growth as well as for fetal growth. In the past women of short stature were advised to restrict their weight gain; however, evidence to support these guidelines has not been found. A weight gain of 5 to 9kg will provide sufficient nutrients for the fetus. Overweight and obese women should be advised to lose weight prior to conception in order to achieve the best pregnancy outcomes

Which combination of sex chromosomes is present in a female? A. XY B. XYY C. XXY D. XX

D. XX Rationale: An XY is the indication for a male. There are normally only two sex chromosomes. The combination of an X chromosome from each parent produces a female

A pregestational diabetic woman at 20 weeks of gestation exhibits the following: thirst, nausea and vomiting, abdominal pain, drowsiness, and increased urination. Her skin is flushed and dry and her breathing is rapid with a fruity odor. A priority nursing action when caring for this woman would be to: A. provide the woman with a simple carbohydrate immediately B. request an order for an antiemetic C. assist the woman into a lateral position to rest D. administer insulin according to the woman's blood glucose level

D. administer insulin according to the woman's blood glucose level Rationale: The woman is exhibiting signs of DKA; insulin is the required treatment, with the dosage dependent on blood glucose level; intravenous fluids may also be required; choice A is the treatment for hypoglycemia; choices B and C, although they may increase the woman's comfort, are not priorities

A nurse who speaks on behalf of a patient is acting in the role of a(n) A. researcher. B. collaborator. C. manager of care. D. advocate.

D. advocate. An advocate speaks on behalf of another. The nurse should be aware of the patient's wishes and needs and intercede for the patient. The nurse as a researcher investigates theoretic or practice issues in nursing. The nurse as collaborator works with other team members to coordinate and manage the patient's care. The nurse as manager supervises other health care workers as they provide patient care.

During a prenatal visit at 36 weeks of gestation, the nurse tested a woman's urine for glucose and protein. The results indicated a trace amount of glucose. The nurse's next action should be to A. retest the urine for accuracy. B. have the woman give another sample for retesting. C. report the results immediately to the physician so further testing can be preformed. D. consider this as a normal result for this stage of pregnancy.

D. consider this as a normal result for this stage of pregnancy. Rationale: Small amounts of glucose in the urine may indicate physiologic spilling that occurs during normal pregnancy, and further testing is not necessary. Larger amounts of glucose in the urine require further testing.

A Native American woman gave birth to a baby girl 12 hours ago. The nurse notes that the woman keeps her baby in the bassinet except for feeding and states that she will wait until she gets home to begin breastfeeding. The nurse recognizes that this behavior is most likely a reflection of A. embarrassment B. delayed attachment C. disappointment that thee baby is a girl D. cultural beliefs regarding the care of newborns

D. cultural beliefs regarding the care of newborns Rationale: Choices A,B, and C are incorrect interpretations based on the woman's culture and customs; Native Americans often use cradle boards and avoid handling their newborns often; they should not be fed colostrum

A pregnant woman has come to the emergency department with complaints of nasal congestion and epistaxis. What is the correct interpretation of these symptoms by the practitioner? A. These conditions are abnormal. Refer the patient to an ear,nose, and throat specialist B. Nasal stuffiness and nosebleeds are caused by a decrease in progesterone C. estrogen relaxes the smooth muscles in the respiratory tract, so congestion and epistaxis are within normal limits D. estrogen causes increased blood supply to the mucous membranes and can result in congestion and nosebleeds

D. estrogen causes increased blood supply to the mucous membranes and can result in congestion and nosebleeds Rationale: The patient should be reassured that these symptoms are within normal limits. No referral is needed at this time. Progesterone is responsible for the heightened awareness of the need to breathe in pregnancy. Progesterone levels increase during pregnancy. Progesterone affects relaxation of the smooth muscles in the respiratory tract. As capillaries become engorged, the upper respiratory tract is affected by the subsequent edema and hyperemia, which causes these conditions, seen commonly during pregnancy.

When preparing a woman for a pelvic examination, the nurse notices that she had undergone a genital mutilation. During the examination, the nurse needs to plan for the woman to A. feel embarrassed because of the mutilation. B. be comfortable with the examination. C. be concerned that a full examination will not be possible. D. experience pain and to make her as comfortable as possible.

D. experience pain and to make her as comfortable as possible. Rationale: Because the introitus is so small and there is scar tissue that is inelastic, the woman will experience pain with the examination. A full examination may be possible depending on the extent of the mutilation.

An older mother is least likely to be concerned about A. importance of having enough rest and sleep. B. peer support. C. family support. D. financial security.

D. financial security. Rationale: Older women are more financially secure and can afford better health care. After childbirth, mature primiparas need more time to recover and have less energy than their younger counterparts. They may find child care an exhausting experience for the first few weeks, particularly if they had a cesarean birth or other complications of pregnancy. Peer and family support may be less available for mature primigravidas.

A step in maternal role attainment that relates to the woman giving up certain aspects of her previous life is termed A. looking for a fit B. roleplaying C. fantasy D. grief work

D. grief work Rationale: This is when the woman observes the behaviors of mothers and compares them with her own expectations. Roleplaying involves searching for opportunities to provide care for infants in the presence of another person. Fantasies allow the woman to try on a variety of behaviors. This usually deals with how the child will look and the characteristics of the child. The woman experiences sadness as she realized that she must give up certain aspects of her previous self and that she can never go back

In the 19th century, the most common cause of infant death was A. tuberculosis B. cancer C. measles D. infectious diarrhea

D. infectious diarrhea Rationale: Infectious diarrhea was caused by contaminated milk. The mothers used milk from infected cows and did not have refrigeration to prevent further bacteria growth. Tuberculosis, cancer ad measles were not common causes of infant death

The most dangerous effect on the fetus of an adolescent woman who smokes cigarettes while pregnant is A. genetic changes and anomalies. B. extensive central nervous system damage. C. fetal addiction to the substance inhaled. D. intrauterine growth restriction.

D. intrauterine growth restriction. Rationale: Cigarette smoking is a cause of low birth weight. Teens are more likely to smoke during pregnancy than other maternal age-groups. There is no no relationship between smoking and genetic anomalies, CNS damage, or fetal addiction.

The major advantage of chorionic villus sampling (CVS) over amniocentesis is that it A. is not an invasive procedure B. does not require hospitalization C. Has less risk of spontaneous abortion D. is performed earlier in pregnancy

D. is performed earlier in pregnancy Rationale: It is an invasive procedure. The woman does have to be in a hospital setting for the fetus and her to be properly assessed during and after the procedure. The risk of an abortion is at the same level for both procedures. CVS is performed between 10 and 13 weeks of gestation, providing earlier results than anmiocentesis

A woman brings her two sons, ages 2 and 4, to the health clinic. She tells the nurse that they have been in the United States for only 1 year and they are homeless. Because of this history, the nurse will assess the children for infections and A. congenital defects B. allergies C. accelerated growth patterns D. malnutrition

D. malnutrition Rationale: Homeless women and children are at high risk for poor nutrition and exposure to various infections. Congenital defects or allergies would not be a result of homelessness. Most homeless children will have decelerated growth and development patterns

A 34 year old woman at 36 weeks of gestation has been scheduled for a biophysical profile. She asks the nurse why the test needs to be performed. The nurse would tell her that the test: A. determines how well her baby will breathe after it is born B. evaluates the response of the baby's heart rate to uterine contractions C. measures her baby's head size and length D. observes her baby's activities in utero to ensure that her baby is getting enough oxygen

D. observes her baby's activities in utero to ensure that her baby is getting enough oxygen Rationale: an amniocentesis with analysis of amniotic fluid for the L/S ratio and presence of phosphatidylglycerol (Pg) is used to determine pulmonary maturity; choice c refers to serial measurements of fetal growth using ultrasound

A 23-year-old postpartum woman is having trouble breastfeeding. Upon assessment, the nurse discovers that the woman does not have a let-down reflex. One reason for this may be that she is lacking the hormone A. progesterone. B. estrogen. C. prolactin. D. oxytocin.

D. oxytocin. Rationale: One purpose of oxytocin is to stimulate contractions in the breast muscles that will push the breast milk down toward the nipple. This is called the let-down reflex. Progesterone will cause a relaxation of the breast muscles if present but will not affect them if the hormone is lacking. Estrogen stimulates the growth of breast tissue in preparation for breastfeeding but does not stimulate the let-down reflex. Prolactin is important for breast milk production but not the let-down reflex.

Factors that may interfere with the family's ability to provide for the needs of its members include A. adequate financial resources. B. adequate family support. C. birth of a healthy infant. D. smoking and abuse of other substances.

D. smoking and abuse of other substances. Rationale: Unhealthy habits such as smoking and abuse of other substances may interfere with family functioning. Other factors include a lack of financial resources, absence of adequate family support, birth of an infant who needs specialized care, an ill child, and inability to make mature decisions that are necessary to provide care for the children.

An obese pregnant women with gestational diabetes is learning self-injection, the nurse evaluates that the women understands the instructions when she: A. washed her hands and put on a pair of clean gloves B. shook the NPH insulin vial vigorously to fully mix the insulin C. drew the NPH into her syringe first D. spread her skin taut and punctured the skin as a 90 degree angle

D. spread her skin taut and punctured the skin as a 90 degree angle Rationale: Washing hands is important but gloves are not necessary for self injection; vial should be gently rotated, not shaken; regular insulin should be drawn into the syringe first; because she is obese, a 90 degree angle with skin taut is recommended

During a childbirth class a woman asks the nurse, "I'm just 8 weeks pregnant. I know the placenta is not fully developed yet, so what is producing all the hormones I need?" The nurse will development her answer on the knowledge that A. the placenta is big enough at this point to produce the hormones necessary. B. extra hormones are not necessary for this stage of the pregnancy. C. the pituitary is working to secrete the extra hormones that are necessary until the placenta develops. D. the corpus luteum secretes the extra hormones necessary until the placenta develops.

D. the corpus luteum secretes the extra hormones necessary until the placenta develops. Rationale: LH causes the follicle to persist as a corpus luteum for about 12 days after ovulation. If conception occurs, the fertilized ovum secretes human chorionic gonadotropin that causes the corpus luteum to persist. The corpus luteum produces the extra estrogen and progesterone necessary to support the pregnancy. The placenta is not mature enough to produce the estrogen and progesterone necessary to support the pregnancy. The corpus luteum will do this until the placenta is mature. Extra estrogen and progesterone are necessary to support the pregnancy. The pituitary will secrete the LH necessary to keep the corpus luteum until conception occurs. After conception, the fertilized ovum will secrete human chorionic gonadotropin to keep the corpus luteum going. The corpus luteum secretes the extra hormones necessary for the pregnancy.


Kaugnay na mga set ng pag-aaral

Exam #4 - Dr. Klein - The University of North Alabama

View Set

CSC251-N801: Advanced JAVA Programming Attendance Quizzes

View Set

Unit 10 - Shoulder - Bones, Joints, Ligaments, Muscles

View Set

Chapter 11- prioritization, delegation, & assignment questions

View Set

Infectious and Non Infectious Diseases

View Set